study guide 3

¡Supera tus tareas y exámenes ahora con Quizwiz!

A patient drinks 8 oz of water. Which of the following is a correct conversion of the patient's intake?

- 240 mL * One fluid oz equals 30 mL; therefore, 8 fluid oz equals 240 mL.

hich of the following demonstrates the correct use of one of the Six Rights of Medication Administration?

- Administering a patient's medication by the route the provider has prescribed * The Six Rights of Medication Administration are the right medication, the right dose, the right patient, the right route, the right time, and the right documentation. Giving the medication by the route prescribed is indeed an application of the Six Rights of Medication Administration.

With which route of drug administration are there no barriers to absorption?

- Intravenous * The definition of absorption is the movement of a drug from its site of administration into the blood. With intravenous administration, the drug is injected directly into a vein. Thus any possible barriers to absorption are bypassed, and the drug is completely and instantaneously absorbed.

A patient who is receiving parenteral nutrition (PN) through a central venous catheter (CVC) has a air embolus. What would the nurse do first? 1. Have the patient perform a Valsalva maneuver 2. Clamp the Intravenous (IV) tubing to prevent more air from entering the line. 3. Have the patient take a deep breath and hold it. 4. Notify the healthcare provider immediately

1. Have the patient perform a Valsalva maneuver Turn the patient on his or her left side to prevent air from entering the left side of the heart. Then have the patient perform a Valsalva maneuver (holding the breath and "bearing down").

While performing an assessment of the integument system, the nurse notes the client's eyeballs are protruding and the upper eyelids are elevated. What term should the nurse use to document this finding? 1. Erythema 2. Cyanosis 3. Exophthalmos 4. Normocephalic

3. Exophthalmos

Which term should the nurse use to best describe the movement of air in and out of the patient's lungs? a. Ventilation b. Diffusion c. Respiration d. Perfusion

A

An 86 year old woman is admitted to the unit with chills and a fever of 104 degrees F. What physiological process explains why she is at risk for dyspnea? A) Fever increases metabolic demands requiring increased oxygen need. B) Blood glucose stores are depleted and the cells do not have energy to use oxygen. C) Carbon dioxide production increases due to hyperventilation. D) Carbon dioxide production decreases due to hypoventilation.

A (fever increases metabolic demands requiring increased oxygen need) (When the body cannot meet the increased oxygenation need, the increased metabolic rate causes breakdown of protein and wasting of respiratory muscles increasing the work of breathing.)

A 2-year-old child is ordered to have eardrops daily. Which action will the nurse take? a. Pull the auricle down and back to straighten the ear canal. b. Pull the auricle upward and outward to straighten the ear canal. c. Sit the child up for 2 to 3 minutes after instilling drops in ear canal. d. Sit the child up to insert the cotton ball into the innermost ear canal

ANS: A Children up to 3 years of age should have the auricle pulled down and back, children 3 years of age to adults should have the auricle pulled upward and outward. Solution should be instilled 1 cm (1/2 in) above the opening of the ear canal. The patient should remain in the side-lying position 2 to 3 minutes. If a cotton ball is needed, place it into the outermost part of the ear canal.

When giving CPR, compressions are causing the heart to pump blood into and out of the lungs to the body's organs. This movement of oxygenated blood is best described as which of the following? a. Ventilation b. Diffusion c. Respiration d. Perfusion

D

A client scheduled for a percutaneous transhepatic cholangiography (PTC) denies allergies to medication. What action by the nurse is best? a. Ask the client about shellfish allergies. b. Document this information on the chart. c. Ensure that the client has a ride home. d. Instruct the client on bowel preparation.

ANS: A PTC uses iodinated dye, so the client should be asked about seafood allergies, specifically to shellfish. Documentation should occur, but this is not the priority. The client will need a ride home afterward if the procedure is done on an outpatient basis. There is no bowel preparation for PTC.

A client is scheduled for a colonoscopy and the nurse has provided instructions on the bowel cleansing regimen. What statement by the client indicates a need for further teaching? a. "It's a good thing I love orange and cherry gelatin." b. "My spouse will be here to drive me home." c. "I should refrigerate the GoLYTELY before use." d. "I will buy a case of Gatorade before the prep."

ANS: A The client should be advised to avoid beverages and gelatin that are red, orange, or purple in color as their residue can appear to be blood. The other statements show a good understanding of the preparation for the procedure.

A nurse assesses a client who had a myocardial infarction and is hypotensive. Which additional assessment finding should the nurse expect? a. Heart rate of 120 beats/min b. Cool, clammy skin c. Oxygen saturation of 90% d. Respiratory rate of 8 breaths/min

ANS: A When a client experiences hypotension, baroreceptors in the aortic arch sense a pressure decrease in the vessels. The parasympathetic system responds by lessening the inhibitory effect on the sinoatrial node. This results in an increase in heart rate and respiratory rate. This tachycardia is an early response and is seen even when blood pressure is not critically low. An increased heart rate and respiratory rate will compensate for the low blood pressure and maintain oxygen saturations and perfusion. The client may not be able to compensate for long, and decreased oxygenation and cool, clammy skin will occur later.

The nurse is teaching about the process of passively moving water from an area of lower particle concentration to an area of higher particle concentration. Which process is the nurse describing? a. Osmosis b. Filtration c. Diffusion d. Active transport

ANS: A The process of moving water from an area of low particle concentration to an area of higher particle concentration is known as osmosis. Filtration is mediated by fluid pressure from an area of higher pressure to an area of lower pressure. Diffusion is passive movement of electrolytes or other particles down the concentration gradient (from areas of higher concentration to areas of lower concentration). Active transport requires energy in the form of adenosine triphosphate (ATP) to move electrolytes across cell membranes against the concentration gradient (from areas of lower concentration to areas of higher concentration).

A nurse is a preceptor for a nurse who just graduated from nursing school. When caring for a patient, the new graduate nurse begins to explain to the patient the purpose of completing a physical assessment. Which statement made by the new graduate nurse requires the preceptor to intervene? a."I will use the information from my assessment to figure out if your antihypertensive medication is working effectively." b."Nursing assessment data are used only to provide information about the effectiveness of your medical care." c."Nurses use data from their patient's physical assessment to determine a patient's educational needs." d."Information gained from physical assessment helps nurses better understand their patients' emotional needs."

ANS: B Nursing assessment data are used to evaluate the effectiveness of all aspects of a patient's care, not just the patient's medical care. Assessment data help to evaluate the effectiveness of medications and to determine a patient's health care needs, including the need for patient education. Nurses also use assessment data to identify patients' psychosocial and cultural needs.

A nurse assesses an older adult client who has multiple chronic diseases. The client's heart rate is 48 beats/min. Which action should the nurse take first? a. Document the finding in the chart. b. Initiate external pacing. c. Assess the client's medications. d. Administer 1 mg of atropine.

ANS: C Pacemaker cells in the conduction system decrease in number as a person ages, resulting in bradycardia. The nurse should check the medication reconciliation for medications that might cause such a drop in heart rate, then should inform the health care provider. Documentation is important, but it is not the priority action. The heart rate is not low enough for atropine or an external pacemaker to be needed.

A nurse assesses clients on a medical-surgical unit. Which client should the nurse identify as having the greatest risk for cardiovascular disease? a. An 86-year-old man with a history of asthma b. A 32-year-old Asian-American man with colorectal cancer c. A 45-year-old American Indian woman with diabetes mellitus d. A 53-year-old postmenopausal woman who is on hormone therapy

ANS: C The incidence of coronary artery disease and hypertension is higher in American Indians than in whites or Asian Americans. Diabetes mellitus increases the risk for hypertension and coronary artery disease in people of any race or ethnicity. Asthma, colorectal cancer, and hormone therapy do not increase risk for cardiovascular disease.

A nurse is preparing to perform a complete physical examination on a weak, older-adult patient with bilateral basilar pneumonia. Which position will the nurse use? a. Prone b. Sims' c. Supine d. Lateral recumbent

ANS: C Supine is the most normally relaxed position. If the patient becomes short of breath easily, raise the head of the bed. Supine position would be easiest for a weak, older-adult person during the examination. Lateral recumbent and prone positions cause respiratory difficulty for any patient with respiratory difficulties. Sims' position is used for assessment of the rectum and the vagina.

A nurse is preparing to administer an injection to a patient. Which statement made by the patient is an indication for the nurse to use the Z-track method? a. "I am allergic to many medications." b. "I'm really afraid that a big needle will hurt." c. "The last shot like that turned my skin colors." d. "My legs are too obese for the needle to go through."

ANS: C The Z-track is indicated when the medication being administered has the potential to irritate sensitive tissues. It is recommended that, when administering IM injections, the Z-track method be used to minimize local skin irritation by sealing the medication in muscle tissue. The Z-track method is not meant to reduce discomfort from the procedure. If a patient is allergic to a medication, it should not be administered. If a patient has additional subcutaneous tissue to go through, a needle of a different size may be selected.

A nurse is teaching the patient with mitral valve problems about the valves in the heart. Starting on the right side of the heart, describe the sequence of the blood flow through these valves. 1. Mitral 2. Aortic 3. Tricuspid 4. Pulmonic a. 1, 3, 2, 4 b. 4, 3, 2, 1 c. 3, 4, 1, 2 d. 2, 4, 1, 3

ANS: C The blood flows through the valves in the following direction: tricuspid, pulmonic, mitral, and aortic

The nurse observes edema in a patient who has venous congestion from right heart failure. Which type of pressure facilitated the formation of the patient's edema? a. Osmotic b. Oncotic c. Hydrostatic d. Concentration

ANS: C Venous congestion increases capillary hydrostatic pressure. Increased hydrostatic pressure causes edema by causing increased movement of fluid into the interstitial area. Osmotic and oncotic pressures involve the concentrations of solutes and can contribute to edema in other situations, such as inflammation or malnutrition. Concentration pressure is not a nursing term

2. Having misplaced a stethoscope, a nurse borrows a colleague's stethoscope. The nurse next enters the patient's room and identifies self, washes hands with soap, and states the purpose of the visit. The nurse performs proper identification of the patient before auscultating the patient's lungs. Which critical health assessment step should the nurse have performed? a.Running warm water over stethoscope b.Draping stethoscope around the neck c. Rubbing stethoscope with betadine d. Cleaning stethoscope with alcohol

ANS: D Bacteria and viruses can be transferred from patient to patient when a stethoscope that is not clean is used. The stethoscope should be cleaned before use on each patient with isopropyl alcohol. Running water over the stethoscope does not kill bacteria. Betadine is an inappropriate cleaning solution and may damage the equipment. Draping the stethoscope around the neck is not advised.

A patient has an order to receive 0.3 mL of U-500 insulin. Which syringe will the nurse obtain to administer the medication? a. 3-mL syringe b. U-100 syringe c. Needleless syringe d. Tuberculin syringe

ANS: D Because there is no syringe currently designed to prepare U-500 insulin, many medication errors result with this kind of insulin. To prevent errors, ensure that the order for U-500 specifies units and volume (e.g., 150 units, 0.3 mL of U-500 insulin), and use tuberculin syringes to draw up the doses. A 3 mL and U-100 can result in inaccurate dosing. A needleless syringe will not be acceptable in this situation.

A nurse assesses a client after administering a prescribed beta blocker. Which assessment should the nurse expect to find? a. Blood pressure increased from 98/42 mm Hg to 132/60 mm Hg b. Respiratory rate decreased from 25 breaths/min to 14 breaths/min c. Oxygen saturation increased from 88% to 96% d. Pulse decreased from 100 beats/min to 80 beats/min

ANS: D Beta blockers block the stimulation of beta1-adrenergic receptors. They block the sympathetic (fight-or-flight) response and decrease the heart rate (HR). The beta blocker will decrease HR and blood pressure, increasing ventricular filling time. It usually does not have effects on beta2-adrenergic receptor sites. Cardiac output will drop because of decreased HR.

The nurse is educating a patient who has recently been diagnosed with chronic obstructive pulmonary disease (COPD). The nurse explains how the gasses in the lungs move between the air spaces and the bloodstream. Which process is the nurse describing? a. Ventilation b. Diffusion c. Respiration d. Perfusion

B

The nurse is admitting a patient who is scheduled to undergo a cardiac catheterization. Which of the following allergies is most important for the nurse to assess before this procedure? A) Iron B) Iodine C) Aspirin D) Penicillin

B) Iodine The physician usually will use an iodine-based contrast to perform this procedure. Therefore it is imperative to know whether or not the patient is allergic to iodine or shellfish.

The exchange of oxygen and carbon dioxide during cellular metabolism is best described as which of the following? a. Ventilation b. Diffusion c. Respiration d. Perfusion

C

A patient is admitted with severe lobar pneumonia. Which of the following assessment findings would indicate that the patient needs airway suctioning? A. Coughing up thick sputum only occasionally B. Coughing up thin, watery sputum after nebulization C. Decreased ability to clear airway through couching D. Lung sounds clear only after coughing

C (Decreased Ability to clear airway through coughing) (impaired ability to cough up mucous due to weakness or very thick secretions indicates a need for suctioning)

Conditions such as shock and severe dehydration resulting from extracellular fluid loss cause:

Hypovolemia Conditions such as shock and severe dehydration cause extracellular fluid loss and reduced circulating blood volume (hypovolemia).

Nurse is administering aspirin 81 mg PO daily as prescribed. Med is schedules for 0800 hours. Which demonstrates proper use of 1 of the "6 Rights" of med. admin?

Nurse identifies pt. w/ name as written on med. admin record (WRONG) **documentation

Over which abdominal quadrant are bowel sounds most active and therefore easier to auscultate?

Right lower quadrant To the right of the umbilicus in the right lower quadrant is the ileocecal value. this is where the small intestine connects to the large intestine, and it is normally very active with bowel sounds. Many nurses begin listening here for that reason. For the average adult, you'll hear 5 to 30 bowel sounds per minute.

What is pharmacology?

The study that deals with chemicals that affect the body's functioning.

What is your primary goal in performing a comprehensive physical assessment?

To develop a plan of care. Remember the nursing process; assessment, diagnosis, planning, implantation, evaluation. Assessment is the first part of the process. It generates the database from which you will make nursing decisions. Your objective in interacting with patients is to identify their needs and concerns and help find solutions. That is the nursing process in action- and your map is the nursing care plan you establish for each patient. Analyzing and synthesizing data will provide the basics for each nursing diagnosis and for the selection of nursing interventions to manage actual or potential problems.

Nurse is preparing to administer an intradermal injection. What should the nurse do to ensure proper technique?

Use a tuberculin syringe w/ a 3/8- to 5/8-inch, 25- to 27-gauge needle

hyoid bone

a horseshoe-shaped bone at the base of the tongue; the only bone in the body not directly attached to another bone

The labor/delivery nurse is caring for a 33-year-old who is in labor with her first child. The patient complained to the nurse about the hemorrhoids that she has experienced during the last month of her pregnancy. She asks, "what can I do to prevent future problems with hemorrhoids?" What is the nurse's best response? a. "Hemorrhoids are caused by defecation of stools that are loose and watery." b. "You need to soften your stools by drinking plenty of fluids." c. "You should eat less carbohydrates." d. "There is nothing that you can do to prevent hemorrhoids."

b. soften your stools by drinking plenty of fluids

The nurse is planning care for a group of patients. Which task will the nurse assign to the nursing assistive personnel (NAP)? a.Performing the first postoperative pouch change b.Maintaining a nasogastric tube c.Administering an enema d.Digitally removing stool

c. administering an enema

The nurse is caring for patients with ostomies. In which ostomy location will the nurse expect very liquid stool to be present? a.Sigmoid b.Transverse c.Ascending d.Descending

c. ascending

A student nurse is caring for a patient who has dehydration as a result of diarrhea. Diarrhea is a result of abnormally fast peristalsis in what organ? a. Jejunum b. Stomach c. Duodenum d. Colon

d. Colon

A nurse will be administering several medications to a patient who is receiving enteral feedings through a small bore nasogastric tube. The nurse administers the medications correctly by:

infusing each medication by gravity and flushing with water before and after instillation. -Medications should be instilled via gravity, flushing before and after with water

trachea

the air passage extending from the throat and larynz to the main bronchi

Parotid gland

the largest of the three main parts of salivary glands, located on either side of the face, just below and in front of the ears

trapezius and sternocleidomastoid

paired muscles that allow movement and provide support to the head and neck

Functional UI may be caused by

poor motivation for continence or cognitive decline

positive nitrogen balance would occur in which condition

pregnancy

Proteinuria

protein in urin

A pt comes in with lower back pain and cystitis. What could possibly be wrong

pylenophritis

Bacteruria can lead to

pylenophritis and urosepsis

Urinalysis

random/first morning

UI involuntary loss of urine occurring at somewhat predictable intervals when pt reaches specific volume

reflex urinary incontinence

cricoid cartilage

ring-like cartilage forming the lower and back part of the larynx

cranial nerve XI

spinal accessory nerve-contraction of trapezius and sternocleidomastoid muscles.

UI involuntary leakage of small volumes associated with intra ab pressure

stress urinary incontinence

STOMA

surgical procedure diverting urine outside of body through ab wall

thyroid gland

the largest of the endocrine glands, situated in the front and sides of the neck just below the thyroid cartilage

.A patient with congestive heart failure reports experiencing increased urination when taking the prescribed medication. The nurse explains that which of the following is true?

the pt is probably taking a diuretic

adam's apple

the thyroid cartlage located just above the cricoid cartilage

UI caused by medical conditions that are treatable and reversible

transient incontinence

.A 34-year-old patient is being seen in the ED for complaints of severe flank pain lasting for 2 days. The ED physician suspects that the patient has hydroureter. Which of the following tests would the nurse expect the health care provider to order?

ultrasound renal bladder

A 57-year-old woman has been incontinent of urine for the past 2 months. Her health care provider has scheduled her to have a test to check for stress urinary incontinence. For which of the following tests should the nurse prepare the patient?

ultrasound renal bladder

Carries waste into bladder

ureter

UI involuntary leaking often associated with strong sense of urgency related to overreactive bladder

urge urinary incontinence

Contractions of the bladder during micturition compress the lower portionof the ureters to prevent _____

urinary back flow

The nurse is caring for a patient with pneumonia who has severe malnutrition. The nurse recognizes that, because of a nutritional status, the patient is at increased risk for: (select all that apply) 1. Heart Disease 2. Sepsis 3. Pleural Effusion 4. Cardiac arrhythmias 5. Diarrhea

2,3,4 Sepsis, pleural effusion, Cardiac arrhythmias Patients who are malnourished on admission are at greater risk of life-threatening complications such as arrhythmia, pleural effusions, sepsis, or hemorrhage during hospitalization.

Pt. is to receive 12.5 mg of prednisone (Deltasone) by mouth daily. med is available in 5 mg tablets. How many tablets should nurse administer for ea. dose?

2 tablets (WRONG) **2.5 tablets

What is Pharmacokinetics?

It is the effect of the body on the drug. It is the movement of drug molecules in the body in relation to the drug's absorption, distribution, metabolism, and excretion.

Nurse is preparing to instill antibiotic ear drops into toddler's ear. Which technique should be used?

Pull pt's auricle down and back to open canal when administering drops

The nurse is caring for a patient who exhibits labored breathing, using accessory muscles, and is coughing up pink frothy sputum. The patient has bilateral lung bases and diminished breath sounds. What are the priority nursing assessments for the nurse to perform prior to notifying the patient's health care provider? Select all that apply. A) SpO2 levels B) Amount, color and consistency of sputum production C) Fluid status D) Change in respiratory rate and pattern (select all that apply)

A B D

.A patient has dehydration. While planning care, the nurse considers that the majority of the patient's total water volume exists in with compartment? a. Intracellular b. Extracellular c. Intravascular d. Transcellular

ANS: A Intracellular (inside the cells) fluid accounts for approximately two thirds of total body water. Extracellular (outside the cells) is approximately one third of the total body water. Intravascular fluid (liquid portion of the blood) and transcellular fluid are two major divisions of the extracellular compartment

A nurse is teaching a patient about medications. Which statement from the patient indicates teaching is effective? a. "My parenteral medication must be taken with food." b. "I will rotate the sites in my left leg when I give my insulin." c. "Once I start feeling better, I will stop taking my antibiotic." d. "If I am 30 minutes late taking my medication, I should skip that dose."

ANS: B For daily insulin, rotate site within anatomical area. Rotating injections within the same body part (instrasite rotation) provides greater consistency in absorption of medication. Parenteral medication absorption is not affected by the timing of meals. Taking a medication 30 minutes late is within the 60-minute window of the time medications should be taken. Medications are usually stopped based on the provider's orders except in extenuating circumstances. With some medications, such as antibiotics, it is crucial that the full course of medication is taken to avoid relapse of infection

A nurse is a preceptor for a nurse who just graduated from nursing school. When caring for a patient, the new graduate nurse begins to explain to the patient the purpose of completing a physical assessment. Which statement made by the new graduate nurse requires the preceptor to intervene? a. "I will use the information from my assessment to figure out if your antihypertensive medication is working effectively." b. "Nursing assessment data are used only to provide information about the effectiveness of your medical care." c. "Nurses use data from their patient's physical assessment to determine a patient's educational needs." d. "Information gained from physical assessment helps nurses better understand their patients' emotional needs

ANS: B Nursing assessment data are used to evaluate the effectiveness of all aspects of a patient's care, not just the patient's medical care. Assessment data help to evaluate the effectiveness of medications and to determine a patient's health care needs, including the need for patient education. Nurses also use assessment data to identify patients' psychosocial and cultural needs

A nurse is teaching staff about the conduction of the heart. In which order will the nurse present the conduction cycle, starting with the first structure? 1. Bundle of His 2. Purkinje network 3. Intraatrial pathways 4. Sinoatrial (SA) node 5. Atrioventricular (AV) node a. 5, 4, 3, 2, 1 b. 4, 3, 5, 1, 2 c. 4, 5, 3, 1, 2 d. 5, 3, 4, 2, 1

ANS: B The conduction system originates with the SA node, the "pacemaker" of the heart. The electrical impulses are transmitted through the atria along intraatrial pathways to the AV node. It assists atrial emptying by delaying the impulse before transmitting it through the Bundle of His and the ventricular Purkinje network

A client is having an esophagogastroduodenoscopy (EGD) and has been given midazolam hydrochloride (Versed). The client's respiratory rate is 8 breaths/min. What action by the nurse is best? a. Administer naloxone (Narcan). b. Call the Rapid Response Team. c. Provide physical stimulation. d. Ventilate with a bag-valve-mask.

ANS: C For an EGD, clients are given mild sedation but should still be able to follow commands. For shallow or slow respirations after the sedation is given, the nurse's first action is to provide a physical stimulation such as a sternal rub and directions to breathe deeply. Naloxone is not the antidote for Versed. The Rapid Response Team is not needed at this point. The client does not need manual ventilation.

A nurse is preparing to give an intramuscular injection into the left ventrogluteal muscle. What should the nurse do to locate the appropriate site?

With the heel of the hand on the greater trochanter, point the index finger up toward the anterior superior iliac spine, extending the other fingers back along the iliac crest.

A 56-year-old patient, who has recently become postmenopausal, made an appointment with her health care provider for symptoms of an UTI. The patient has had three previously diagnosed UTIs in the past 4 months. She asks the nurse if this is a normal occurrence with postmenopausal women. What is the best response from the nurse?

Yes, because as women go through menopause, the lining of the urethra becomes more susceptible to infections.

sublingual gland

a salivary gland on either side of the tongue

Nurse teaches daughter of older adult pt. how to instill eyedrops in pt's R eye. What indicates that the daughter understands?

"I will pull down her lower eyelid and drop the med in."

Topics to Review:

- 6 Rights - NG tube -Pre-Admin Assessment -Inhaler Spacer -Dosage calculator

which of the following patients is exhibiting drug tolerance?

- A patient requires an increased dose of a medication to achieve continued therapeutic benefit. * As tolerance develops to a medication, a patient requires higher and higher doses of that medication to achieve the desired effect.

Which of the following represents the correct administration of the prescribed medication?

- Amoxicillin 1 g PO prescribed; two 500-mg tablets given * To determine the correct dosage, start with the amount prescribed: 1 g (gram). To determine how many tablets to give, divide the dose ordered by the dose on hand and multiply the result by the amount on hand. So, 1 g (dose ordered, and also equivalent to 1,000 mg) divided by 500 mg (dose on hand) = 2, then 2 X 1 (amount on hand) = 2 tablets. So this is the correct amount to give.

You have a handwritten medication order that is difficult to read. Which of the following is the most appropriate action to take to avoid an error in medication administration?

- Call the medical provider for clarification of the order. * There is no other way to be sure about what was intended other than confirming it with the person who wrote the order.

You are reading the physician's orders and note date and time of the prescriptions, as well as the physician's signature. Which of the following prescriptions is complete?

- Digoxin (Lanoxin) 1.25 mg PO daily * This order is complete with medication dose, the route, and the frequency of administration.

Which of the following is your highest priority action for ensuring overall safety during medication administration?

- Identify the patient by two acceptable methods. * One of the six rights of medication administration is to identify that you are giving the medication to the correct patient. It is required that you check the medication administration record against the patient's identification bracelet, and use a second method of patient identification, such as asking the patient his birth date.

Which of the following is the most appropriate documentation of a patient's response to a pain medication?

- The patient reports pain decreased to 3/10, 30 minutes after medication administration. * Using a standardized instrument is the most appropriate method of documenting a response to pain medicatio

An uncommon, unexpected, or individual drug response thought to result from a genetic predisposition is called

- an idiosyncratic effect. * An idiosyncratic effect is an uncommon, unexpected, or individual drug response thought to result from a genetic predisposition.

You are giving a patient several PO medications to take. The patient tells you that she can only take one pill at a time. It is appropriate to

- remain at the bedside until you are sure the patient has taken all of the medications. * It is your responsibility to remain with the patient and observe that she has swallowed each medication. It is unacceptable to leave medications unattended for any period of time.

A drug's generic name is the

- same as its nonproprietary name. * A drug's generic name is its nonproprietary or noncommercial name. Each drug has only one generic name. For example, acetaminophen is the generic name for the drug marketed as Tylenol, while ibuprofen is the generic name for the drugs Advil, Motrin, and others.

You are caring for two patients and have just completed giving medications to the first when you realize that you gave her the medications ordered for your other patient. What should you do?

...

You are scheduled to give an intramuscular injection in the ventrogluteal site and remember learning that it should be administered using the Z-track technique. When you mention this to the nurse in the hospital who has been taking care of your patient, she tells you to be sure to remember the air bubble. You remember your instructor telling you not to use an air bubble. What should you do?

...

The nurse is preparing for morning rounds. What should the nurse avoid delegating to unlicensed assistive personnel? 1. Vital signs 2. Filling of water pitchers 3. Skull and face assessment 4. Ambulation of surgical clients

3. Skull and face assessment

A Pt is about to receive 30 mg of ketoralac (Toradol) IM/6 hr for 48hr. The medication is available in a 60mg/2mL vial. How many mL should the nurse administer for each dose?

1 mL 30mg / 60mg = 0.5 x 2mL = 1mL

A client has been receiving a new medication to address specific symptoms. The nurse will perform a physical examination to determine? Select all that apply. 1. the progress of the client's health problem. 2. the physiological impact of the prescribed medication. 3. baseline data. 4. data to support nursing diagnoses. 5. areas for health promotion.

1, 2

The nurse is assessing the musculoskeletal status of a 4-year-old child. What findings should the nurse consider as being expected in this client? Select all that apply. 1. Lordosis 2. Genu valgus 3. Genu varum 4. Pronation of the feet 5. Asymmetric leg abduction

1, 2

The nurse is assessing the nose and sinuses of a client. Which findings should the nurse identify as being within normal limits? Select all that apply. 1. Nose straight 2. Nares symmetrical 3. No tenderness over the bridge 4. Air movement restricted in one nare 5. Clear drainage from one nare

1, 2, 3

The nurse is preparing to assess a client with the Glasgow Coma Scale. Which areas is the nurse assessing in this patient? Select all that apply. 1. Eye response 2. Motor response 3. Verbal response 4. Orientation 5. Musculoskeletal response

1, 2, 3

The nurse is preparing to complete a physical examination on a client. What should the nurse realize as being the purpose for this examination? Select all that apply. 1. Obtain baseline data. 2. Obtain data to help determine nursing diagnoses. 3. Identify areas for disease prevention. 4. Identify the client's employment status. 5. Obtain data about the client's leisure activities.

1, 2, 3

The nurse is utilizing the technique of inspection during a physical examination with a client. When using this technique, the nurse will take which actions? Select all that apply. 1. Visually observe a body area. 2. Obtain information through the sense of smell. 3. Obtain information through the sense of hearing. 4. Examine the body through the use of touch. 5. Strike the body to elicit a sound from a body part.

1, 2, 3

A client is experiencing abdominal pain. What assessments should the nurse perform to assess this complaint? Select all that apply. 1. Inspect the abdomen. 2. Auscultate the abdomen. 3. Palpate the abdomen. 4. Assess vital signs. 5. Assess peripheral pulses.

1, 2, 3, 4

The nurse is planning to perform indirect percussion on an area of a client's body during a physical examination. Which actions should the nurse take to use this assessment technique? Select all that apply. 1. Place the middle finger of the nondominant hand on the client's skin. 2. Use the tip of the flexed middle finger of the other hand to strike the middle finger of the nondominant hand. 3. Perform a striking motion by moving the wrist. 4. Perform short, rapid, firm blows. 5. Use a stethoscope to transmit sounds to the ears.

1, 2, 3, 4

The nurse is preparing to conduct an assessment of the heart. Where should the nurse place the stethoscope to auscultate heart sounds? Select all that apply. 1. Aortic region 2. Pulmonic region 3. Tricuspid valve region 4. Abdomen 5. Mitral valve region

1, 2, 3, 5

The nurse is preparing to perform an eye assessment. What equipment should the nurse have available to complete this assessment? Select all that apply. 1. Penlight 2. Snellen's chart 3. Sterile gloves 4. Gauze square 5. Millimeter ruler

1, 2, 4, 5

The nurse is teaching a program on healthy nutrition at the senior community center. Which points should be included in the program for older adults? (select all that apply) 1. Avoid grapefruit and grapefruit juice, which impair drug absorption. 2. Increase the amount of carbohydrates for energy. 3. Take a multivitamin that includes vitamin D for bone health. 4. Cheese and eggs are good sources of protein. 5. Limit fluids to decrease the risk of edema.

1, 3, 4 1. Avoid grapefruit and grapefruit juice, which impair drug absorption. 3. Take a multivitamin that includes vitamin D for bone health. 4. Cheese and eggs are good sources of protein. Cheese, eggs, and peanut butter are also useful high-protein alternatives. Vitamin D supplements are important for improving strength and balance, strengthening bone health, and preventing bone fractures and falls. Grapefruit and grapefruit juice can interfere with warfarin (Coumadin) (anticoagulant), preventing its breakdown. This would lead to an increased risk of bleeding.

The nurse is educating the patient and his family about the parenteral nutrition. Which aspect related to this form of nutrition would be appropriate to include? (select all that apply.) 1. The purpose of the fat emulsion in parenteral nutrition is to prevent a deficiency in essential fatty acids. 2. We can give you a parenteral nutrition through your peripheral intravenous line to prevent further infection. 3. The fat emulsion will help control hyperglycemia during periods of stress. 4. The parenteral nutrition will help your wounds heal. 5. Since we just started the parenteral nutrition, we will only infuse it at 50% of your daily needs of the next 6 hours.

1,3,4 1. The purpose of the fat emulsion in parenteral nutrition is to prevent a deficiency in essential fatty acids. 3. The fat emulsion will help control hyperglycemia during periods of stress. 4. The parenteral nutrition will help your wounds heal. Sometimes adding intravenous fat emulsions to parenteral nutrition supports the patient's need for supplemental kilocalories, prevents essential fatty acid deficiencies, and helps control hyperglycemia during periods of stress. Parenteral nutrition is administered at 50% of the patient's daily needs for the first 24 hours to assess how he or she is tolerating the infusion.

The nurse is preparing a client for an abdominal examination. What should the nurse prior to beginning the examination? 1. Ask the client to urinate. 2. Ask the client to drink 8 ounces of water. 3. Assess vital signs. 4. Assess heart rate.

1. Ask the client to urinate.

The nurse is preparing to conduct a mental status assessment. What should the nurse include in this assessment? 1. Cognitive and affective functions 2. Cognitive and effective functions 3. Affective and memory functions 4. Affective and knowledge functions

1. Cognitive and affective functions

.Patients with urinary incontinence are unable to completely empty their bladder. The nurse can assist a patient to void by using which of the following methods? (Select all that apply.)

1. Completing manual bladder compression 2.Having the patient assume the normal position for voiding 3.Running water in the sink

The nurse is assessing the peripheral vascular status of an older client. Which finding should the nurse consider as being normal for this client? 1. Easy to palpate upper extremity arteries 2. Easy to palpate lower extremity arteries 3. Reduction in the number of varicosities 4. Increase in diastolic blood pressure

1. Easy to palpate upper extremity arteries

The nurse is performing a musculoskeletal assessment on a client admitted with a possible stroke. When testing for muscle grip strength, the nurse should ask the client to perform which action? 1. Grasp the nurse's index and middle fingers while the nurse tries to pull the fingers out. 2. Hold an arm up and resist while the nurse tries to push it down. 3. Flex each arm and then try to extend it against the nurse's attempt to keep the arm in flexion. 4. Shrug the shoulders against the resistance of the nurse's hands.

1. Grasp the nurse's index and middle fingers while the nurse tries to pull the fingers out.

The nurse is planning a physical examination of a client following a head-to-toe format. In which order should the nurse conduct this assessment? 1. Head, upper extremities, abdomen, lower extremities 2. Neck, head, vital signs, chest and back 3. Lower extremities, abdomen, upper extremities, chest and back 4. Head, neck, lower extremities, abdomen

1. Head, upper extremities, abdomen, lower extremities

The nurse is assisting the physician who is preparing to test a sexually active female client for cervical cancer. What should the nurse expect the health care provider to perform? 1. Pap test 2. Breast exam 3. Rectal exam 4. Abdominal exam

1. Pap test

The proper needle length when giving an intramuscular injection in the ventrogluteal area to an average-sized adult is ... ?

1.5 inches in ventrogluteal area

When suctioning secretions that are collecting in an endotracheal tube, the nurse does not apply suction for longer than:

15 seconds Applying suction for too long can result in complications such as hypoxemia and cardiac dysrhythmias. Thus the nurse is always aware of the length of time that suctioning is applied to an airway. If the suctioning time is too short, the suction catheter may not remove the secretions. If the suctioning time is too long, hypoxemia and/or cardiac dysrhythmias could result.

A client with known chronic obstructive pulmonary disease (COPD) is admitted to the emergency department with multiple minor injuries following an automobile accident. To ensure adequate ventilation the nurse applies a nasal cannula providing oxygen at what rate and for what reason?

2 L/min to prevent elevating the arterial oxygen tension (PaO2), which would suppress the hypoxic drive

The nurse would delegate which of the following to nursing assistive personnel (NAP)? (Select all that apply.) 1. Repositioning and retaping a patient's nasogastric tube 2. Performing glucose monitoring every 6 hours on a patient 3. Documenting PO intake on a patient who is on a calorie count for 72 hours 4. Administering enteral feeding bolus after tubing placement has been verified 5. Hanging a new bag of enteral feeding

2, 3 2. Performing glucose monitoring every 6 hours on a patient 3. Documenting PO intake on a patient who is on a calorie count for 72 hours The skills of measuring blood glucose level after skin puncture (capillary puncture) and writing down the amount the patient ate can be delegated to NAP. The nurse needs to administer enteral feeding because of the risk of aspiration. The nasogastric tube should never be repositioned by the NAP for risk of causing injury to the patient.

Which patients are at high risk for nutritional deficits? (Select all that apply.) 1. The divorced computer programmer who eats precooked food from a local restaurant. 2. The middle-aged female with celiac disease who does not follow her gluten-free diet 3. The 45-year-old patient with type II diabetes who monitors her carbohydrate intake and exercises regularly 4. The 25-year-old patient with Crohn's disease who follows a strict diet but does not take vitamins or iron supplements 5. The 65-year-old patient with gallbladder disease whose electrolyte, albumin, and protein levels are normal

2, 4 2. The middle-aged female with celiac disease who does not follow her gluten-free diet 4. The 25-year-old patient with Crohn's disease who follows a strict diet but does not take vitamins or iron supplements Patients suffering from celiac disease or Crohn's disease need to take vitamin and iron supplements regularly because they have a deficit resulting from malabsorption.

The nurse is concerned that an older client has nutritional deficiencies. What did the nurse find when assessing this client's nails to make this clinical decision? Select all that apply. 1. White spots 2. Curved nails 3. Deep purple areas 4. Spoon-shaped nails 5. Bands across the nails

2, 4, 5

While performing a health assessment, in which position should the nurse place the client for inspection of the jugular veins? 1. 90-degree angle 2. 30- to 45-degree angle 3. 15-degree angle 4. 60-degree angle

2. 30- to 45-degree angle

The nurse is preparing to administer a cardiotonic drug to a client. Which assessment should the nurse perform before administering the medication? 1. Respiratory rate 2. Apical pulse 3. Popliteal pulse 4. Capillary blanch test

2. Apical pulse

A patient is receiving total parenteral nutrition (TPN). What is the primary intervention the nurse should follow to prevent a central line infection? 1. Institute isolation precautions 2. Clean the central line port through which the TPN is infusing with antiseptic 3. Change the TPN tubing every 24 hours 4. Monitor glucose levels to watch and assess for glucose intolerance

2. Clean the central line port through which the TPN is infusing with antiseptic Use either alcohol or an alcoholic solution of chlorhexidine gluconate to clean the injection port or catheter hub 15 seconds before and after each time it is used to reduce the risk of a central line infection.

The nurse sees the nursing assistive personnel (NAP) perform the following intervention for a patient receiving continuous enteral feedings. Which action would require immediate attention? 1. Fastening tube to the gown with new tape 2. Placing patient supine while giving a bath 3. Hanging a new container of enteral feeding 4. Ambulating patient with enteral feeding

2. Placing patient supine while giving a bath A patient receiving continuous enteral feedings should never be placed supine because it increases the risk for pulmonary aspiration. If the nurse needs to lay the patient in the supine position, the feedings should be stopped and restarted when the head of the bed is at 45 degrees.

The nurse is performing a health assessment and notes a yellow tinge to the sclera of the eye. The nurse should document this as being 1. cyanosis. 2. jaundice. 3. pallor. 4. erythema.

2. jaundice.

A patient is to recieve 12.5 mg of prednisone (Deltasone) by mouth daily. The medication is available in 5 mg tablets. How many tablets should the nurse administer for each dose?

2.5 tablets -Divide the dose ordered by the dose on hand.

The nurse is preparing to assess a client's reflexes. What equipment should the nurse gather before entering the room? 1. Sterile gloves 2. Clean gloves 3. Percussion hammer 4. Penlight

3. Percussion hammer

The nurse is performing a lung assessment on a client with suspected pneumonia. Which finding should the nurse report to the physician immediately? 1. Chest symmetrical 2. Breath sounds equal bilaterally 3. Asymmetrical chest expansion 4. Bilateral symmetric vocal fremitus

3. Asymmetrical chest expansion

The patient's blood glucose level is 330 mg/dL. What is the priority nursing intervention? 1. Recheck by performing another blood glucose test 2. Call the primary health care provider. 3. Check the medial record to see if there is a medication order for abnormal glucose levels. 4. Monitor and recheck in 2 hours.

3. Check the medial record to see if there is a medication order for abnormal glucose levels. Check the medical record to see if there is a medication order for deviations in glucose level; if not, notify the health care provider. As the nurse you want to get the patient's blood sugar as close to normal as possible.

During the assessment of a client's breasts, the nurse finds both breasts rounded, slightly unequal in size, skin smooth and intact, and nipples without discharge. What should the nurse do next? 1. Notify the charge nurse. 2. Notify the physician. 3. Document the findings in the nurse's notes as normal. 4. Document the findings in the nurse's notes as abnormal.

3. Document the findings in the nurse's notes as normal.

The nurse is preparing to perform a health assessment of the abdomen. In which order should the nurse perform the assessment? 1. Auscultate, percuss, palpate, inspect 2. Inspect, auscultate, palpate, percuss 3. Inspect, auscultate, percuss, palpate 4. Palpate, percuss, auscultate, inspect

3. Inspect, auscultate, percuss, palpate

The nurse is assessing peripheral pulses on a client with suspected peripheral vascular disease. Which finding should the nurse report to the physician immediately? 1. Pulses equal bilaterally 2. Full pulsations 3. Thready pulses 4. Pulses present bilaterally

3. Thready pulses

The nurse is preparing the morning assignments. Which assessment could the nurse delegate to unlicensed assistive personnel? 1. Neurological assessment 2. Musculoskeletal assessment 3. Vital signs assessment 4. Female genital assessment

3. Vital signs assessment

Normal urine capactiy

300-600 mL

A patient's gastric residual volume was 250 mL @ 0800 and 350 mL @ 0900. What is the appropriate nursing action? 1. Assess bowel sounds 2. Raise the head of the bed to at least 45 degrees 3. Position the patient on his or her right side to promote stomach emptying 4. Do not reinstall aspirate and hold the feeding until you talk to the primary care provider.

4. Do not reinstall aspirate and hold the feeding until you talk to the primary care provider. Do not administer feeding when a single gastric residual volume exceeds 500 mL or when two consecutive measurements (taken 1 hour apart) each exceed 250 mL because of the potential for aspiration.

What is the correct sequence for suctioning a patient?1. Open kit and basin.2. Apply gloves.3. Lubricate catheter.4. Verify functioning of suction device and pressure.5. Connect suctioning tubing to suction catheter.6. Increase supplemental oxygen.7. Reapply oxygen.8. Suction airway.

4 6 1 3 2 5 8 7

The nurse is caring for a client following a cerebrovascular accident (stroke). The client is able to comprehend what is being said to him; however, he is unable to respond by speech or writing. What type of aphasia should the nurse realize this patient is demonstrating? 1. Auditory aphasia 2. Acoustic aphasia 3. Sensory aphasia 4. Expressive aphasia

4. Expressive aphasia

Which statement made by a patient of a 2-month-old infant requires further education? 1. I'll continue to use formula for the baby until he is a least a year old. 2. I'll make sure that I purchase iron-fortified formula. 3. I'll start feeding the baby cereal at 4 months. 4. I'm going to alternate formula with whole milk starting next month.

4. I'm going to alternate formula with whole milk starting next month. Infants should not have regular cow's milk during the first year of life. It is too concentrated for the infant's kidneys to manage. There is also an increased risk for developing milk-product allergies.

The nurse ovulated which laboratory values to assess a patient's potential for wound healing? 1. Fluid status 2. Potassium 3. Lipids 4. Nitrogen balance

4. Nitrogen balance Nitrogen balance is important to determining serum protein status. A negative nitrogen balance is present when catabolic states exist. When a patient has a decreased protein level, he or she is at risk for delayed wound healing.

The nurse caring for a patient with dysphagia and is feeding her a pureed chicken diet when she begins to choke. What is the priority nursing intervention? 1. Suction her mouth and throat 2. Turn her on their side 3. Put on oxygen at 2-L nasal cannula 4. Stop feeding her and place on NPO

4. Stop feeding her and place on NPO Stop feeding and place patient on NPO. If choking persists, suction airway. Notify health care provider.

A patient is receiving both parenteral (PN) and enteral nutrition (EN). When would the nurse collaborate with the health care provider and request discontinuing parental nutrition? 1. When 25% of the patient's of the patient's nutritional needs are met by the tube feedings. 2. When bowel sounds return 3. When central line has been in for 10 days 4. When 75% of the patient's nutritional needs are met by the tube feedings.

4. When 75% of the patient's nutritional needs are met by the tube feedings When meeting 75% of nutritional needs by enteral feedings or reliable dietary intake, it is usually safe to discontinue PN therapy.

The nurse is inserting a small-bore nasoenteric tube before starting enteral feedings. Place the following steps in order to perform this procedure. 1. Place patient is high-fowler's position 2. Have patient flex head toward chest. 3. Assess patient's gag reflex. 4. Determine length of the tube to be inserted. 5. Obtain radiological confirmation of the tube placement. 6. Check pH of gastric aspirate for verifying placement. 7. Identify the patient with two identifiers.

7, 1, 3, 4, 2, 5, 6

At a recent staff meeting the staff educator discussed the importance of catheter care for the prevention of urinary tract infections (UTI). What percentage of health careassociated infections result from indwelling catheter use?

80%

A patient has been admitted to the cardiac unit with the diagnosis of bradycardia. The patient states "I am confused about what the doctor said is wrong with me, he said my pacemaker is not working. I don't have a pacemaker." What is the nurse's best response? a. "The sinoatrial node is the 'pacemaker' of your heart." b. "Myocardial contractility determines your heart rate." c. "The atrioventricular node is the 'pacemaker' of your heart." d. "The ventricular Purkinje network determines your heart rate."

A

A patient reports chest pain. The nurse is attempting to assess the pain to differentiate the pain as cardiac, respiratory, or gastrointestinal. The nurse can properly identify the pain as cardiac in origin when the patient states that the cardiac pain: a. does not occur with respiratory variations. b. is peripheral and may radiate to the scapular areas. c. is aggravated by inspiratory movements. d. is nonradiating and occurs during inspiration.

A

A patient was admitted to the surgical unit after surgical removal of an abdominal tumor. As the nurse performs a postsurgical assessment, the patient's blood pressure is 90/54, heart rate is 94, and respiratory rate is 22. Based on these values, the nurse should be most concerned with which of the following conditions? a. Hypovolemia b. Left-sided heart failure c. Right-sided heart failure d. Hypervolemia

A

A patient with a tracheostomy is experiencing thick and tenacious secretions. To maintain this patient's airway, what is the most appropriate action for the nurse? a. Tracheal suctioning b. Oropharyngeal suctioning c. Nasotracheal suctioning d. Orotracheal suctioning

A

The guardians of a premature infant who was delivered at 31-weeks' gestation is expected to be discharged from the hospital within the next few days. The guardians have voiced concern regarding how to prevent respiratory syncytial virus (RSV) exposure to the baby. What is the best response from the nurse? a. "You will need to limit the baby's exposure to crowds of people." b. "You need to make sure that the car seat is facing backward in the back seat of your car." c. "You do not need to be concerned; the baby has a natural protection against this disease." d. "You must sterilize all the bottles for the first 6 months."

A

The patient is experiencing shortness of breath. Which of the following lab tests indirectly indicates the oxygen level in the blood system? a. Hemoglobin b. White blood cell count c. Electrolytes d. Creatinine

A

Two hours after surgery, the nurse assesses a patient who had a chest tube inserted during surgery. There is 200 ml of dark red drainage in the chest tube at this time. What is the appropriate action for the nurse to perform? A) Record the amount and continue to monitor drainage. B) Notify the physician. C) Strip the chest tube starting at the chest D) Increase the suction by 10 mm Hg

A (Dark red drainage after surgery (50-200 ml per hour in first 3 hours) is expected but be aware of sudden increases greater than 100 ml per hour after the first three hours especially if becomes bright red in color.)

The nurse goes to assess a new patient and finds him short of breath with a rate of 32 and lying supine in bed. What is the priority nursing action? Raise the head of the bed to 60 degrees or higher. Get his oxygen saturation with a pulse oximeter. Take his blood pressure and respiratory rate. Notify the health care provider of his shortness of breath.

A (Raising the head of the bed will bring the diaphragm down and allow for better chest expansion thus improving oxygenation.)

The nurse is providing care for a patient who has decreased cardiac output related to heart failure. The nurse recognizes that cardiac output is A) Calculated by multiplying the patient's stroke volume by the heart rate. B) The average amount of blood ejected during one complete cardiac cycle. C) Determined by measuring the electrical activity of the heart and the patient's heart rate. D) The patient's average resting heart rate multiplied by the patient's mean arterial blood pressure.

A) Calculated by multiplying the patient's stroke volume by the heart rate. Cardiac output is determined by multiplying the patient's stroke volume by heart rate, thus identifying how much blood is pumped by the heart over a 1-minute period. Electrical activity of the heart and blood pressure are not direct components of cardiac output.

A patient was admitted following a motor vehicle accident with multiple fractured ribs. Respiratory assessment includes signs/symptoms of secondary pneumothorax. Which are the most common assessment findings associated with a pneumothorax? (Select all that apply). A) Sharp pleuritic pain that worsens on inspiration B) crackles over lung bases of affected lung C) Tracheal deviation toward the affected lung D) worsening dyspnea E) absent lung sounds to auscultation on affected side

A C D E (When the lung collapses, as with a pneumothorax, the thoracic space fills with air, which irritates the parietal pleura causing inspiratory pain. Because of the collapsed lung there is reduced gas exchange in the affected area, reduced oxygenation and dyspnea result. When an area of the lung collapses, breath sounds over affected area are absent.)

While performing a cardiovascular assessment, you might encounter a variety of pulsations and sounds. Which of the following is a normal sounds?

A brief thump felt near the fourth or fifth intercostal space near the left midclavicular line. This is where you would inspect and palpate for the point of maximal impulse. Also called an apical pulsation, it occurs as the apex of the heart bumps against the chest wall with each heartbeat. The apical impulse is not always visible but can be felt as a brief thump. This is a normal and expected finding when you are preparing to auscultate an apical pulse.

Cyanosis, the blue discoloration of the skin and mucous membranes caused by the presence of desaturated hemoglobin in capillaries, is:

A late sign of hypoxia Cyanosis, blue discoloration of the skin and mucous membranes caused by the presence of desaturated hemoglobin in capillaries, is a late sign of hypoxia. The presence or absence of cyanosis is not a reliable measure of oxygen status.

Which of the following cardiovascular effects of aging should the nurse anticipate when providing care for older adults (select all that apply)? A) Arterial stiffening B) Increased blood pressure C) Increased maximal heart rate D) Decreased maximal heart rate E) Increased recovery time from activity

A) Arterial stiffening B) Increased blood pressure D) Decreased maximal heart rate E) Increased recovery time from activity Well-documented cardiovascular effects of the aging process include arterial stiffening, possible increased blood pressure, and an increased amount of time that is required for recovery from activity. Maximal heart rate tends to decrease rather than increase with age.

A nurse is caring for a patient immediately following a transesophageal echocardiogram (TEE). Which of the following assessments are appropriate for this patient (select all that apply)? A) Assess for return of gag reflex. B) Assess groin for hematoma or bleeding. C) Monitor vital signs and oxygen saturation. D) Position patient supine with head of bed flat. E) Assess lower extremities for circulatory compromise.

A) Assess for return of gag reflex. C) Monitor vital signs and oxygen saturation. The patient undergoing a TEE has been given conscious sedation and has had the throat numbed with a local anesthetic spray, thus eliminating the gag reflex until the effects wear off. Therefore it is imperative that the nurse assess for gag reflex return before allowing the patient to eat or drink. Vital signs and oxygen saturation are also important assessment parameters resulting from the use of sedation. A TEE does not involve invasive procedures of the circulatory blood vessels. Therefore it is not necessary to monitor the patient's groin or lower extremities in relation to this procedure.

An adult collapsed at the grocery store. Before being transported to the emergency department, an automated external defibrillator (AED) was used. The nurse remembers learning that the AED is effective in saving a life because of which factors? (Select all that apply.) a. Use of an AED strengthens the chain of survival. b. The AED can be used by nonmedical personnel. c. The AED sends heart rhythm to the closest emergency room (ER) for the physician to analyze and give orders. d. Every minute without defibrillation decreases the survival rate by 7% to 10%. e. The AED will automatically deliver a shock to the victim after announcing, "Everyone stand back."

A,B,D

On entering the room, a nurse finds the patient sitting upright in bed with the upper torso resting on the over-bed table. The nurse assesses that this patient is experiencing acute hypoxemia. Which of the following are symptoms of acute hypoxemia? (Select all that apply.) a. Cyanosis b. Arrhythmias c. Eupnea d. Restlessness e. Diaphoresis

A,B,D,E

The home care nurse is admitting a patient with COPD. The primary healthcare provider has ordered O2 /nasal cannula at 3 L/min. To promote safety, the nurse would instruct the patient on which of the following safety measures? (Select all that apply.) a. Place "No smoking" signs in all areas where oxygen will be used. b. Instruct family and visitors who smoke that they must smoke a minimum of 10 feet from the patient. c. Store tanks in a small closet, trunk of a car, or near the hot water heater. d. Oxygen tanks should stay a minimum of 6 feet from space heaters, fireplaces, and appliances with an electric motor. e. Know the exit routes and where the fire extinguisher are located in the home.

A,D,E

A nurse is preparing to administer a medication to a client. The medication was scheduled for administration at 0900. Which of the following are acceptable administration times for this medication? (Select all that apply.) A. 0905 B. 0825 C. 1000 D. 0840 E. 0935

A. CORRECT: A medication should be administered within 30 min of the scheduled time. 0905 is within 30 min of a scheduled administration time of 0900. B. INCORRECT: 0825 is not within 30 min of a scheduled administration time of 0900. C. INCORRECT: 1000 is not within 30 min of a scheduled administration time of 0900. D. CORRECT: 0840 is within 30 min of a scheduled administration time of 0900. E. INCORRECT: 0935 is not within 30 min of a scheduled administration time of 0900.

A nurse educator is teaching a module on safe medication administration to newly hired nurses. Which of the following statements by a newly hired nurse indicate understanding of the nurse's responsibility when implementing medication therapy? (Select all that apply.) A. "I will observe for medication side effects." B. "I will monitor for therapeutic effects." C. "I will prescribe the appropriate dose." D. "I will change the dose if adverse effects occur." E. "I will refuse to give a medication if I believe it is unsafe."

A. CORRECT: The nurse is responsible for observing for medication side effects. This is within a nurse's scope of practice. B. CORRECT: The nurse is responsible for monitoring for therapeutic effects. This is within a nurse's scope of practice. C. INCORRECT: The provider, not the nurse, is responsible for prescribing the appropriate dose. This is outside of the nurse's scope of practice. D. INCORRECT: The provider, not the nurse, is responsible for changing the dose if adverse effects occur. This is outside of the nurse's scope of practice. E. CORRECT: The nurse is responsible for recognizing if a medication could potentially harm a client. It is within the nurse's scope of practice to refuse to administer the medication and contact the provider.

A nurse prepares an injection of morphine (Duramorph) to administer to a client who reports pain. Prior to administering the medication, the nurse is called to another room to assist another client onto a bedpan. She asks a second nurse to give the injection. Which of the following actions should the second nurse take? A. Offer to assist the client needing the bedpan. B. Administer the injection prepared by the other nurse. C. Prepare another syringe and administer the injection. D. Tell the client needing the bedpan she will have to wait for her nurse.

A. CORRECT: The second nurse should offer to assist the client needing the bedpan. This will allow the nurse who prepared the injection to administer it. B. INCORRECT: A nurse should only administer medications that he prepared. C. INCORRECT: Preparing another syringe will delay the administration of the pain medication. D. INCORRECT: Telling the client to wait is not an acceptable option for a client needing a bedpan.

If a nurse experiences a problem reading a physician's medication order, the most appropriate action will be to A. Call the physician to verify order. B. Call the pharmacist to verify order. C. Consult with other nursing staff to verify. D. Withhold the medication until physician makes rounds.

A. Call the physician to verify order

A nurse is providing teaching to an older adult client to promote adherence with medication administration. Which of the following instructions should the nurse include? (Select all that apply.) A. Adjust dosages according to daily weight. B. Place pills in daily pill holders. C. Provide liquid forms if the client has difficulty swallowing pills. D. Ask a relative to assist periodically. E. Request child-guard caps on medication containers.

A. INCORRECT: Adjustment of dosages is prescribed by the provider. Instructing the client to base dosages according to daily weight increases the risk for error in medication administration. B. CORRECT: Organizing medications in daily pill holders promotes medication adherence. C. CORRECT: Providing a form of medication that is easier for the client to swallow promotes medication adherence. D. CORRECT: Including the client's support system promotes medication adherence. E. INCORRECT: The older adult client may have difficulty opening child-guard caps. Request INCORRECT: Adjustment of dosages is prescribed by the provider. Instructing the client to base dosages according to daily weight increases the risk for error in medication administration.

A nurse is preparing medications for a preschool-age client. Which of the following factors should the nurse recognize as altering how children are affected by a medication? (Select all that apply.) A. Increased gastric acid production B. Lower blood pressure C. Higher body water content D. Increased absorption of topical medications E. Increased gastric emptying time

A. INCORRECT: Children have decreased gastric acid production B. CORRECT: Children have a lower blood pressure. C. CORRECT: Children have a higher body water content. D. CORRECT: Children have an increased absorption of topical medications. E. INCORRECT: Children have a slower gastric emptying time.

A nurse is working with a newly hired nurse who is administering medications to clients. Which of the following actions by the newly hired nurse indicates an understanding of medication error prevention? A. Taking all medications out of the unit-dose wrappers before entering the client's room B. Checking with the provider when a single dose requires administration of multiple tablets C. Administering a medication, then looking up the usual dosage range D. Relying on another nurse to clarify a medication prescription

A. INCORRECT: The nurse should not take unit-dose medications out of wrappers until at the bedside when performing the third check of medication administration. This prevents errors. The nurse can encourage client involvement and provide teaching at this time. B. CORRECT: If a single dose requires multiple tablets, it is possible that an error has occurred in the transcription of the medication. This action could prevent a medication error. C. INCORRECT: Reviewing usual dosage range prior to adminstration may uncover an inaccurate dosage. D. INCORRECT: If the prescription is unclear, the nurse should contact the provider for clarification, not another nurse.

A nurse is preparing to administer digoxin (Lanoxin) to a client who states, "I don't want to take that medication. I do not want one more pill." Which of the following responses by the nurse is appropriate in this situation? A. "Your physician prescribed it for you, so you really should take it." B. "Well, let's just get it over quickly then." C. "Okay, I'll just give you your other medications." D. "Tell me your concerns with taking this medication."

A. INCORRECT: This response is dismissive of the client's concerns. B. INCORRECT: The nurse is dismissing the client's concerns about taking the medication by continuing with medication administration. C. INCORRECT: Although clients have the right to refuse a medication, the nurse should provide information regarding the risk of refusal instead of proceeding with medication administration. D. CORRECT: Although clients have the right to refuse a medication, the nurse is correct in determining the reason for refusal by asking the client his concerns. After gathering the client's concerns, the nurse can provide information regarding the risk of refusal and provide information for an informed decision. At that point, if the client still exercises his right to refuse a medication, the nurse should notify appropriate personnel and document the refusal and actions taken.

The nurse is administering a sustained-release capsule to a new patient. The patient insists that he cannot swallow pills. What is the nurse's next best course of action? A. Ask the prescriber to change the order B. Crush the pill with a mortar and pestle C. Hide the capsule in a piece of solid food D. Open the capsule and sprinkle it over pudding

A. Rationale: Enteric-coated or sustained-release capsules should not be crushed; the nurse needs to contact the prescriber to change the medication to a form that is liquid or can be crushed.

The nurse takes a medication to a patient, and the patient tells him or her to take it away because she is not going to take it. What is the nurse's next action? A. Ask the patient's reason for refusal B. Explain that she must take the medication C. Take the medication away and chart the patient's refusal D. Tell the patient that her physician knows what is best for her

A. Rationale: When patients refuse a medication, first ask why they are refusing it.

A nurse educator is teaching a module on the six rights of safe medication administration to a group of newly licensed nurses. Use the ATI Active Learning Template: Basic Concept to complete this item to include the following: A. Related Content: List the six rights of safe medication administration. B. Underlying Principles: List at least three acceptable identifiers that can be used to verify the client's identification.

A. Related Content: Six Rights of Safe Medication Administration ● Right client - Verify the client's identification each time a medication is given. The Joint Commission requires that the nurse use two client identifiers when administering medications. ● Right medication - Correctly interpret medication prescription (verify completeness and clarity). Read labels three times: when the container is selected, when removing the dose from container, and when the container is replaced. ● Right dose - Calculate the correct medication dose; check a drug reference to ensure the dose is within the usual range. Ask another nurse to verify the dose if uncertain of the calculation. ● Right time - Administer medication on time to maintain a consistent therapeutic blood level. It is generally acceptable to administer the medication 1/2 hr before or after the scheduled time. Refer to the drug reference or facility policy for exceptions. ● Right route - Most common routes of administration are oral, topical, subcutaneous, intramuscular (IM), and intravenous (IV). Select the correct preparation for the ordered route. Know how to administer medication safely and correctly. ● Right documentation - Immediately record pertinent information, including the client's response to the medication. Document the medication after administration, not before. B. Underlying Principles ● Acceptable identifiers include the client's name, an assigned identification number, telephone number, birth date, or other person-specific identifier. ● The nurse can use bar code scanners to identify clients.

A patient has an order to receive 12.5 mg of hydrochlorothiazide. The nurse has on hand a 25 mg tablet of hydrochlorothiazide. How many tablet(s) will the nurse administer? a. 1/2 tablet b. 1 tablet c. 1 1/2 tablets d. 2 tablets

ANS: A 1/2 tablet will be given. The nurse is careful to perform nursing calculations to ensure proper medication administration. The dose ordered is 12.5. The dose on hand is 25. 12.5/25 = 1/2 tablet.

A nurse is preparing to administer medications to an older adult client who has vascular insufficiency and impaired kidney function. Use the ATI Active Learning Template: Basic Concept to complete this item to include the following sections: A. Underlying Principles: ● Discuss medication considerations related to vascular insufficiency. ● Discuss medication considerations related to impaired kidney function. ● Identify at least four additional physiologic changes associated with aging that impact pharmacokinetics. B. Nursing Interventions: Identify at least three interventions to decrease the risk of adverse effects.

A. Underlying Principles ● Vascular insufficiency prevents distribution of a medication to affected tissue. The nurse should document and monitor for the medication's effectiveness and report concerns to the provider. ● Impaired kidney function prevents or delays medication excretion, which increases the risk for toxicity. Decreased kidney function is the major cause of medication accumulation leading to toxicity. ● Physiologic changes ◯ Increased gastric pH ◯ Decreased gastrointestinal motility and gastric emptying time ◯ Decreased hepatic enzyme function ◯ Decreased protein-binding sites B. Nursing Interventions ◯ Decreased body water ◯ Increased body fat, decreased lean body mass ◯ Impaired memory or altered mental state ◯ Decreased mobility and dexterity ● Obtain a complete medication history and include all OTC medications. ● Make sure medication therapy starts at the lowest possible dose. ● Assess/monitor for therapeutic and adverse effects. ● Assess/monitor for medication-medication and medication-food interactions. ● Notify the provider of adverse effects.

A charge nurse is leading a peer group discussion about factors affecting medication dosages and responses. Which of the following statements by a fellow nurse indicates the need for further teaching? A. "Clients who have a higher BMI may require larger doses of medications." B. "Client tolerance to medication is a result of decreased metabolism." C. "Clients who have a placebo effect can have increased effectiveness of medication." D. "Client gender can affect responsiveness to medications."

A.INCORRECT: Clients who have a greater body mass may require larger doses of medications. B. CORRECT: This statement requires further teaching. Tolerance refers to a reduced responsiveness to medication. Accumulation refers to increased medication concentration in the body due to decreased metabolism. C. INCORRECT: The placebo effect describes positive medication effects influenced by psychological factors. D. INCORRECT: Client gender can affect responses to medication.

A nurse in an outpatient clinic is providing teaching to a client who is in her first trimester of pregnancy. Which of the following statements is appropriate for the nurse to include? A. "You will need to have a rubella vaccine if not received prior to pregnancy." B. "You can safely take over-the-counter medications." C. "You should avoid any vitamin preparations containing iron." D. "Your provider can prescribe medication for nausea if needed."

A.INCORRECT: Live-virus vaccines, including rubella, are contraindicated during pregnancy due to possible teratogenic effects. B. INCORRECT: Most medications, including over-the-counter, are considered potentially harmful to the fetus. The client should avoid any medications unless prescribed by her provider. C. INCORRECT: Nutritional supplements that include iron are commonly used during pregnancy to support the health of the mother and fetus. D. CORRECT: Providers can prescribe medications for the treatment of nausea and other discomforts of pregnancy.

A nurse is providing medication teaching for a client who is lactating. Which of the following is appropriate to include to minimize secretion in breast milk? A. Drink a full glass of milk with each dose of medication. B. Recommend medications that have an extended half-life. C. Take each prescribed dose right after breastfeeding. D. Pump breast milk and freeze prior to feeding to the infant.

A.INCORRECT: The intake of food or fluid with medication does not affect secretion of the medication into the breast milk. B. INCORRECT: The client should avoid medications that have an extended half-life due to increased secretion in breast milk. C. CORRECT: Taking medication immediately after breastfeeding helps minimize medication concentration in the next feeding. D. INCORRECT: Pumping and freezing breast milk does not affect the secretion of the medication into the breast milk.

1. A nurse is preparing to perform a lung assessment on a patient and discovers through the nursing history the patient smokes. The nurse figures the pack-years for this patient who has smoked two and a half (2 1/2) packs a day for 20 years. Which value will the nurse record in the patient's medical record? Record answer as a whole number. _________ pack-years

ANS: 50 Pack-years = Number of years smoking × Number of packs per day: 20 × 2.5 = 50.

26. During a routine physical examination of a 70-year-old patient, a blowing sound is auscultated over the carotid artery. Which assessment finding will the nurse report to the health care provider? a. Bruit b. Thrill c. Phlebitis d. Right-sided heart failure

ANS: A A bruit is the sound of turbulence of blood passing through a narrowed blood vessel and is auscultated as a blowing sound. A bruit can reflect cardiovascular disease in the carotid artery of middle-aged to older adults. Intensity or loudness is related to the rate of blood flow through the heart or the amount of blood regurgitated. A thrill is a continuous palpable sensation that resembles the purring of a cat. Jugular venous distention, not bruit, is a possible sign of right-sided heart failure. Some patients with heart disease have distended jugular veins when sitting. Phlebitis is an inflammation of a vein that occurs commonly after trauma to the vessel wall, infection, immobilization, and prolonged insertion of IV catheters. It affects predominantly peripheral veins.

The nurse is planning to administer a tuberculin test with a 27-gauge, -inch needle. At which angle will the nurse insert the needle? a. 15 degree b. 30 degree c. 45 degree d. 90 degree

ANS: A A 27-gauge, -inch needle is used for intradermal injections such as a tuberculin test, which should be inserted at a 5- to 15-degree angle, just under the dermis of the skin. Placing the needle at 30 degrees, 45 degrees, or 90 degrees will place the medication too deep.

30. A nurse is caring for a group of patients. Which patient will the nurse see first? a. An adult with an S4 heart sound b. A young adult with an S3 heart sound c. An adult with vesicular lung sounds in the lung periphery d. A young adult with bronchovesicular breath sounds between the scapula posteriorly

ANS: A A fourth heart sound (S4) occurs when the atria contract to enhance ventricular filling. An S4 is often heard in healthy older adults, children, and athletes, but it is not normal in adults. Because S4 also indicates an abnormal condition, report it to a health care provider. An S3 is considered abnormal in adults over 31 years of age but can often be heard normally in children and young adults. Vesicular lungs sounds in the periphery and bronchovesicular lung sounds in between the scapula are normal findings.

22. During a genitourinary examination of a 30-year-old male patient, the nurse identifies a small amount of a white, thick substance on the patient's uncircumcised glans penis. What is the nurse's next step? a. Record this as a normal finding. b. Avoid embarrassing questions about sexual activity. c. Notify the provider about a suspected sexually transmitted infection. d. Tell the patient to avoid doing self-examinations until symptoms clear.

ANS: A A small amount of thick, white smegma sometimes collects under the foreskin in the uncircumcised male and is considered normal. Penile pain or swelling, genital lesions, and urethral discharge are signs and symptoms that may indicate sexually transmitted infections (STI). All men 15 years and older need to perform a male-genital self-examination monthly. The nurse needs to assess a patient's sexual history and use of safe sex habits. Sexual history reveals risks for STI and HIV.

4. A nurse is conducting Weber's test. Which action will the nurse take? a. Place a vibrating tuning fork in the middle of patient's forehead. b. Place a vibrating tuning fork on the patient's mastoid process. c. Compare the number of seconds heard by bone versus air conduction. d. Compare the patient's degree of joint movement to the normal level.

ANS: A During Weber's test (lateralization of sound), the nurse places the vibrating tuning fork in the middle of the patient's forehead. During a Rinne test (comparison of air and bone conduction), the nurse places a vibrating tuning fork on the patient's mastoid process and compares the length of time air and bone conduction is heard. Comparing the patient's degree of joint movement to the normal level is a test for range of motion.

23. The nurse is preparing for a rectal examination of a nonambulatory male patient. In which position will the nurse place the patient? a. Sims' b. Knee-chest c. Dorsal recumbent d. Forward bending with flexed hips

ANS: A Nonambulatory patients are best examined in a side-lying Sims' position. Forward bending would require the patient to be able to stand upright. Knees to chest would be difficult to maintain in a nonambulatory male and is embarrassing and uncomfortable. Dorsal recumbent does not provide adequate access for a rectal examination and is used for abdominal assessment because it promotes relaxation of abdominal muscles.

8. A nurse identifies lice during a child's scalp assessment. The nurse teaches the parents about hair care. Which information from the parents indicates the nurse needs to follow up? a. We will use lindane-based shampoos. b. We will use the sink to wash hair. c. We will use a fine-toothed comb. d. We will use a vinegar hair rinse.

ANS: A Products containing lindane, a toxic ingredient, often cause adverse reactions; the nurse will need to follow up to correct the misconception. All the rest are correct. Instruct parents who have children with head lice to shampoo thoroughly with pediculicide (shampoo available at drugstores) in cold water at a basin or sink, comb thoroughly with a fine-toothed comb, and discard the comb. A dilute solution of vinegar and water helps loosen nits.

16. The patient is a 45-year-old African-American male who has come in for a routine annual physical. Which type of preventive screening does the nurse discuss with the patient? a. Digital rectal examination of the prostate b. Complete eye examination every year c. CA 125 blood test once a year d. Colonoscopy every 3 years

ANS: A Recommended preventive screenings include a digital rectal examination of the prostate and prostate-specific antigen test starting at age 50. CA 125 blood tests are indicated for women at high risk for ovarian cancer. Patients over the age of 65 need to have complete eye examinations yearly. Colonoscopy every 10 years is recommended in patients 50 years of age and older.

9. A parent calls the school nurse with questions regarding the recent school vision screening. Snellen chart examination revealed 20/60 for both eyes. Which response by the nurse is the best regarding the eye examination results? a. Your child needs to see an ophthalmologist. b. Your child is suffering from strabismus. c. Your child may have presbyopia. d. Your child has cataracts.

ANS: A The child needs an eye examination with an ophthalmologist or optometrist. Normal vision is 20/20. The larger the denominator, the poorer the patient's visual acuity. For example, a value of 20/60 means that the patient, when standing 20 feet away, can read a line that a person with normal vision can read from 60 feet away. Strabismus is a (congenital) condition in which both eyes do not focus on an object simultaneously: The eyes appear crossed. Acuity may not be affected; Snellen test does not test for strabismus. Presbyopia is impaired near vision that occurs in middle-aged and older adults and is caused by loss of elasticity of the lens. Cataracts, a clouding of the lens, develop slowly and progressively after age 35 or suddenly after trauma.

12. A male student comes to the college health clinic. He hesitantly describes that he found something wrong with his testis when taking a shower. Which assessment finding will alert the nurse to possible testicular cancer? a. Hard, pea-sized testicular lump b. Rubbery texture of testes c. Painful enlarged testis d. Prolonged diuretic use

ANS: A The most common symptoms of testicular cancer are a painless enlargement of one testis and the appearance of a palpable, small, hard lump, about the size of a pea, on the front or side of the testicle. Normally, the testes feel smooth, rubbery, and free of nodules. Use of diuretics, sedatives, or antihypertensives can lead to erection or ejaculation problems.

6. A febrile preschool-aged child presents to the after-hours clinic. Varicella (chickenpox) is diagnosed on the basis of the illness history and the presence of small, circumscribed skin lesions filled with serous fluid. Which type of skin lesion will the nurse report? a. Vesicles b. Wheals c. Papules d. Pustules

ANS: A Vesicles are circumscribed, elevated skin lesions filled with serous fluid that measure less than 1 cm. Wheals are irregularly shaped, elevated areas of superficial localized edema that vary in size. They are common with mosquito bites and hives. Papules are palpable, circumscribed, solid elevations in the skin that are smaller than 1 cm. Pustules are elevations of skin similar to vesicles, but they are filled with pus and vary in size like acne.

To promote comfort after a colonoscopy, in what position does the nurse place the client? a. Left lateral b. Prone c. Right lateral d. Supine

ANS: A After colonoscopy, clients have less discomfort and quicker passage of flatus when placed in the left lateral position.

An older client has had an instance of drug toxicity and asks why this happens, since the client has been on this medication for years at the same dose. What response by the nurse is best? a. "Changes in your liver cause drugs to be metabolized differently." b. "Perhaps you don't need as high a dose of the drug as before." c. "Stomach muscles atrophy with age and you digest more slowly." d. "Your body probably can't tolerate as much medication anymore."

ANS: A Decreased liver enzyme activity depresses drug metabolism, which leads to accumulation of drugs—possibly to toxic levels. The other options do not accurately explain this age-related change.

An emergency room nurse obtains the health history of a client. Which statement by the client should alert the nurse to the occurrence of heart failure? a. "I get short of breath when I climb stairs." b. "I see halos floating around my head." c. "I have trouble remembering things." d. "I have lost weight over the past month."

ANS: A Dyspnea on exertion is an early manifestation of heart failure and is associated with an activity such as stair climbing. The other findings are not specific to early occurrence of heart failure.

The nurse is preparing for a rectal examination of a nonambulatory male patient. In which position will the nurse place the patient? a. Sims' b. Knee-chest c. Dorsal recumbent d. Forward bending with flexed hips

ANS: A Nonambulatory patients are best examined in a side-lying Sims' position. Forward bending would require the patient to be able to stand upright. Knees to chest would be difficult to maintain in a nonambulatory male and is embarrassing and uncomfortable. Dorsal recumbent does not provide adequate access for a rectal examination and is used for abdominal assessment because it promotes relaxation of abdominal muscles.

The nurse knows that a client with prolonged prothrombin time (PT) values (not related to medication) probably has dysfunction in which organ? a. Kidneys b. Liver c. Spleen d. Stomach

ANS: B Severe acute or chronic liver damage leads to a prolonged PT secondary to impaired synthesis of clotting proteins. The other organs are not related to this issue.

The health care provider has ordered a hypotonic intravenous (IV) solution to be administered. Which IV bag will the nurse prepare? a. 0.45% sodium chloride (1/2 NS) b. 0.9% sodium chloride (NS) c. Lactated Ringer's (LR) d. Dextrose 5% in Lactated Ringer's (D5LR)

ANS: A 0.45% sodium chloride is a hypotonic solution. NS and LR are isotonic. D5LR is hypertonic.

A nurse is caring for a patient who has poor tissue perfusion as the result of hypertension. When the patient asks what to eat for breakfast, which meal should the nurse suggest? a. A cup of nonfat yogurt with granola and a handful of dried apricots b. Whole wheat toast with butter and a side of bacon c. A bowl of cereal with whole milk and a banana d. Omelet with sausage, cheese, and onions

ANS: A A 2000-calorie diet of fruits, vegetables, and low-fat dairy foods that are high in fiber, potassium, calcium, and magnesium and low in saturated and total fat helps prevent and reduce the effects of hypertension. Nonfat yogurt with granola is a good source of calcium, fiber, and potassium; dried apricots add a second source of potassium. Although cereal and a banana provide fiber and potassium, skim milk should be substituted for whole milk to decrease fat. An omelet with sausage and cheese is high in fat. Butter and bacon are high in fat.

During a routine physical examination of a 70-year-old patient, a blowing sound is auscultated over the carotid artery. Which assessment finding will the nurse report to the health care provider? a. Bruit b. Thrill c. Phlebitis d. Right-sided heart failure

ANS: A A bruit is the sound of turbulence of blood passing through a narrowed blood vessel and is auscultated as a blowing sound. A bruit can reflect cardiovascular disease in the carotid artery of middle-aged to older adults. Intensity or loudness is related to the rate of blood flow through the heart or the amount of blood regurgitated. A thrill is a continuous palpable sensation that resembles the purring of a cat. Jugular venous distention, not bruit, is a possible sign of right-sided heart failure. Some patients with heart disease have distended jugular veins when sitting. Phlebitis is an inflammation of a vein that occurs commonly after trauma to the vessel wall, infection, immobilization, and prolonged insertion of IV catheters. It affects predominantly peripheral veins

A nurse is caring for a group of patients. Which patient will the nurse see first? a. An adult with an S4 heart sound b. A young adult with an S3 heart sound c. An adult with vesicular lung sounds in the lung periphery d. A young adult with bronchovesicular breath sounds between the scapula posteriorly

ANS: A A fourth heart sound (S4) occurs when the atria contract to enhance ventricular filling. An S4 is often heard in healthy older adults, children, and athletes, but it is not normal in adults. Because S4 also indicates an abnormal condition, report it to a health care provider. An S3 is considered abnormal in adults over 31 years of age but can often be heard normally in children and young adults. Vesicular lungs sounds in the periphery and bronchovesicular lung sounds in between the scapula are normal findings

A patient has carbon dioxide retention from lung problems. Which type of diet will the nurse most likely suggest for this patient? a. Low-carbohydrate b. Low-caffeine c. High-caffeine d. High-carbohydrate

ANS: A A low-carbohydrate diet is best. Diets high in carbohydrates play a role in increasing the carbon dioxide load for patients with carbon dioxide retention. As carbohydrates are metabolized, an increased load of carbon dioxide is created and excreted via the lungs. A low- or high-caffeine diet is not as important as the carbohydrate load

A nurse is caring for a patient with peripheral intravenous (IV) therapy. Which task will the nurse assign to the nursing assistive personnel? a. Recording intake and output b. Regulating intravenous flow rate c. Starting peripheral intravenous therapy d. Changing a peripheral intravenous dressing

ANS: A A nursing assistive personnel (NAP) can record intake and output. An RN cannot delegate regulating flow rate, starting an IV, or changing an IV dressing to an NAP.

During a genitourinary examination of a 30-year-old male patient, the nurse identifies a small amount of a white, thick substance on the patient's uncircumcised glans penis. What is the nurse's next step? a. Record this as a normal finding. b. Avoid embarrassing questions about sexual activity. c. Notify the provider about a suspected sexually transmitted infection. d. Tell the patient to avoid doing self-examinations until symptoms clear.

ANS: A A small amount of thick, white smegma sometimes collects under the foreskin in the uncircumcised male and is considered normal. Penile pain or swelling, genital lesions, and urethral discharge are signs and symptoms that may indicate sexually transmitted infections (STI). All men 15 years and older need to perform a male-genital self-examination monthly. The nurse needs to assess a patient's sexual history and use of safe sex habits. Sexual history reveals risks for STI and HIV

While preparing medications, the nurse knows one of the drug is an acidic medication. In which area does the nurse anticipate the drug will be absorbed? a. Stomach b. Mouth c. Small intestine d. Large intestine

ANS: A Acidic medications pass through the gastric mucosa rapidly. Medications that are basic are not absorbed before reaching the small intestine

The nurse administers a central nervous system stimulant to a patient. Which assessment finding indicates to the nurse that an idiosyncratic event is occurring? a. Falls asleep during daily activities b. Presents with a pruritic rash c. Develops restlessness d. Experiences alertness

ANS: A An idiosyncratic event is a reaction opposite to what the effects of the medication normally are, or the patient overreacts or underreacts to the medication. Falls asleep is an opposite effect of what a central nervous system stimulant should do. A stimulant should make a patient restless and alert. A pruritic (itch) rash could indicate an allergic reaction.

The patient is experiencing angina pectoris. Which assessment finding does the nurse expect when conducting a history and physical examination? a. Experiences chest pain after eating a heavy meal b. Experiences adequate oxygen saturation during exercise c. Experiences crushing chest pain for more than 20 minutes d. Experiences tingling in the left arm that lasts throughout the morning

ANS: A Angina pectoris is chest pain that results from limited oxygen supply. Often pain is precipitated by activities such as exercise, stress, and eating a heavy meal and lasts 3 to 5 minutes. Symptoms of angina pectoris are relieved by rest and/or nitroglycerin. Adequate oxygen saturation occurs with rest; inadequate oxygen saturation occurs during exercise. Pain lasting longer than 20 minutes or arm tingling that persists could be a sign of myocardial infarction

The nurse prepares a pain injection for a patient but had to check on another patient and asks a new nurse to give the medication. Which action by the new nurse is best? a. Do not give the medication. b. Administer the medication just this once. c. Give the medication for any pain score greater than 8. d. Avoid the issue and pretend to not hear the request.

ANS: A Because the nurse who administers the medication is responsible for any errors related to it, nurses administer only the medications they prepare. You cannot delegate preparation of medication to another person and then administer the medication to the patient. The right medication cannot be verified by the new nurse; do not violate the six rights. Do not administer the medication even one time. Do not administer the medication regardless of the pain rating. Avoiding the issue is not appropriate or safe.

A nurse teaches the patient about the prescribed buccal medication. Which statement by the patient indicates teaching by the nurse is successful? a. "I should let the medication dissolve completely." b. "I will place the medication in the same location." c. "I can only drink water, not juice, with this medication." d. "I better chew my medication first for faster distribution."

ANS: A Buccal medications should be placed in the side of the cheek and allowed to dissolve completely. Buccal medications act with the patient's saliva and mucosa. The patient should not chew or swallow the medication or take any liquids with it. The patient should rotate sides of the cheek to avoid irritating the mucosal lining.

The nurse is careful to monitor a patient's cardiac output. Which goal is the nurse trying to achieve? a. To determine peripheral extremity circulation b. To determine oxygenation requirements c. To determine cardiac dysrhythmias d. To determine ventilation status

ANS: A Cardiac output indicates how much blood is being circulated systemically throughout the body to the periphery. The amount of blood ejected from the left ventricle each minute is the cardiac output. Oxygen status would be determined by pulse oximetry and the presence of cyanosis. Cardiac dysrhythmias are an electrical impulse monitored through ECG results. Ventilation status is measured by respiratory rate, pulse oximetry, and capnography. Capnography provides instant information about the patient's ventilation. Ventilation status does not depend solely on cardiac output.

The patient has an intravenous (IV) line and the nurse needs to remove the gown. In which order will the nurse perform the steps, starting with the first one? 1. Remove the sleeve of the gown from the arm without the IV. 2. Remove the sleeve of the gown from the arm with the IV. 3. Remove the IV solution container from its stand. 4. Pass the IV bag and tubing through the sleeve. a. 1, 2, 3, 4 b. 2, 3, 4, 1 c. 3, 4, 1, 2 d. 4, 1, 2, 3

ANS: A Change regular gowns by following these steps for maximum speed and arm mobility: (1) To remove a gown, remove the sleeve of the gown from the arm without the IV line, maintaining the patient's privacy. (2) Remove the sleeve of the gown from the arm with the IV line. (3) Remove the IV solution container from its stand, and pass it and the tubing through the sleeve. (If this involves removing the tubing from an EID, use the roller clamp to slow the infusion to prevent the accidental infusion of a large volume of solution or medication.)

A patient with chronic obstructive pulmonary disease (COPD) asks the nurse why clubbing occurs. Which response by the nurse is most therapeutic? a. "Your disease doesn't send enough oxygen to your fingers." b. "Your disease affects both your lungs and your heart, and not enough blood is being pumped." c. "Your disease will be helped if you pursed-lip breathe." d. "Your disease often makes patients lose mental status."

ANS: A Clubbing of the nail bed can occur with COPD and other diseases that cause prolonged oxygen deficiency or chronic hypoxemia. Pursed-lipped breathing helps the alveoli stay open but is not the cause of clubbing. Loss of mental status is not a normal finding with COPD and will not result in clubbing. Low oxygen and not low circulating blood volume is the problem in COPD that results in clubbing

A nurse is conducting Weber's test. Which action will the nurse take? a. Place a vibrating tuning fork in the middle of patient's forehead. b. Place a vibrating tuning fork on the patient's mastoid process. c. Compare the number of seconds heard by bone versus air conduction. d. Compare the patient's degree of joint movement to the normal level.

ANS: A During Weber's test (lateralization of sound), the nurse places the vibrating tuning fork in the middle of the patient's forehead. During a Rinne test (comparison of air and bone conduction), the nurse places a vibrating tuning fork on the patient's mastoid process and compares the length of time air and bone conduction is heard. Comparing the patient's degree of joint movement to the normal level is a test for range of motion

A patient needs assistance in eliminating an anesthetic gaseous medication (nitrous oxide). Which action will the nurse take? a. Encourage the patient to cough and deep-breathe. b. Suction the patient's respiratory secretions. c. Suggest voiding every 2 hours. d. Increase fluid intake

ANS: A Gaseous and volatile medications are excreted through gas exchange (lungs). Deep breathing and coughing will assist in clearing the medication more quickly. It is a gaseous medication and cannot be suctioned out of the lungs. It is not excreted through the kidneys so fluids and voiding will not help.

A nurse is preparing to administer an antibiotic medication at 1000 to a patient but gets busy in another room. When should the nurse give the antibiotic medication? a. By 1030 b. By 1100 c. By 1130 d. By 1200

ANS: A Give time-critical scheduled medications (e.g., antibiotics, anticoagulants, insulin, anticonvulsants, and immunosuppressive agents) at the exact time ordered (within 30 minutes before or after scheduled dose). Give non-time critical scheduled medications within a range of either 1 or 2 hours of scheduled dose. 1100, 1130, and 1200 are too late.

The nurse administers an intravenous (IV) hypertonic solution to a patient. In which direction will the fluid shift? a. From intracellular to extracellular b. From extracellular to intracellular c. From intravascular to intracellular d. From intravascular to interstitial

ANS: A Hypertonic solutions will move fluid from the intracellular to the extracellular (intravascular). A hypertonic solution has a concentration greater than normal body fluids, so water will shift out of cells because of the osmotic pull of the extra particles. Movement of water from the extracellular (intravascular) into cells (intracellular) occurs when hypotonic fluids are administered. Distribution of fluid between intravascular and interstitial spaces occurs by filtration, the net sum of hydrostatic and osmotic pressures.

The nurse is administering medications to several patients. Which action should the nurse take? a. Advise a patient after a corticosteroid inhaler treatment to rinse mouth with water. b. Administer an intravenous medication through tubing that is infusing blood. c. Pinch up the deltoid muscle of an adult patient receiving a vaccination. d. Aspirate before administering a subcutaneous injection in the abdomen.

ANS: A If the patient uses a corticosteroid, have him or her rinse the mouth out with water or salt water or brush teeth after inhalation to reduce risk of fungal infection. Piercing a blood vessel during a subcutaneous injection is very rare. Therefore, aspiration is not necessary when administering subcutaneous injections. When giving immunizations to adults: to avoid injection into subcutaneous tissue, spread the skin of the selected vaccine administration site taut between the thumb and forefinger, isolating the muscle. Never administer IV medications through tubing that is infusing blood, blood products, or parenteral nutrition solutions.

An order is written for phenytoin 500 mg IM q3-4h prn for pain. The nurse recognizes that treatment of pain is not a standard therapeutic indication for this drug. The nurse believes that the health care provider meant to write hydromorphone. What should the nurse do? a. Call the health care provider to clarify the order. b. Give the patient hydromorphone, as it was meant to be written. c. Administer the medication and monitor the patient frequently. d. Refuse to give the medication and notify the nurse supervisor.

ANS: A If there is any question about a medication order because it is incomplete, illegible, vague, or not understood, contact the health care provider before administering the medication. The nurse cannot change the order without the prescriber's consent; this is out of the nurse's scope of practice. Ultimately, the nurse can be held responsible for administering an incorrect medication. If the prescriber is unwilling to change the order and does not justify the order in a reasonable and evidence-based manner, the nurse may refuse to give the medication and notify the supervisor

The patient is breathing normally. Which process does the nurse consider is working properly when the patient inspires? a. Stimulation of chemical receptors in the aorta b. Reduction of arterial oxygen saturation levels c. Requirement of elastic recoil lung properties d. Enhancement of accessory muscle usage

ANS: A Inspiration is an active process, stimulated by chemical receptors in the aorta. Reduced arterial oxygen saturation levels indicate hypoxemia, an abnormal finding. Expiration is a passive process that depends on the elastic recoil properties of the lungs, requiring little or no muscle work. Prolonged use of the accessory muscles does not promote effective ventilation and causes fatigue.

The nurse is calculating intake and output on a patient. The patient drinks 150 mL of orange juice at breakfast, voids 125 mL after breakfast, vomits 250 mL of greenish fluid, sucks on 60 mL of ice chips, and for lunch consumes 75 mL of chicken broth. Which totals for intake and output will the nurse document in the patient's medical record? a. Intake 255; output 375 b. Intake 285; output 375 c. Intake 505; output 125 d. Intake 535; output 125

ANS: A Intake = 150 mL of orange juice, 60 mL of ice chips (but only counts as 30 since ice chips are half of the amount), and 75 mL of chicken broth; 150 + 30 + 75 = 255. Output = 125 mL of urine (void) and 250 mL of vomitus; 125 + 250 = 375.

A nurse is caring for a patient with chronic obstructive pulmonary disease (COPD) who is receiving 2 L/min of oxygen. Which oxygen delivery device is most appropriate for the nurse to administer the oxygen? a. Nasal cannula b. Simple face mask c. Non-rebreather mask d. Partial non-rebreather mask

ANS: A Nasal cannulas deliver oxygen from 1 to 6 L/min. All other devices (simple face mask, nonrebreather mask, and partial non-rebreather mask) are intended for flow rates greater than 6 L/min.

What is the nurse's priority action to protect a patient from medication error? a. Reading medication labels at least 3 times before administering b. Administering as many of the medications as possible at one time c. Asking anxious family members to leave the room before giving a medication d. Checking the patient's room number against the medication administration record

ANS: A One step to take to prevent medication errors is to read labels at least 3 times before administering the medication. The nurse should address the family's concerns about medications before administering them. Do not discount their anxieties. The medication administration record should be checked against the patient's hospital identification band; a room number is not an acceptable identifier. Medications should be given when scheduled, and medications with special assessment indications should be separated. Giving medications at one time can cause the patient to aspirate

The nurse is caring for a patient with fluid volume overload. Which physiological effect does the nurse most likely expect? a. Increased preload b. Increased heart rate c. Decreased afterload d. Decreased tissue perfusion

ANS: A Preload refers to the amount of blood in the left ventricle at the end of diastole; an increase in circulating volume would increase the preload of the heart. Afterload refers to resistance; increased pressure would lead to increased resistance, and afterload would increase. A decrease in tissue perfusion would be seen with hypovolemia. A decrease in fluid volume would cause an increase in heart rate as the body is attempting to increase cardiac output.

A nurse identifies lice during a child's scalp assessment. The nurse teaches the parents about hair care. Which information from the parents indicates the nurse needs to follow up? a. We will use lindane-based shampoos. b. We will use the sink to wash hair. c. We will use a fine-toothed comb. d. We will use a vinegar hair rinse

ANS: A Products containing lindane, a toxic ingredient, often cause adverse reactions; the nurse will need to follow up to correct the misconception. All the rest are correct. Instruct parents who have children with head lice to shampoo thoroughly with pediculicide (shampoo available at drugstores) in cold water at a basin or sink, comb thoroughly with a fine-toothed comb, and discard the comb. A dilute solution of vinegar and water helps loosen nits

The patient is a 45-year-old African-American male who has come in for a routine annual physical. Which type of preventive screening does the nurse discuss with the patient? a. Digital rectal examination of the prostate b. Complete eye examination every year c. CA 125 blood test once a year d. Colonoscopy every 3 years

ANS: A Recommended preventive screenings include a digital rectal examination of the prostate and prostate-specific antigen test starting at age 50. CA 125 blood tests are indicated for women at high risk for ovarian cancer. Patients over the age of 65 need to have complete eye examinations yearly. Colonoscopy every 10 years is recommended in patients 50 years of age and older

The patient has right-sided heart failure. Which finding will the nurse expect when performing an assessment? a. Peripheral edema b. Basilar crackles c. Chest pain d. Cyanosis

ANS: A Right-sided heart failure results from inability of the right side of the heart to pump effectively, leading to a systemic backup. Peripheral edema, distended neck veins, and weight gain are signs of right-sided failure. Basilar crackles can indicate pulmonary congestion from left-sided heart failure. Cyanosis and chest pain result from inadequate tissue perfusion.

A patient who is being discharged today is going home with an inhaler. The patient is to administer 2 puffs of the inhaler twice daily. The inhaler contains 200 puffs. When should the nurse appropriately advise the patient to refill the medication? a. 6 weeks from the start of using the inhaler b. As soon as the patient leaves the hospital c. When the inhaler is half empty. d. 50 days after discharge

ANS: A Six weeks will be about the time the inhaler will need to be refilled. The inhaler should last the patient 50 days (2 puffs× 2/twice daily = 4; 200/4 = 50); the nurse should advise the patient to refill the prescription when there are 7 to 10 days of medication remaining. Refilling it as soon as the patient leaves the hospital or when the inhaler is half empty is too early. If the patient waits 50 days, the patient will run out of medication before it can be refilled.

The nurse is reviewing laboratory results. Which cation will the nurse observe is the most abundant in the blood? a. Sodium b. Chloride c. Potassium d. Magnesium

ANS: A Sodium is the most abundant cation in the blood. Potassium is the predominant intracellular cation. Chloride is an anion (negatively charged) rather than a cation (positively charged). Magnesium is found predominantly inside cells and in bone.

A nurse is reviewing the electrocardiogram (ECG) results. Which portion of the conduction system does the nurse consider when evaluating the P wave? a. SA node b. AV node c. Bundle of His d. Purkinje fibers

ANS: A The P wave represents the electrical conduction through both atria; the SA node initiates electrical conduction through the atria. The AV node conducts down through the bundle of His and the Purkinje fibers to cause ventricular contraction.

A nurse explains the function of the alveoli to a patient with respiratory problems. Which information about the alveoli's function will the nurse share with the patient? a. Carries out gas exchange b. Regulates tidal volume c. Produces hemoglobin d. Stores oxygen

ANS: A The alveolus is a capillary membrane that allows gas exchange of oxygen and carbon dioxide during respiration. The alveoli do not store oxygen, regulate tidal volume, or produce hemoglobin

A parent calls the school nurse with questions regarding the recent school vision screening. Snellen chart examination revealed 20/60 for both eyes. Which response by the nurse is the best regarding the eye examination results? a. Your child needs to see an ophthalmologist. b. Your child is suffering from strabismus. c. Your child may have presbyopia. d. Your child has cataracts

ANS: A The child needs an eye examination with an ophthalmologist or optometrist. Normal vision is 20/20. The larger the denominator, the poorer the patient's visual acuity. For example, a value of 20/60 means that the patient, when standing 20 feet away, can read a line that a person with normal vision can read from 60 feet away. Strabismus is a (congenital) condition in which both eyes do not focus on an object simultaneously: The eyes appear crossed. Acuity may not be affected; Snellen test does not test for strabismus. Presbyopia is impaired near vision that occurs in middle-aged and older adults and is caused by loss of elasticity of the lens. Cataracts, a clouding of the lens, develop slowly and progressively after age 35 or suddenly after trauma.

Which coughing technique will the nurse use to help a patient clear central airways? a. Huff b. Quad c. Cascade d. Incentive spirometry

ANS: A The huff cough stimulates a natural cough reflex and is generally effective only for clearing central airways. While exhaling, the patient opens the glottis by saying the word huff. The quad cough technique is for patients without abdominal muscle control such as those with spinal cord injuries. While the patient breathes out with a maximal expiratory effort, the patient or nurse pushes inward and upward on the abdominal muscles toward the diaphragm, causing the cough. With the cascade cough the patient takes a slow, deep breath and holds it for 2 seconds while contracting expiratory muscles. Then he or she opens the mouth and performs a series of coughs throughout exhalation, thereby coughing at progressively lowered lung volumes. This technique promotes airway clearance and a patent airway in patients with large volumes of sputum. Incentive spirometry encourages voluntary deep breathing by providing visual feedback to patients about inspiratory volume. It promotes deep breathing and prevents or treats atelectasis in the postoperative patient.

Four patients arrive at the emergency department at the same time. Which patient will the nurse see first? a. An infant with temperature of 102.2° F and diarrhea for 3 days b. A teenager with a sprained ankle and excessive edema c. A middle-aged adult with abdominal pain who is moaning and holding her stomach d. An older adult with nausea and vomiting for 3 days with blood pressure 112/60

ANS: A The infant should be seen first. An infant's proportion of total body water (70% to 80% total body weight) is greater than that of children or adults. Infants and young children have greater water needs and immature kidneys. They are at greater risk for extracellular volume deficit and hypernatremia because body water loss is proportionately greater per kilogram of weight. A teenager with excessive edema from a sprained ankle can wait. A middle-aged adult moaning in pain can wait as can an older adult with a blood pressure of 112/60.

A patient is experiencing respiratory acidosis. Which organ system is responsible for compensation in this patient? a. Renal b. Endocrine c. Respiratory d. Gastrointestinal

ANS: A The kidneys (renal) are responsible for respiratory acidosis compensation. A problem with the respiratory system causes respiratory acidosis, so another organ system (renal) needs to compensate. Problems with the gastrointestinal and endocrine systems can cause acid-base imbalances, but these systems cannot compensate for an existing imbalance

The nurse is caring for a diabetic patient in renal failure who is in metabolic acidosis. Which laboratory findings are consistent with metabolic acidosis? a. pH 7.3, PaCO2 36 mm Hg, HCO3- 19 mEq/L b. pH 7.5, PaCO2 35 mm Hg, HCO3- 35 mEq/L c. pH 7.32, PaCO2 47 mm Hg, HCO3- 23 mEq/L d. pH 7.35, PaCO2 40 mm Hg, HCO3- 25 mEq/L

ANS: A The laboratory values that reflect metabolic acidosis are pH 7.3, PaCO2 36 mm Hg, HCO3- 19 mEq/L. A laboratory finding of pH 7.5, PaCO2 35 mm Hg, HCO3- 35 mEq/L is metabolic alkalosis. pH 7.32, PaCO2 47 mm Hg, HCO3- 23 mEq/L is respiratory acidosis. pH 7.35, PaCO2 40 mm Hg, HCO3- 25 mEq/L values are within normal range.

A nurse is caring for a group of patients. Which patient should the nurse see first? a. A patient with hypercapnia wearing an oxygen mask b. A patient with a chest tube ambulating with the chest tube unclamped c. A patient with thick secretions being tracheal suctioned first and then orally d. A patient with a new tracheostomy and tracheostomy obturator at bedside

ANS: A The mask is contraindicated for patients with carbon dioxide retention (hypercapnia) because retention can be worsened; the nurse must see this patient first to correct the problem. All the rest are using correct procedures and do not need to be seen first. A chest tube should not be clamped when ambulating. Clamping a chest tube is contraindicated when ambulating or transporting a patient. Clamping can result in a tension pneumothorax. Use nasotracheal suctioning before pharyngeal suctioning whenever possible. The mouth and pharynx contain more bacteria than the trachea. Keep tracheostomy obturator at bedside with a fresh (new) tracheostomy to facilitate reinsertion of the outer cannula if dislodged

A male student comes to the college health clinic. He hesitantly describes that he found something wrong with his testis when taking a shower. Which assessment finding will alert the nurse to possible testicular cancer? a. Hard, pea-sized testicular lump b. Rubbery texture of testes c. Painful enlarged testis d. Prolonged diuretic use

ANS: A The most common symptoms of testicular cancer are a painless enlargement of one testis and the appearance of a palpable, small, hard lump, about the size of a pea, on the front or side of the testicle. Normally, the testes feel smooth, rubbery, and free of nodules. Use of diuretics, sedatives, or antihypertensives can lead to erection or ejaculation problems

A nurse has withdrawn a narcotic from the medication dispenser and must waste a portion of the medication. What should the nurse do? a. Have another nurse witness the wasted medication. b. Return the wasted medication to the medication dispenser. c. Place the wasted portion of the medication in the sharps container. d. Exit the medication room to call the health care provider to request an order that matches the dosages

ANS: A The nurse should follow Nurse Practice Acts and safe narcotic administration guidelines by having a nurse witness the "wasted" medication. The nurse cannot return the wasted medication to the medication dispenser. Wasted portions of medications are not placed in sharps containers. The nurse should not leave the narcotic unattended and call the health care provider to obtain matching dosages; the nurse is expected to obtain the correct dose.

The nurse is caring for a group of patients. Which patient will the nurse see first? a. A patient with D5W hanging with the blood b. A patient with type A blood receiving type O blood c. A patient with intravenous potassium chloride that is diluted d. A patient with a right mastectomy and an intravenous site in the left arm

ANS: A The nurse will see the patient with D5W and blood to prevent a medication error. When preparing to administer blood, prime the tubing with 0.9% sodium chloride (normal saline) to prevent hemolysis or breakdown of RBCs. All the rest are normal. A patient with type A blood can receive type O. Type O is considered the universal donor. A patient with a mastectomy should have the IV in the other arm. Potassium chloride should be diluted, and it is never given IV push.

A patient has a myocardial infarction. On which primary blood vessel will the nurse focus care to reduce ischemia? a. Superior vena cava b. Pulmonary artery c. Coronary artery d. Carotid artery

ANS: C A myocardial infarction is the lack of blood flow due to obstruction to the coronary artery, which supplies the heart with blood. The superior vena cava returns blood back to the heart. The pulmonary artery supplies deoxygenated blood to the lungs. The carotid artery supplies blood to the brain.

A nurse is caring for a patient who is receiving pain medication through a saline lock. After obtaining a good blood return when the nurse is flushing the patient's peripheral IV, the patient reports pain. Upon assessment, the nurse notices a red streak that is warm and tender to the touch. What is the nurse's initial action? a. Do not administer the pain medication. b. Administer the pain medication slowly. c. Apply a warm compress to the site. d. Apply a cool compress to the site

ANS: A The patient has phlebitis; the initial nursing action is do not administer the medication. The medication should not be given slowly. A cool or warm compress may be used later depending upon protocol, but it is not the first action.

The nurse closely monitors an older adult for signs of medication toxicity. Which physiological change is the reason for the nurse's action? a. Reduced glomerular filtration b. Reduced esophageal stricture c. Increased gastric motility d. Increased liver mass

ANS: A The reduced glomerular filtration rate delays excretion, increasing chance for toxicity. In older adults, gastric motility and liver mass decrease. Esophageal stricture is not a physiological change associated with normal aging

A nurse is assessing a patient who is receiving a blood transfusion and finds that the patient is anxiously fidgeting in bed. The patient is afebrile and dyspneic. The nurse auscultates crackles in both lung bases and sees jugular vein distention. On which transfusion complication will the nurse focus interventions? a. Fluid volume overload b. Hemolytic reaction c. Anaphylactic shock d. Septicemia

ANS: A The signs and symptoms are concurrent with fluid volume overload. Anaphylactic shock would have presented with urticaria, dyspnea, and hypotension. Septicemia would include a fever. A hemolytic reaction would consist of flank pain, chills, and fever.

The nurse is caring for a patient who needs oxygen via a nasal cannula. Which task can the nurse delegate to the nursing assistive personnel? a. Applying the nasal cannula b. Adjusting the oxygen flow c. Assessing lung sounds d. Setting up the oxygen

ANS: A The skill of applying (not adjusting oxygen flow) a nasal cannula or oxygen mask can be delegated to nursing assistive personnel (NAP). The nurse is responsible for assessing the patient's respiratory system, response to oxygen therapy, and setup of oxygen therapy, including adjustment of oxygen flow rate.

A nurse is caring for a diabetic patient with a bowel obstruction and has orders to ensure that the volume of intake matches the output. In the past 4 hours, the patient received dextrose 5% with 0.9% sodium chloride through a 22-gauge catheter infusing at 150 mL/hr and has eaten 200 mL of ice chips. The patient also has an NG suction tube set to low continuous suction that had 300-mL output. The patient has voided 400 mL of urine. After reporting these values to the health care provider, which order does the nurse anticipate? a. Add a potassium supplement to replace loss from output. b. Decrease the rate of intravenous fluids to 100 mL/hr. c. Administer a diuretic to prevent fluid volume excess. d. Discontinue the nasogastric suctioning

ANS: A The total fluid intake and output equals 700 mL, which meets the provider goals. Patients with nasogastric suctioning are at risk for potassium deficit, so the nurse would anticipate a potassium supplement to correct this condition. Remember to record half the volume of ice chips when calculating intake. The other measures would be unnecessary because the net fluid volume is equal

The nurse is creating a plan of care for an obese patient who is suffering from fatigue related to ineffective breathing. Which intervention best addresses a short-term goal the patient could achieve? a. Sleeping on two to three pillows at night b. Limiting the diet to 1500 calories a day c. Running 30 minutes every morning d. Stopping smoking immediately

ANS: A To achieve a short-term goal, the nurse should plan a lifestyle change that the patient can make immediately that will have a quick effect. Sleeping on several pillows at night will immediately relieve orthopnea and open the patient's airway, thereby reducing fatigue. Running 30 minutes a day will improve cardiopulmonary health, but a patient needs to build up exercise tolerance. Smoking cessation is another process that many people have difficulty doing immediately. A more realistic short-term goal would be to gradually reduce the number of cigarettes smoked. Limiting caloric intake can help a patient lose weight, but this is a gradual process and is not reasonable for a shortterm goal.

A nurse is caring for a patient with continuous cardiac monitoring for heart dysrhythmias. Which rhythm will cause the nurse to intervene immediately? a. Ventricular tachycardia b. Atrial fibrillation c. Sinus rhythm d. Paroxysmal supraventricular tachycardia

ANS: A Ventricular tachycardia and ventricular fibrillation are life-threatening rhythms that require immediate intervention. Ventricular tachycardia is a life-threatening dysrhythmia because of the decreased cardiac output and the potential to deteriorate into ventricular fibrillation or sudden cardiac death. Atrial fibrillation is a common dysrhythmia in older adults and is not as serious as ventricular tachycardia. Sinus rhythm is normal. Paroxysmal supraventricular tachycardia is a sudden, rapid onset of tachycardia originating above the AV node. It often begins and ends spontaneously

A febrile preschool-aged child presents to the after-hours clinic. Varicella (chickenpox) is diagnosed on the basis of the illness history and the presence of small, circumscribed skin lesions filled with serous fluid. Which type of skin lesion will the nurse report? a. Vesicles b. Wheals c. Papules d. Pustules

ANS: A Vesicles are circumscribed, elevated skin lesions filled with serous fluid that measure less than 1 cm. Wheals are irregularly shaped, elevated areas of superficial localized edema that vary in size. They are common with mosquito bites and hives. Papules are palpable, circumscribed, solid elevations in the skin that are smaller than 1 cm. Pustules are elevations of skin similar to vesicles, but they are filled with pus and vary in size like acne

The nurse is preparing to administer medications to two patients with the same last name. After the administration, the nurse realizes that did not check the identification of the patient before administering medication. Which action should the nurse complete first? a. Return to the room to check and assess the patient. b. Administer the antidote to the patient immediately. c. Alert the charge nurse that a medication error has occurred. d. Complete proper documentation of the medication error in the patient's chart.

ANS: A When an error occurs, the patient's safety and well-being are the top priorities. You first assess and examine the patient's condition and notify the health care provider of the incident as soon as possible. The nurse's first priority is to establish the safety of the patient by assessing the patient. Second, notify the charge nurse and the health care provider. Administer antidote if required. Finally, the nurse needs to complete proper documentation

A nurse is teaching about risk factors for cardiopulmonary disease. Which risk factor should the nurse describe as modifiable? a. Stress b. Allergies c. Family history d. Gender

ANS: A Young and middle-age adults are exposed to multiple cardiopulmonary risk factors: an unhealthy diet, lack of exercise, stress, over-the-counter and prescription drugs not used as intended, illegal substances, and smoking. Reducing these modifiable factors decreases a patient's risk for cardiac or pulmonary diseases. A nonmodifiable risk factor is family history; determine familial risk factors such as a family history of lung cancer or cardiovascular disease. Other nonmodifiable risk factors include allergies and gender.

2. A nurse is assessing several patients. Which assessment findings will cause the nurse to follow up? (Select all that apply.) a. Orthopnea b. Nonpalpable lymph nodes c. Pleural friction rub present d. Crackles in lower lung lobes e. Grade 5 muscle function level f. A 160-degree angle between nail plate and nail

ANS: A, C, D Abnormal findings will cause a nurse to follow up. Orthopnea is abnormal and indicates cardiovascular or respiratory problems. Pleural friction rub is abnormal and indicated an inflamed pleura. Crackles are adventitious breath sounds and indicate random, sudden reinflation of groups of alveoli, indicating disruptive passage of air through small airways. Lymph nodes should be nonpalpable; palpable lymph nodes are abnormal. Grade 5 muscle function is normal. A 160-degree angle between nail plate and nail is normal; a larger degree angle is abnormal and indicates clubbing.

While assessing an adult client's skull, the nurse observes that the client's skull and facial bones are larger and thicker than usual. The nurse should assess the client for a. parotid gland enlargement b. acromegaly c. Paget disease d. cushing syndrome

b. acromegaly

The nurse is planning to assess an adult client's thyroid gland. The nurse should plan to a. ask the client to raise the chin b. approach the client posteriorly c. turn the client's neck slightly backward d. place the fingers above the cricoid cartilage

b. approach the client posteriorly

1. A nurse is assessing a patient's cranial nerve IX. Which items does the nurse gather before conducting the assessment? (Select all that apply.) a. Vial of sugar b. Snellen chart c. Tongue blade d. Ophthalmoscope e. Lemon applicator

ANS: A, C, E Cranial nerve IX is the glossopharyngeal, which controls taste and ability to swallow. The nurse asks the patient to identify sour (lemon) or sweet (sugar) tastes on the back of the tongue and uses a tongue blade to elicit a gag reflex. Ophthalmoscopes are used for vision. A Snellen chart is used to test cranial nerve II (optic).

Who should you verify medications calculations with and why? A. The Nurse Assistant, to ensure accuracy. B. The pharmacy, because only the pharmacist knows accurate calculations. C. The patient, because they are a math teacher. D. Another Nurse, to ensure accuracy.

D. Another nurse, to ensure accuracy.

24. A teen patient is tearful and reports locating lumps in her breasts. Other history obtained is that she is currently menstruating. Physical examination reveals soft and movable cysts in both breasts that are painful to palpation. The nurse also notes that the patient's nipples are erect, but the areola is wrinkled. Which action will the nurse take after talking with the health care provider? a. Reassure patient that her symptoms are normal. b. Discuss the possibility of fibrocystic disease as the probable cause. c. Consult a breast surgeon because of the abnormal nipples and areola. d. Tell the patient that the symptoms may get worse when her period ends.

ANS: B A common benign condition of the breast is benign (fibrocystic) breast disease. This patient has symptoms of fibrocystic disease, which include bilateral lumpy, painful breasts sometimes accompanied by nipple discharge. Symptoms are more apparent during the menstrual period. When palpated, the cysts (lumps) are soft, well differentiated, and movable. Deep cysts feel hard. Although a common condition, benign breast disease is not normal; therefore, the nurse does not tell the patient that this is a normal finding. During examination of the nipples and areolae, the nipple sometimes becomes erect with wrinkling of the areola. Therefore, consulting a breast surgeon to treat her nipples and areolae is not appropriate.

21. A teen female patient reports intermittent abdominal pain for 12 hours. No dysuria is present. Which action will the nurse take when performing an abdominal assessment? a. Assess the area that is most tender first. b. Ask the patient about the color of her stools. c. Recommend that the patient take more laxatives. d. Avoid sexual references such as possible pregnancy.

ANS: B Abdominal pain can be related to bowels. If stools are black or tarry (melena), this may indicate gastrointestinal alteration. The nurse should caution patients about the dangers of excessive use of laxatives or enemas. There is not enough information about the abdominal pain to recommend laxatives. Determine if the patient is pregnant, and note her last menstrual period. Pregnancy causes changes in abdominal shape and contour. Assess painful areas last to minimize discomfort and anxiety.

25. A nurse is performing a mental status examination and asks an adult patient what the statement "Don't cry over spilled milk" means. Which area is the nurse assessing? a. Long-term memory b. Abstract thinking c. Recent memory d. Knowledge

ANS: B For an individual to explain common phrases such as "A stitch in time saves nine" or "Don't cry over spilled milk" requires a higher level of intellectual function or abstract thinking. Knowledge-based assessment is factual. Assess knowledge by asking how much the patient knows about the illness or the reason for seeking health care. To assess past (long-term) memory, ask the patient to recall the maiden name of the patient's mother, a birthday, or a special date in history. It is best to ask open-ended questions rather than simple yes/no questions. Patients demonstrate immediate recall (recent memory) by repeating a series of numbers in the order in which they are presented or in reverse order.

32. The nurse is assessing an adult patient's patellar reflex. Which finding will the nurse record as normal? a. 1+ b. 2+ c. 3+ d. 4+

ANS: B Grade reflexes as follows: 0: No response; 1+: Sluggish or diminished; 2+: Active or expected response; 3+: More brisk than expected, slightly hyperactive; and 4+: Brisk and hyperactive with intermittent or transient clonus.

14. The nurse is examining a female with vaginal discharge. Which position will the nurse place the patient for proper examination? a. Sitting b. Lithotomy c. Knee-chest d. Dorsal recumbent

ANS: B Lithotomy is the position for examination of female genitalia. The lithotomy position provides for the maximum exposure of genitalia and allows the insertion of a vaginal speculum. Sitting does not allow adequate access for speculum insertion and is better used to visualize upper body parts. Dorsal recumbent is used to examine the head and neck, anterior thorax and lungs, breasts, axillae, heart, and abdomen. Knee-chest provides maximal exposure of the rectal area but is embarrassing and uncomfortable.

37. An older-adult patient is taking aminoglycoside for a severe infection. Which assessment is the priority? a. Eyes b. Ears c. Skin d. Reflexes

ANS: B Older adults are especially at risk for hearing loss caused by ototoxicity (injury to auditory nerve) resulting from high maintenance doses of antibiotics (e.g., aminoglycosides). While eyes and skin are important, they are not the priority. Reflexes are expected to be diminished in older adults.

18. An older-adult patient is being seen for chronic entropion. Which condition will the nurse assess for in this patient? a. Ptosis b. Infection c. Borborygmi d. Exophthalmos

ANS: B The diagnosis of entropion can lead to lashes of the lids irritating the conjunctiva and cornea. Irritation can lead to infection. Exophthalmos is a bulging of the eyes and usually indicates hyperthyroidism. An abnormal drooping of the lid over the pupil is called ptosis. In the older adult, ptosis results from a loss of elasticity that accompanies aging. Hyperactive sounds are loud, "growling" sounds called borborygmi, which indicate increased GI motility.

29. While assessing the skin of an 82-year-old patient, a nurse discovers nonpainful, ruby red papules on the patient's trunk. What is the nurse's next action? a. Explain that the patient has basal cell carcinoma and should watch for spread. b. Document cherry angiomas as a normal older adult skin finding. c. Tell the patient that this is a benign squamous cell carcinoma. d. Record the presence of petechiae.

ANS: B The skin is normally free of lesions, except for common freckles or age-related changes such as skin tags, senile keratosis (thickening of skin), cherry angiomas (ruby red papules), and atrophic warts. Basal cell carcinoma is most common in sun-exposed areas and frequently occurs in a background of sun-damaged skin; it almost never spreads to other parts of the body. Squamous cell carcinoma is more serious than basal cell and develops on the outer layers of sun-exposed skin; these cells may travel to lymph nodes and throughout the body. Report abnormal lesions to the health care provider for further examination. Petechiae are nonblanching, pinpoint-size, red or purple spots on the skin caused by small hemorrhages in the skin layers.

A patient is receiving vancomycin. Which function is the priority for the nurses to assess? a. Vision b. Hearing c. Heart tones d. Bowel sounds

ANS: B A side effect of vancomycin is ototoxicity—hearing. It does not affect vision, heart tones, or bowel sounds.

The student nurse studying the gastrointestinal system understands that chyme refers to what? a. Hormones that reduce gastric acidity b. Liquefied food ready for digestion c. Nutrients after being absorbed d. Secretions that help digest food

ANS: B Before being digested, food must be broken down into a liquid form. This liquid is called chyme. Secretin is the hormone that inhibits acid production and decreases gastric motility. Absorption is carried out as the nutrients produced by digestion move from the lumen of the GI tract into the body's circulatory system for uptake by individual cells. The secretions that help digest food include hydrochloric acid, bile, and digestive enzymes.

A client who has been taking antibiotics reports severe, watery diarrhea. About which test does the nurse teach the client? a. Colonoscopy b. Enzyme-linked immunosorbent assay (ELISA) toxin A+B c. Ova and parasites d. Stool culture

ANS: B Clients taking antibiotics are at risk for Clostridium difficile infection. The most common test for this disorder is a stool sample for ELISA toxin A+B. Colonoscopy, ova and parasites, and stool culture are not warranted at this time.

A nurse obtains the health history of a client who is newly admitted to the medical unit. Which statement by the client should alert the nurse to the presence of edema? a. "I wake up to go to the bathroom at night." b. "My shoes fit tighter by the end of the day." c. "I seem to be feeling more anxious lately." d. "I drink at least eight glasses of water a day."

ANS: B Weight gain can result from fluid accumulation in the interstitial spaces. This is known as edema. The nurse should note whether the client feels that his or her shoes or rings are tight, and should observe, when present, an indentation around the leg where the socks end. The other answers do not describe edema.

A client is recovering from an esophagogastroduodenoscopy (EGD) and requests something to drink. What action by the nurse is best? a. Allow the client cool liquids only. b. Assess the client's gag reflex. c. Remind the client to remain NPO. d. Tell the client to wait 4 hours.

ANS: B The local anesthetic used during this procedure will depress the client's gag reflex. After the procedure, the nurse should ensure that the gag reflex is intact before offering food or fluids. The client does not need to be restricted to cool beverages only and is not required to wait 4 hours before oral intake is allowed. Telling the client to remain NPO does not inform the client of when he or she can have fluids, nor does it reflect the client's readiness for them.

A nurse begins infusing a 250-mL bag of IV fluid at 1845 on Monday and programs the pump to infuse at 50 mL/hr. At what time should the infusion be completed? a. 2300 Monday b. 2345 Monday c. 0015 Tuesday d. 0045 Tuesday

ANS: B 250 mL ÷ 50 mL/hr = 5 hr 1845 + 5 hr = 2345, which would be 2345 on Monday.

A health care provider orders lorazepam (Ativan) 1 mg orally 2 times a day. The dose available is 0.5 mg per tablet. How many tablet(s) will the nurse administer for each dose? a. 1 b. 2 c. 3 d. 4

ANS: B The nurse will give 2 tablets. It will take 2 tablets (0.5) to equal 1 mg OR ordered dose (1) over dose on hand (0.5). 1/0.5 = 2 tablets.

A client presents to the emergency department reporting severe abdominal pain. On assessment, the nurse finds a bulging, pulsating mass in the abdomen. What action by the nurse is the priority? a. Auscultate for bowel sounds. b. Notify the provider immediately. c. Order an abdominal flat-plate x-ray. d. Palpate the mass and measure its size.

ANS: B This observation could indicate an abdominal aortic aneurysm, which could be life threatening and should never be palpated. The nurse notifies the provider at once. An x-ray may be indicated. Auscultation is part of assessment, but the nurse's priority action is to notify the provider.

The patient is to receive phenytoin (Dilantin) at 0900. When will be the ideal time for the nurse to schedule a trough level? a. 0800 b. 0830 c. 0900 d. 0930

ANS: B Trough levels are generally drawn 30 minutes before the drug is administered. If the medication is administered at 0900, the trough should be drawn at 0830.

A teen patient is tearful and reports locating lumps in her breasts. Other history obtained is that she is currently menstruating. Physical examination reveals soft and movable cysts in both breasts that are painful to palpation. The nurse also notes that the patient's nipples are erect, but the areola is wrinkled. Which action will the nurse take after talking with the health care provider? a. Reassure patient that her symptoms are normal. b. Discuss the possibility of fibrocystic disease as the probable cause. c. Consult a breast surgeon because of the abnormal nipples and areola. d. Tell the patient that the symptoms may get worse when her period ends

ANS: B A common benign condition of the breast is benign (fibrocystic) breast disease. This patient has symptoms of fibrocystic disease, which include bilateral lumpy, painful breasts sometimes accompanied by nipple discharge. Symptoms are more apparent during the menstrual period. When palpated, the cysts (lumps) are soft, well differentiated, and movable. Deep cysts feel hard. Although a common condition, benign breast disease is not normal; therefore, the nurse does not tell the patient that this is a normal finding. During examination of the nipples and areolae, the nipple sometimes becomes erect with wrinkling of the areola. Therefore, consulting a breast surgeon to treat her nipples and areolae is not appropriate.

A nurse is administering a blood transfusion. Which assessment finding will the nurse report immediately? a. Blood pressure 110/60 b. Temperature 101.3° F c. Poor skin turgor and pallor d. Heart rate of 100 beats/min

ANS: B A fever should be reported immediately and the blood transfusion stopped. All other assessment findings are expected. Blood is given to elevate blood pressure, improve pallor, and decrease tachycardia

The nurse determines that an older-adult patient is at risk for infection due to decreased immunity. Which plan of care best addresses the prevention of infection for the patient? a. Inform the patient of the importance of finishing the entire dose of antibiotics. b. Encourage the patient to stay up-to-date on all vaccinations. c. Schedule patient to get annual tuberculosis skin testing. d. Create an exercise routine to run 45 minutes every day

ANS: B A nursing care plan for preventative health measures should be reasonable and feasible. Keeping upto-date on vaccinations is important because vaccine reduces the severity of illnesses and serious complications. Determine if and when the patient has had a pneumococcal or influenza (flu) vaccine. This is especially important when assessing older adults because of their increased risk for respiratory disease. Although it is important to finish the full course of antibiotics, it is not a preventative health measure. Scheduling annual tuberculosis skin tests does not address prevention and is an unreliable indictor of tuberculosis in older patients. The exercise routine should be reasonable to increase compliance; exercise is recommended only 3 to 4 times a week for 30 to 60 minutes, and walking, rather than running, is an efficient method.

In which patient will the nurse expect to see a positive Chvostek sign? a. A 7-year-old child admitted for severe burns b. A 24-year-old adult admitted for chronic alcohol abuse c. A 50-year-old patient admitted for an acute exacerbation of hyperparathyroidism d. A 75-year-old patient admitted for a broken hip related to osteoporosis

ANS: B A positive Chvostek sign is representative of hypocalcemia or hypomagnesemia. Hypomagnesemia is common with alcohol abuse. Hypocalcemia can be brought on by alcohol abuse and pancreatitis (which also can be affected by alcohol consumption). Burn patients frequently experience extracellular fluid volume deficit. Hyperparathyroidism causes hypercalcemia. Immobility is associated with hypercalcemia.

Which patient using an inhaler would benefit most from using a spacer? a. A 15 year old with a repaired cleft palate who is alert b. A 25 year old with limited coordination of the extremities c. A 50 year old with hearing impairment who uses a hearing aid d. A 72 year old with left-sided hemiparesis using a dry powder inhaler

ANS: B A spacer is indicated for a patient who has difficulty coordinating the steps, like patients with limited mobility/coordination. An alert adolescent with a repaired cleft palate would not need a spacer. Hearing impairment may make teaching the patient to use the inhaler difficult, but it does not indicate the need for a spacer. Although a patient with left-sided hemiparesis could have coordination problems, a patient using a dry powder inhaler does not require the use of spacers.

A teen female patient reports intermittent abdominal pain for 12 hours. No dysuria is present. Which action will the nurse take when performing an abdominal assessment? a. Assess the area that is most tender first. b. Ask the patient about the color of her stools. c. Recommend that the patient take more laxatives. d. Avoid sexual references such as possible pregnancy.

ANS: B Abdominal pain can be related to bowels. If stools are black or tarry (melena), this may indicate gastrointestinal alteration. The nurse should caution patients about the dangers of excessive use of laxatives or enemas. There is not enough information about the abdominal pain to recommend laxatives. Determine if the patient is pregnant, and note her last menstrual period. Pregnancy causes changes in abdominal shape and contour. Assess painful areas last to minimize discomfort and anxiety

The nurse needs to closely monitor the oxygen status of an older-adult patient undergoing anesthesia because of which age-related change? a. Thinner heart valves cause lipid accumulation and fibrosis. b. Diminished respiratory muscle strength may cause poor chest expansion. c. Alterations in mental status prevent patients' awareness of ineffective breathing. d. An increased number of pacemaker cells make proper anesthesia induction more difficult.

ANS: B Age-related changes in the thorax that occur from ossification of costal cartilage, decreased space between vertebrae, and diminished respiratory muscle strength lead to problems with chest expansion and oxygenation,whereby the patient will have difficulty excreting anesthesia gas. The nurse needs to monitor the patient's oxygen status carefully to make sure the patient does not retain too much of the drug. Older adults experience alterations in cardiac function as a result of calcification of the conduction pathways, thicker and stiffer heart valves caused by lipid accumulation and fibrosis, and a decrease in the number of pacemaker cells in the SA node. Altered mental status is not a normal agerelated change; it indicates possible cardiac and/or respiratory problems.

A nurse is caring for a patient who was in a motor vehicle accident that resulted in cervical trauma to C4. Which assessment is the priority? a. Pulse b. Respirations c. Temperature d. Blood pressure

ANS: B Respirations and oxygen saturation are the priorities. Cervical trauma at C3 to C5 usually results in paralysis of the phrenic nerve. When the phrenic nerve is damaged, the diaphragm does not descend properly, thus reducing inspiratory lung volumes and causing hypoxemia. While pulse and blood pressure are important, respirations are the priority. Temperature is not a high priority in this situation.

A patient is at risk for aspiration. Which nursing action is most appropriate? a. Give the patient a straw to control the flow of liquids. b. Have the patient self-administer the medication. c. Thin out liquids so they are easier to swallow. d. Turn the head toward the stronger side

ANS: B Aspiration occurs when food, fluid, or medication intended for GI administration inadvertently enters the respiratory tract. To minimize aspiration risk, allow the patient, if capable, to self-administer medication. Patients should also hold their own cup to control how quickly they take in fluid. Some patients at risk for aspiration may require thickened liquids; thinning liquids does not decrease aspiration risk. Patients at risk for aspiration should not be given straws because use of a straw decreases the control the patient has over volume intake. Turning the head toward the weaker side helps the medication move down the stronger side of the esophagus

A nurse teaches a patient about atelectasis. Which statement by the patient indicates an understanding of atelectasis? a. "Atelectasis affects only those with chronic conditions such as emphysema." b. "It is important to do breathing exercises every hour to prevent atelectasis." c. "If I develop atelectasis, I will need a chest tube to drain excess fluid." d. "Hyperventilation will open up my alveoli, preventing atelectasis."

ANS: B Atelectasis develops when alveoli do not expand. Breathing exercises, especially deep breathing and incentive spirometry, increase lung volume and open the airways, preventing atelectasis. Deep breathing also opens the pores of Kohn between alveoli to allow sharing of oxygen between alveoli. Atelectasis can affect anyone who does not deep breathe. A chest tube is for pneumothorax or hemothorax. It is deep breathing, not hyperventilation, that prevents atelectasis.

A nurse is performing the three accuracy checks before administering an oral liquid medication to a patient. When will the nurse perform the second accuracy check? a. At the patient's bedside b. Before going to the patient's room c. When checking the medication order d. When selecting medication from the unit-dose drawer

ANS: B Before going to the patient's room, compare the patient's name and name of medication on the label of prepared drugs with MAR for the second accuracy check. Selecting the correct medication from the stock supply, unit-dose drawer, or automated dispensing system (ADS) is the first check. The third accuracy check is comparing names of medications on labels with MAR at the patient's bedside. Checking the orders is not one of the three accuracy checks but should be done if there is any confusion about an order.

The nurse is giving an IM injection. Upon aspiration, the nurse notices blood return in the syringe. What should the nurse do? a. Administer the injection at a slower rate. b. Withdraw the needle and prepare the injection again. c. Pull the needle back slightly and inject the medication. d. Give the injection and hold pressure over the site for 3 minutes.

ANS: B Blood return upon aspiration indicates improper placement, and the injection should not be given. Instead withdraw the needle, dispose of the syringe and needle properly, and prepare the medication again. Administering the medication into a blood vessel could have dangerous adverse effects, and the medication will be absorbed faster than intended owing to increased blood flow. Holding pressure is not an appropriate intervention. Pulling back the needle slightly does not guarantee proper placement of the needle and medication administration

The home health nurse recommends that a patient with respiratory problems install a carbon monoxide detector in the home. What is the rationale for the nurse's action? a. Carbon monoxide detectors are required by law in the home. b. Carbon monoxide tightly binds to hemoglobin, causing hypoxia. c. Carbon monoxide signals the cerebral cortex to cease ventilations. d. Carbon monoxide combines with oxygen in the body and produces a deadly toxin.

ANS: B Carbon monoxide binds tightly to hemoglobin; therefore, oxygen is not able to bind to hemoglobin and be transported to tissues, causing hypoxia. A carbon monoxide detector is not required by law, does not signal the cerebral cortex to cease ventilations, and does not combine with oxygen but with hemoglobin to produce a toxin

A nurse is attempting to administer an oral medication to a child, but the child refuses to take the medication. A parent is in the room. Which statement by the nurse to the parent is best? a. "Please hold your child's arms down, so I can give the full dose." b. "I will prepare the medication for you and observe if you would like to try to administer the medication." c. "Let's turn the lights off and give your child a moment to fall asleep before administering the medication." d. "Since your child loves applesauce, let's add the medication to it, so your child doesn't resist."

ANS: B Children often have difficulties taking medication, but it can be less traumatic for the child if the parent administers the medication and the nurse supervises. Another nurse should help restrain a child if needed; the parent acts as a comforter, not a restrainer. Holding down the child is not the best option because it may further upset the child. Never administer an oral medication to a sleeping child. Don't mix medications into the child's favorite foods, because the child might start to refuse the food.

The nurse is educating a student nurse on caring for a patient with a chest tube. Which statement from the student nurse indicates successful learning? a. "I should clamp the chest tube when giving the patient a bed bath." b. "I should report if I see continuous bubbling in the water-seal chamber." c. "I should strip the drains on the chest tube every hour to promote drainage." d. "I should notify the health care provider first, if the chest tube becomes dislodged."

ANS: B Correct care of a chest tube involves knowing normal and abnormal functioning of the tube. A constant or intermittent bubbling in the water-seal chamber indicates a leak in the drainage system, and the health care provider must be notified immediately. Stripping the tube is not routinely performed as it increases pressure. If the tubing disconnects from the drainage unit, instruct the patient to exhale as much as possible and to cough. This maneuver rids the pleural space of as much air as possible. Temporarily reestablish a water seal by immersing the open end of the chest tube into a container of sterile water. The chest tube should not be clamped unless necessary; if so, the length of time clamped would be minimal to reduce the risk of pneumothorax

A patient is admitted for a bowel obstruction and has had a nasogastric tube set to low intermittent suction for the past 3 days. Which arterial blood gas values will the nurse expect to observe? a. Respiratory alkalosis b. Metabolic alkalosis c. Metabolic acidosis d. Respiratory acidosis

ANS: B The patient is losing acid from the nasogastric tube so the patient will have metabolic alkalosis. Lung problems will produce respiratory alkalosis or acidosis. Metabolic acidosis will occur when too much acid is in the body like kidney failure.

A nurse is performing a mental status examination and asks an adult patient what the statement "Don't cry over spilled milk" means. Which area is the nurse assessing? a. Long-term memory b. Abstract thinking c. Recent memory d. Knowledge

ANS: B For an individual to explain common phrases such as "A stitch in time saves nine" or "Don't cry over spilled milk" requires a higher level of intellectual function or abstract thinking. Knowledge-based assessment is factual. Assess knowledge by asking how much the patient knows about the illness or the reason for seeking health care. To assess past (long-term) memory, ask the patient to recall the maiden name of the patient's mother, a birthday, or a special date in history. It is best to ask openended questions rather than simple yes/no questions. Patients demonstrate immediate recall (recent memory) by repeating a series of numbers in the order in which they are presented or in reverse order.

The nurse is assessing an adult patient's patellar reflex. Which finding will the nurse record as normal? a. 1+ b. 2+ c. 3+ d. 4+

ANS: B Grade reflexes as follows: 0: No response; 1+: Sluggish or diminished; 2+: Active or expected response; 3+: More brisk than expected, slightly hyperactive; and 4+: Brisk and hyperactive with intermittent or transient clonus

The nurse is caring for a patient with a tracheostomy tube. Which nursing intervention is most effective in promoting effective airway clearance? a. Suctioning respiratory secretions several times every hour b. Administering humidified oxygen through a tracheostomy collar c. Instilling normal saline into the tracheostomy to thin secretions before suctioning d. Deflating the tracheostomy cuff before allowing the patient to cough up secretions

ANS: B Humidification from air humidifiers or humidified oxygen tracheostomy collars can help prevent drying of secretions that cause occlusion. Suctioning should be done only as needed; too frequent suctioning can damage the mucosal lining, resulting in thicker secretions. Normal saline should not be instilled into a tracheostomy; research showed no benefit with this technique. The purpose of the tracheostomy cuff is to keep secretions from entering the lungs; the nurse should not deflate the tracheostomy cuff unless instructed to do so by the health care provider.

A patient is in need of immediate pain relief for a severe headache. Which medication will the nurse administer to be absorbed the quickest? a. Acetaminophen 650 mg PO b. Hydromorphone 4 mg IV c. Ketorolac 8 mg IM d. Morphine 6 mg SQ

ANS: B IV is the fastest route for absorption owing to the increase in blood flow. The richer the blood supply to the site of administration, the faster a medication is absorbed. Medications administered intravenously enter the bloodstream and act immediately, whereas those given in other routes take time to enter the bloodstream and have an effect. Oral, subcutaneous (SQ), and intramuscular (IM) are others ways to deliver medication but with less blood flow, slowing absorption

The nurse is examining a female with vaginal discharge. Which position will the nurse place the patient for proper examination? a. Sitting b. Lithotomy c. Knee-chest d. Dorsal recumbent

ANS: B Lithotomy is the position for examination of female genitalia. The lithotomy position provides for the maximum exposure of genitalia and allows the insertion of a vaginal speculum. Sitting does not allow adequate access for speculum insertion and is better used to visualize upper body parts. Dorsal recumbent is used to examine the head and neck, anterior thorax and lungs, breasts, axillae, heart, and abdomen. Knee-chest provides maximal exposure of the rectal area but is embarrassing and uncomfortable

The nurse receives the patient's most recent blood work results. Which laboratory value is of greatest concern? a. Sodium of 145 mEq/L b. Calcium of 15.5 mg/dL c. Potassium of 3.5 mEq/L d. Chloride of 100 mEq/L

ANS: B Normal calcium range is 8.4 to 10.5 mg/dL; therefore, a value of 15.5 mg/dL is abnormally high and of concern. The rest of the laboratory values are within their normal ranges: sodium 136 to 145 mEq/L; potassium 3.5 to 5.0 mEq/L; and chloride 98 to 106 mEq/L

An older-adult patient is taking aminoglycoside for a severe infection. Which assessment is the priority? a. Eyes b. Ears c. Skin d. Reflexes

ANS: B Older adults are especially at risk for hearing loss caused by ototoxicity (injury to auditory nerve) resulting from high maintenance doses of antibiotics (e.g., aminoglycosides). While eyes and skin are important, they are not the priority. Reflexes are expected to be diminished in older adults.

The nurse observes that the patient's calcium is elevated. When checking the phosphate level, what does the nurse expect to see? a. Increased b. Decreased c. Equal to calcium d. No change in phosphate

ANS: B Phosphate will decrease. Serum calcium and phosphate have an inverse relationship. When one is elevated, the other decreases, except in some patients with end-stage renal disease.

A nurse is assessing a patient. Which assessment finding should cause a nurse to further assess for extracellular fluid volume deficit? a. Moist mucous membranes b. Postural hypotension c. Supple skin turgor d. Pitting edema

ANS: B Physical examination findings of deficit include postural hypotension, tachycardia, thready pulse, dry mucous membranes, and poor skin turgor. Pitting edema indicates that the patient may be retaining excess extracellular fluid

A nurse is administering oral medications to patients. Which action will the nurse take? a. Remove the medication from the wrapper, and place it in a cup labeled with the patient's information. b. Place all of the patient's medications in the same cup, except medications with assessments. c. Crush enteric-coated medication, and place it in a medication cup with water. d. Measure liquid medication by bringing liquid medication cup to eye level.

ANS: B Placing medications that require preadministration assessment in a separate cup serves as a reminder to check before the medication is given, making it easier for the nurse to withhold medication if necessary. Medications should not be removed from their package until they are in the patient's room because this makes identification of the pill easier and reduces contamination. When measuring a liquid, the nurse should use the meniscus level to measure; make sure it is at eye level on a hard surface like a countertop. Enteric coated medications should not be crushed

A nurse is administering a diuretic to a patient and teaching the patient about foods to increase. Which food choices by the patient will best indicate successful teaching? a. Milk and cheese b. Potatoes and fresh fruit c. Canned soups and vegetables d. Whole grains and dark green leafy vegetables

ANS: B Potatoes and fruits are high in potassium. Milk and cheese are high in calcium. Canned soups and vegetables are high in sodium. Whole grains and dark green leafy vegetables are high in magnesium

Which blood gas result will the nurse expect to observe in a patient with respiratory alkalosis? a. pH 7.60, PaCO2 40 mm Hg, HCO3- 30 mEq/L b. pH 7.53, PaCO2 30 mm Hg, HCO3- 24 mEq/L c. pH 7.35, PaCO2 35 mm Hg, HCO3- 26 mEq/L d. pH 7.25, PaCO2 48 mm Hg, HCO3- 23 mEq/L

ANS: B Respiratory alkalosis should show an alkalotic pH and decreased CO2 (respiratory) values, with a normal HCO3-. In this case, pH 7.53 is alkaline (normal = 7.35 to 7.45), PaCO2 is 30 (normal 35 to 45 mm Hg), and HCO3- is 24 (normal = 22 to 26 mEq/L). A result of pH 7.60, PaCO2 40 mm Hg, HCO3- 30 mEq/L is metabolic alkalosis. pH 7.35, PaCO2 35 mm Hg, HCO3- 26 mEq/L is within normal limits. pH 7.25, PaCO2 48 mm Hg, HCO3- 23 mEq/L is respiratory acidosis.

A 2-year-old child is brought into the emergency department after ingesting a medication that causes respiratory depression. For which acid-base imbalance will the nurse most closely monitor this child? a. Respiratory alkalosis b. Respiratory acidosis c. Metabolic acidosis d. Metabolic alkalosis

ANS: B Respiratory depression leads to hypoventilation. Hypoventilation results in retention of CO2 and respiratory acidosis. Respiratory alkalosis would result from hyperventilation, causing a decrease in CO2 levels. Metabolic acid-base imbalance would be a result of kidney dysfunction, vomiting, diarrhea, or other conditions that affect metabolic acids.

A nurse is caring for a patient whose ECG presents with changes characteristic of hypokalemia. Which assessment finding will the nurse expect? a. Dry mucous membranes b. Abdominal distention c. Distended neck veins d. Flushed skin

ANS: B Signs and symptoms of hypokalemia are muscle weakness, abdominal distention, decreased bowel sounds, and cardiac dysrhythmias. Distended neck veins occur in fluid overload. Thready peripheral pulses indicate hypovolemia. Dry mucous membranes and flushed skin are indicative of dehydration and hypernatremia.

The nurse is suctioning a patient with a tracheostomy tube. Which action will the nurse take? a. Set suction regulator at 150 to 200 mm Hg. b. Limit the length of suctioning to 10 seconds. c. Apply suction while gently rotating and inserting the catheter. d. Liberally lubricate the end of the suction catheter with a water-soluble solution

ANS: B Suctioning passes should be limited to 10 seconds to avoid hypoxemia. Suction for a tracheostomy should be set at 100 to 150 mm Hg. Excessive lubrication can clog the catheter or occlude the airway; lubricant is not necessary for oropharyngeal or artificial airway (tracheostomy) suctioning. Suction should never be applied on insertion

An older-adult patient is being seen for chronic entropion. Which condition will the nurse assess for in this patient? a. Ptosis b. Infection c. Borborygmi d. Exophthalmos

ANS: B The diagnosis of entropion can lead to lashes of the lids irritating the conjunctiva and cornea. Irritation can lead to infection. Exophthalmos is a bulging of the eyes and usually indicates hyperthyroidism. An abnormal drooping of the lid over the pupil is called ptosis. In the older adult, ptosis results from a loss of elasticity that accompanies aging. Hyperactive sounds are loud, "growling" sounds called borborygmi, which indicate increased GI motility.

A nurse is caring for a patient with left-sided hemiparesis who has developed bronchitis and has a heart rate of 105 beats/min, blood pressure of 156/90 mm Hg, and respiration rate of 30 breaths/min. Which nursing diagnosis is apriority? a. Risk for skin breakdown b. Impaired gas exchange c. Activity intolerance d. Risk for infection

ANS: B The most important nursing intervention is to maintain airway and circulation for this patient; therefore, Impaired gas exchange is the first nursing priority. Activity intolerance is a concern but is not the priority in this case. Risk for skin breakdown and Risk for infection are also important but do not address an immediate impairment with physiologic integrity

The nurse is caring for two patients with the same last name. In this situation which right of medication administration is the priority to reduce the chance of an error? a. Right medication b. Right patient c. Right dose d. Right route

ANS: B The nurse should ask the patient to verify identity and should check the patient's ID bracelet against the medication record to ensure right patient. Acceptable patient identifiers include the patient's name, an identification number assigned by a health care facility, or a telephone number. Do not use the patient's room number as an identifier. To identify a patient correctly in an acute care setting, compare the patient identifiers on the MAR with the patient's identification bracelet while at the patient's bedside. Right medication, right dose, and right route are equally as important, but in this situation, right patient is the priority (two patients with the same last name).

A patient prefers not to take the daily allergy pill this morning because it causes drowsiness throughout the day. Which response by the nurse is best? a. "The physician ordered it; therefore, you must take your medication every morning at the same time whether you're drowsy or not." b. "Let's see if we can change the time you take your pill to 9 PM, so the drowsiness occurs when you would normally be sleeping." c. "You can skip this medication on days when you need to be awake and alert." d. "Try to get as much done as you can before you take your pill, so you can sleep in the afternoon."

ANS: B The nurse should use knowledge about the medication to educate the patient about potential response to medications. Then the medication schedule can be altered based on that knowledge. It is the patient's right to refuse medication; however, the nurse should educate the patient on the importance and effects of the medication. Asking a patient to fit a schedule around a medication is unreasonable and will decrease compliance. The nurse should be supportive and should offer solutions to manage medication effects.

While the nurse is changing the ties on a tracheostomy collar, the patient coughs, dislodging the tracheostomy tube. Which action will the nurse take first? a. Press the emergency response button. b. Insert a spare tracheostomy with the obturator. c. Manually occlude the tracheostomy with sterile gauze. d. Place a face mask delivering 100% oxygen over the nose and mouth

ANS: B The nurse's first priority is to establish a stable airway by inserting a spare trach into the patient's airway; ideally an obturator should be used. The nurse could activate the emergency response team if the patient is still unstable after the tracheostomy is placed. A patient with a tracheostomy breathes through the tube, not the nose or mouth; a face mask would not be an effective method of getting air into the lungs. Manually occluding pressure over the tracheostomy site is not appropriate and would block the patient's only airway.

While assessing the skin of an 82-year-old patient, a nurse discovers nonpainful, ruby red papules on the patient's trunk. What is the nurse's next action? a. Explain that the patient has basal cell carcinoma and should watch for spread. b. Document cherry angiomas as a normal older adult skin finding. c. Tell the patient that this is a benign squamous cell carcinoma. d. Record the presence of petechiae.

ANS: B The skin is normally free of lesions, except for common freckles or age-related changes such as skin tags, senile keratosis (thickening of skin), cherry angiomas (ruby red papules), and atrophic warts. Basal cell carcinoma is most common in sun-exposed areas and frequently occurs in a background of sun-damaged skin; it almost never spreads to other parts of the body. Squamous cell carcinoma is more serious than basal cell and develops on the outer layers of sun-exposed skin; these cells may travel to lymph nodes and throughout the body. Report abnormal lesions to the health care provider for further examination. Petechiae are nonblanching, pinpoint-size, red or purple spots on the skin caused by small hemorrhages in the skin layers

Upon auscultation of the patient's chest, the nurse hears a whooshing sound at the fifth intercostal space. What does this finding indicate to the nurse? a. The beginning of the systolic phase b. Regurgitation of the mitral valve c. The opening of the aortic valve d. Presence of orthopnea

ANS: B When regurgitation occurs, there is a backflow of blood into an adjacent chamber. For example, in mitral regurgitation the mitral leaflets do not close completely. When the ventricles contract, blood escapes back into the atria, causing a murmur, or "whooshing" sound. The systolic phase begins with ventricular filling and closing of the aortic valve, which is heard as the first heart sound, S1. Orthopnea is an abnormal condition in which a patient uses multiple pillows when reclining to breathe easier or sits leaning forward with arms elevated

34. The nurse completed assessments on several patients. Which assessment finding will the nurse record as normal? a. Pulse strength 3 b. 1+ pitting edema c. Constricting pupils when directly illuminated d. Hyperactive bowel sounds in all four quadrants

ANS: C A normal finding is pupils constricting when directly illuminated with a penlight. A pulse strength of 3 indicates a full or increased pulse; 2 is normal. 1+ pitting edema is abnormal; there should be no edema for a normal finding. Hyperactive bowel sounds are abnormal and indicate increased GI motility; normal bowel sounds are active.

28. The paramedics transport an adult involved in a motor vehicle accident to the emergency department. On physical examination, the patient's level of consciousness is reported as opening eyes to pain and responding with inappropriate words and flexion withdrawal to painful stimuli. Which value will the nurse report for the patient's Glasgow Coma Scale score? a. 5 b. 7 c. 9 d. 11

ANS: C According to the guidelines of the Glasgow Coma Scale, the patient has a score of 9. Opening eyes to pain is 2 points; inappropriate word use is 3 points; and flexion withdrawal is 4 points. The total for this patient is 2 + 3 + 4 = 9.

27. The nurse considers several new female patients to receive additional teaching on the need for more frequent Pap test and gynecological examinations. Which assessment findings reveal the patient at highest risk for cervical cancer and having the greatest need for patient education? a. 13 years old, nonsmoker, not sexually active b. 15 years old, social smoker, celibate c. 22 years old, smokes 1 pack of cigarettes per day, has multiple sexual partners d. 50 years old, stopped smoking 30 years ago, has history of multiple pregnancies

ANS: C Females considered to be at higher risk include those who smoke, have multiple sex partners, and have a history of sexually transmitted infections. Of all the assessment findings listed, the 22-year-old smoker with multiple sexual partners has the greatest number of risk factors for cervical cancer. The other patients are at lower risk: not sexually active, celibate, and do not smoke.

11. During a sexually transmitted illness presentation to high-school students, the nurse recommends the human papillomavirus (HPV) vaccine series. Which condition is the nurse trying to prevent? a. Breast cancer b. Ovarian cancer c. Cervical cancer d. Testicular cancer

ANS: C Human papillomavirus (HPV) infection increases the person's risk for cervical cancer. HPV vaccine is recommended for females aged 11 to 12 years but can be given to females ages 12 through 26; males can also receive the vaccine. HPV is not a risk factor for breast, ovarian, and testicular cancer.

19. During a school physical examination, the nurse reviews the patient's current medical history. The nurse discovers the patient has allergies. Which assessment finding is consistent with allergies? a. Clubbing b. Yellow discharge c. Pale nasal mucosa d. Puffiness of nasal mucosa

ANS: C Pale nasal mucosa with clear discharge indicates allergy. Clubbing is due to insufficient oxygenation at the periphery resulting from conditions such as chronic emphysema and congenital heart disease; it is noted in the nails. A sinus infection results in yellowish or greenish discharge. Habitual use of intranasal cocaine and opioids causes puffiness and increased vascularity of the nasal mucosa.

17. An advanced practice nurse is preparing to assess the external genitalia of a 25-year-old American woman of Chinese descent. Which action will the nurse do first? a. Place the patient in the lithotomy position. b. Drape the patient to enhance patient comfort. c. Assess the patient's feelings about the examination. d. Ask the patient if she would like her mother to be present in the room.

ANS: C Patients who are Chinese American often believe that examination of the external genitalia is offensive. Before proceeding with the examination, the nurse first determines how the patient feels about the procedure and explains the procedure to answer any questions and to help the patient feel comfortable with the assessment. Once the patient is ready to have her external genitalia examined, the nurse places the patient in the lithotomy position and drapes the patient appropriately. Typically, nurses ask adolescents if they want a parent present during the examination. The patient in this question is 25 years old; asking if she would like her mother to be present is inappropriate.

3. A nurse is preparing to perform a complete physical examination on a weak, older-adult patient with bilateral basilar pneumonia. Which position will the nurse use? a. Prone b. Sims' c. Supine d. Lateral recumbent

ANS: C Supine is the most normally relaxed position. If the patient becomes short of breath easily, raise the head of the bed. Supine position would be easiest for a weak, older-adult person during the examination. Lateral recumbent and prone positions cause respiratory difficulty for any patient with respiratory difficulties. Sims' position is used for assessment of the rectum and the vagina.

31. A nurse is auscultating different areas on an adult patient. Which technique should the nurse use during an assessment? a. Uses the bell to listen for lung sounds b. Uses the diaphragm to listen for bruits c. Uses the diaphragm to listen for bowel sounds d. Uses the bell to listen for high-pitched murmurs

ANS: C The bell is best for hearing low-pitched sounds such as vascular (bruits) and certain heart sounds (low-pitched murmurs), and the diaphragm is best for listening to high-pitched sounds such as bowel and lung sounds and high-pitched murmurs.

38. The patient has had a stroke that has affected the ability to speak. The patient becomes extremely frustrated when trying to speak. The patient responds correctly to questions and instructions but cannot form words coherently. Which type of aphasia is the patient experiencing? a. Sensory b. Receptive c. Expressive d. Combination

ANS: C The two types of aphasias are sensory (or receptive) and motor (or expressive). The patient cannot form words coherently, indicating expressive or motor aphasia is present. The patient responds correctly to questions and instructions, indicating receptive or sensory aphasia is not present. Patients sometimes suffer a combination of receptive and expressive aphasia, but this is not the case here.

15. On admission, a patient weighs 250 pounds. The weight is recorded as 256 pounds on the second inpatient day. Which condition will the nurse assess for in this patient? a. Anorexia b. Weight loss c. Fluid retention d. Increased nutritional intake

ANS: C This patient has gained 6 pounds in a 24-hour period. A weight gain of 5 pounds (2.3 kg) or more in a day indicates fluid retention problems, not nutritional intake. A weight loss is considered significant if the patient has lost more than 5% of body weight in a month or 10% in 6 months. A downward trend may indicate a reduction in nutritional reserves that may be caused by decreased intake such as anorexia.

36. The nurse is assessing skin turgor. Which technique will the nurse use? a. Press lightly on the forearm. b. Press lightly on the fingertips. c. Grasp a fold of skin on the sternal area. d. Grasp a fold of skin on the back of the hand.

ANS: C To assess skin turgor, grasp a fold of skin on the back of the forearm or sternal area with the fingertips and release. Since the skin on the back of the hand is normally loose and thin, turgor is not reliably assessed at that site. Pressing lightly on the forearm can be used to assess for pitting edema or pain or sense of touch. Pressing lightly on the fingertips and observing nail color is assessing capillary refill.

39. The nurse is assessing the tympanic membranes of an infant. Which action by the nurse demonstrates proper technique? a. Pulls the auricle upward and backward. b. Holds handle of the otoscope between the thumb and little finger. c. Uses an inverted otoscope grip while pulling the auricle downward and back. d. Places the handle of the otoscope between the thumb and index finger while pulling the auricle upward.

ANS: C Using the inverted otoscope grip while pulling the auricle downward and back is a common approach with infant/child examinations because it prevents accidental movement of the otoscope deeper into the ear canal, as could occur with an unexpected pediatric reaction to the ear examination. The other techniques could result in injury to the infant's tympanic membrane. Insert the scope while pulling the auricle upward and backward in the adult and older child. Hold the handle of the otoscope in the space between the thumb and index finger, supported on the middle finger.

35. The patient presents to the clinic with dysuria and hematuria. How does the nurse proceed to assess for kidney inflammation? a. Uses deep palpation posteriorly. b. Lightly palpates each abdominal quadrant. c. Percusses posteriorly the costovertebral angle at the scapular line. d. Inspects abdomen for abnormal movement or shadows using indirect lighting.

ANS: C With the patient sitting or standing erect, use direct or indirect percussion to assess for kidney inflammation. With the ulnar surface of the partially closed fist, percuss posteriorly the costovertebral angle at the scapular line. If the kidneys are inflamed, the patient feels tenderness during percussion. Use a systematic palpation approach for each quadrant of the abdomen to assess for muscular resistance, distention, abdominal tenderness, and superficial organs or masses. Light palpation would not detect kidney tenderness because the kidneys sit deep within the abdominal cavity. Posteriorly, the lower ribs and heavy back muscles protect the kidneys, so they cannot be palpated. Kidney inflammation will not cause abdominal movement. However, to inspect the abdomen for abnormal movement or shadows, the nurse should stand on the patient's right side and inspect from above the abdomen using direct light over the abdomen.

A nurse assesses a client who is recovering after a left-sided cardiac catheterization. Which assessment finding requires immediate intervention? a. Urinary output less than intake b. Bruising at the insertion site c. Slurred speech and confusion d. Discomfort in the left leg

ANS: C A left-sided cardiac catheterization specifically increases the risk for a cerebral vascular accident. A change in neurologic status needs to be acted on immediately. Discomfort and bruising are expected at the site. If intake decreases, a client can become dehydrated because of dye excretion. The second intervention would be to increase the client's fluid status. Neurologic changes would take priority.

A client had a colonoscopy and biopsy yesterday and calls the gastrointestinal clinic to report a spot of bright red blood on the toilet paper today. What response by the nurse is best? a. Ask the client to call back if this happens again today. b. Instruct the client to go to the emergency department. c. Remind the client that a small amount of bleeding is possible. d. Tell the client to come in to the clinic this afternoon.

ANS: C After a colonoscopy with biopsy, a small amount of bleeding is normal. The nurse should remind the client of this and instruct him or her to go to the emergency department for large amounts of bleeding, severe pain, or dizziness.

The nurse is preparing to administer an injection into the deltoid muscle of an adult patient. Which needle size and length will the nurse choose? a. 18 gauge × 1 1/2 inch b. 23 gauge × 1/2 inch c. 25 gauge × 1 inch d. 27 gauge × 5/8 inch

ANS: C For an intramuscular injection into an adult deltoid muscle, a 25-gauge, 1-inch needle is recommended. An 18-gauge needle is too big. While a 23-gauge needle can be used, a 1/2-inch needle is too small. A 27-gauge, 5/8 -inch needle is used for intradermal.

A nurse assesses an older adult client who is experiencing a myocardial infarction. Which clinical manifestation should the nurse expect? a. Excruciating pain on inspiration b. Left lateral chest wall pain c. Disorientation and confusion d. Numbness and tingling of the arm

ANS: C In older adults, disorientation or confusion may be the major manifestation of myocardial infarction caused by poor cardiac output. Pain manifestations and numbness and tingling of the arm could also be related to the myocardial infarction. However, the nurse should be more concerned about the new onset of disorientation or confusion caused by decreased perfusion.

A nurse assesses a client 2 hours after a cardiac angiography via the left femoral artery. The nurse notes that the left pedal pulse is weak. Which action should the nurse take? a. Elevate the leg and apply a sandbag to the entrance site. b. Increase the flow rate of intravenous fluids. c. Assess the color and temperature of the left leg. d. Document the finding as "left pedal pulse of +1/4."

ANS: C Loss of a pulse distal to an angiography entry site is serious, indicating a possible arterial obstruction. The pulse may be faint because of edema. The left pulse should be compared with the right, and pulses should be compared with previous assessments, especially before the procedure. Assessing color (pale, cyanosis) and temperature (cool, cold) will identify a decrease in circulation. Once all peripheral and vascular assessment data are acquired, the primary health care provider should be notified. Simply documenting the findings is inappropriate. The leg should be positioned below the level of the heart or dangling to increase blood flow to the distal portion of the leg. Increasing intravenous fluids will not address the client's problem.

The assistive personnel reports that an older patient is complaining of shortness of breath and palpitations. The nurse connects the patient to an electrocardiogram monitor and analyzes the rhythm with normal P wave and normal QRS and T waves. The rate is 116 beats per minute and regular. The nurse identifies this rhythm as which of the following? a. Sinus bradycardia b. Ventricular tachycardia c. Sinus tachycardia d. Normal sinus rhythm

C

When the nurse administers an IM corticosteroid injection, the nurse aspirates. What is the rationale for the nurse aspirating? a. Prevent the patient from choking. b. Increase the force of the injection. c. Ensure proper placement of the needle. d. Reduce the discomfort of the injection.

ANS: C The purpose of aspiration is to ensure that the needle is in the muscle and not in the vascular system. Blood return upon aspiration indicates improper placement, and the injection should not be given. While a patient can aspirate fluid and food into the lungs, this is not related to the reason for why a nurse pulls back the syringe plunger after inserting the needle (aspirates) before injecting the medication. Reducing discomfort and prolonging absorption time are not reasons for aspirating medications.

The nurse suspects the patient has increased afterload. Which piece of equipment should the nurse obtain to determine the presence of this condition? a. Pulse oximeter b. Oxygen cannula c. Blood pressure cuff d. Yankauer suction tip catheter

ANS: C A blood pressure cuff is needed. The diastolic aortic pressure is a good clinical measure of afterload. Afterload is the resistance to left ventricular ejection. In hypertension the afterload increases, making cardiac workload also increase. A pulse oximeter is used to monitor the level of arterial oxygen saturation; it will not help determine increased afterload. While an oxygen cannula may be needed to help decrease the effects of increased afterload, it will not help determine the presence of afterload. A Yankauer suction tip catheter is used to suction the oral cavity

The nurse completed assessments on several patients. Which assessment finding will the nurse record as normal? a. Pulse strength 3 b. 1+ pitting edema c. Constricting pupils when directly illuminated d. Hyperactive bowel sounds in all four quadrants

ANS: C A normal finding is pupils constricting when directly illuminated with a penlight. A pulse strength of 3 indicates a full or increased pulse; 2 is normal. 1+ pitting edema is abnormal; there should be no edema for a normal finding. Hyperactive bowel sounds are abnormal and indicate increased GI motility; normal bowel sounds are active.

The paramedics transport an adult involved in a motor vehicle accident to the emergency department. On physical examination, the patient's level of consciousness is reported as opening eyes to pain and responding with inappropriate words and flexion withdrawal to painful stimuli. Which value will the nurse report for the patient's Glasgow Coma Scale score? a. 5 b. 7 c. 9 d. 11

ANS: C According to the guidelines of the Glasgow Coma Scale, the patient has a score of 9. Opening eyes to pain is 2 points; inappropriate word use is 3 points; and flexion withdrawal is 4 points. The total for this patient is 2 + 3 + 4 = 9.

The health care provider asks the nurse to monitor the fluid volume status of a heart failure patient and a patient at risk for clinical dehydration. Which is the most effective nursing intervention for monitoring both of these patients? a. Assess the patients for edema in extremities. b. Ask the patients to record their intake and output. c. Weigh the patients every morning before breakfast. d. Measure the patients' blood pressures every 4 hours.

ANS: C An effective measure of fluid retention or loss is daily weights; each kilogram (2.2 pounds) change is equivalent to 1 liter of fluid gained or lost. This measurement should be performed at the same time every day using the same scale and the same amount of clothing. Although intake and output records are important assessment measures, some patients are not able to keep their own records themselves. Blood pressure can decrease with extracellular volume (ECV) deficit but will not necessarily increase with recent ECV excess (heart failure patient). Edema occurs with ECV excess but not with clinical dehydration.

A nurse auscultates heart sounds. When the nurse hears S2, which valves is the nurse hearing close? a. Aortic and mitral b. Mitral and tricuspid c. Aortic and pulmonic d. Mitral and pulmonic

ANS: C As the ventricles empty, the ventricular pressures decrease, allowing closure of the aortic and pulmonic valves, producing the second heart sound, S2. The mitral and tricuspid produce the first heart sound, S1. The aortic and mitral do not close at the same time. The mitral and pulmonic do not close at the same time

A patient has heart failure and cardiac output is decreased. Which formula can the nurse use to calculate cardiac output? a. Myocardial contractility × Myocardial blood flow b. Ventricular filling time/Diastolic filling time c. Stroke volume × Heart rate d. Preload/Afterload

ANS: C Cardiac output can be calculated by multiplying the stroke volume and the heart rate. The other options are not measures of cardiac output

The nurse considers several new female patients to receive additional teaching on the need for more frequent Pap test and gynecological examinations. Which assessment findings reveal the patient at highest risk for cervical cancer and having the greatest need for patient education? a. 13 years old, nonsmoker, not sexually active b. 15 years old, social smoker, celibate c. 22 years old, smokes 1 pack of cigarettes per day, has multiple sexual partners d. 50 years old, stopped smoking 30 years ago, has history of multiple pregnancies

ANS: C Females considered to be at higher risk include those who smoke, have multiple sex partners, and have a history of sexually transmitted infections. Of all the assessment findings listed, the 22-yearold smoker with multiple sexual partners has the greatest number of risk factors for cervical cancer. The other patients are at lower risk: not sexually active, celibate, and do not smoke

During a sexually transmitted illness presentation to high-school students, the nurse recommends the human papillomavirus (HPV) vaccine series. Which condition is the nurse trying to prevent? a. Breast cancer b. Ovarian cancer c. Cervical cancer d. Testicular cancer

ANS: C Human papillomavirus (HPV) infection increases the person's risk for cervical cancer. HPV vaccine is recommended for females aged 11 to 12 years but can be given to females ages 12 through 26; males can also receive the vaccine. HPV is not a risk factor for breast, ovarian, and testicular cancer

The nurse is evaluating the effectiveness of the intravenous fluid therapy in a patient with hypernatremia. Which finding indicates goal achievement? a. Urine output increases to 150 mL/hr. b. Systolic and diastolic blood pressure decreases. c. Serum sodium concentration returns to normal. d. Large amounts of emesis and diarrhea decrease.

ANS: C Hypernatremia is diagnosed by elevated serum sodium concentration. Blood pressure is not an accurate indicator of hypernatremia. Emesis and diarrhea will not stop because of intravenous therapy. Urine output is influenced by many factors, including extracellular fluid volume. A large dilute urine output can cause further hypernatremia.

A nurse has just received a bag of packed red blood cells (RBCs) for a patient. What is the longest time the nurse can let the blood infuse? a. 30 minutes b. 2 hours c. 4 hours d. 6 hours

ANS: C Ideally a unit of whole blood or packed RBCs is transfused in 2 hours. This time can be lengthened to 4 hours if the patient is at risk for extracellular volume excess. Beyond 4 hours there is a risk for bacterial contamination of the blood.

A patient is to receive a proton pump inhibitor through a nasogastric (NG) feeding tube. Which is the mostimportant nursing action to ensure effective absorption? a. Thoroughly shake the medication before administering. b. Position patient in the supine position for 30 minutes to 1 hour. c. Hold feeding for at least 30 minutes after medication administration. d. Flush tube with 10 to 15 mL of water, after all medications are administered

ANS: C If a medication needs to be given on an empty stomach or is not compatible with the feeding (e.g., phenytoin, carbamazepine [Tegretol], warfarin [Coumadin], fluoroquinolones, proton pump inhibitors), hold the feeding for at least 30 minutes before or 30 minutes after medication administration. Thoroughly shaking the medication mixes the medication before administration but does not affect absorption. Flushing the tube after all medications should be 30 to 60 mL of water; 15 to 30 mL of water is used for flushing between medications. Patients with NG tubes should never be positioned supine but instead should be positioned at least to a 30-degree angle to prevent aspiration, provided no contraindication condition is known

A nurse is caring for a 5-year-old patient whose temperature is 101.2° F. The nurse expects this patient to hyperventilate. Which factor does the nurse remember when planning care for this type of hyperventilation? a. Anxiety over illness b. Decreased drive to breathe c. Increased metabolic demands d. Infection destroying lung tissues

ANS: C Increased body temperature (fever) increases the metabolic rate, thereby increasing carbon dioxide production. The increased carbon dioxide level stimulates an increase in the patient's rate and depth of respiration, causing hyperventilation. Anxiety can cause hyperventilation, but this is not the direct cause from a fever. Sleep causes a decreased respiratory drive; hyperventilation speeds up breathing. The cause of the fever in this question is unknown.

While performing an assessment, the nurse hears crackles in the patient's lung fields. The nurse also learns that the patient is sleeping on three pillows to help with the difficulty breathing during the night. Which condition will the nurse most likely observe written in the patient's medical record? a. Atrial fibrillation b. Myocardial ischemia c. Left-sided heart failure d. Right-sided heart failure

ANS: C Left-sided heart failure results in pulmonary congestion, the signs and symptoms of which include shortness of breath, cough, crackles, and paroxysmal nocturnal dyspnea (difficulty breathing when lying flat). Right-sided heart failure is systemic and results in peripheral edema, weight gain, and distended neck veins. Atrial fibrillation is often described as an irregularly irregular rhythm; rhythm is irregular because of the multiple pacemaker sites. Myocardial ischemia results when the supply of blood to the myocardium from the coronary arteries is insufficient to meet myocardial oxygen demands, producing angina or myocardial infarction

A patient is to receive 1000 mL of 0.9% sodium chloride intravenously at a rate of 125 mL/hr. The nurse is using microdrip gravity drip tubing. Which rate will the nurse calculate for the minute flow rate (drops/min)? a. 12 drops/min b. 24 drops/min c. 125 drops/min d. 150 drops/min

ANS: C Microdrip tubing delivers 60 drops/mL. Calculation for a rate of 125 mL/hr using microdrip tubing: (125 mL/1 hr)(60 drops/1 mL)(1 hr/60 min) = 125 drop/min.

A patient with a pneumothorax has a chest tube inserted and is placed on low constant suction. Which finding requires immediate action by the nurse? a. The patient reports pain at the chest tube insertion site that increases with movement. b. Fifty milliliters of blood gushes into the drainage device after the patient coughs. c. No bubbling is present in the suction control chamber of the drainage device. d. Yellow purulent discharge is seen leaking out from around the dressing site.

ANS: C No bubbling in the suction control chamber indicates an obstruction of the drainage system. An obstruction causes increased pressure, which can cause a tension pneumothorax, which can be life threatening. The nurse needs to determine whether the leak is inside the thorax or in the tubing and act from there. Occasional blood gushes from the lung owing to lung expansion, as during a cough; this is reserve drainage. Drainage over 100 mL/hr after 3 hours of chest tube placement is cause for concern. Yellow purulent drainage indicates an infection that should be reported to the health care provider but is not as immediately life threatening as the lack of bubbling in the suction control chamber.

During a school physical examination, the nurse reviews the patient's current medical history. The nurse discovers the patient has allergies. Which assessment finding is consistent with allergies? a. Clubbing b. Yellow discharge c. Pale nasal mucosa d. Puffiness of nasal mucosa

ANS: C Pale nasal mucosa with clear discharge indicates allergy. Clubbing is due to insufficient oxygenation at the periphery resulting from conditions such as chronic emphysema and congenital heart disease; it is noted in the nails. A sinus infection results in yellowish or greenish discharge. Habitual use of intranasal cocaine and opioids causes puffiness and increased vascularity of the nasal mucosa

A nurse is caring for a patient who is taking warfarin (Coumadin) and discovers that the patient is taking garlic to help with hypertension. Which condition will the nurse assess for in this patient? a. Increased cholesterol level b. Distended jugular vein c. Bleeding d. Angina

ANS: C Patients taking warfarin (Coumadin) for anticoagulation prolong the prothrombin time (PT)/international normalized ratio (INR) results if they are taking gingko biloba, garlic, or ginseng with the anticoagulant. The drug interaction can precipitate a life-threatening bleed. Increased cholesterol levels are associated with saturated fat dietary intake. A distended jugular vein and peripheral edema are associated with damage to the right side of the heart. Angina is temporary ischemia of the heart muscle.

An advanced practice nurse is preparing to assess the external genitalia of a 25-year-old American woman of Chinese descent. Which action will the nurse do first? a. Place the patient in the lithotomy position. b. Drape the patient to enhance patient comfort. c. Assess the patient's feelings about the examination. d. Ask the patient if she would like her mother to be present in the room

ANS: C Patients who are Chinese American often believe that examination of the external genitalia is offensive. Before proceeding with the examination, the nurse first determines how the patient feels about the procedure and explains the procedure to answer any questions and to help the patient feel comfortable with the assessment. Once the patient is ready to have her external genitalia examined, the nurse places the patient in the lithotomy position and drapes the patient appropriately. Typically, nurses ask adolescents if they want a parent present during the examination. The patient in this question is 25 years old; asking if she would like her mother to be present is inappropriate

A patient presents to the emergency department with reports of vomiting and diarrhea for the past 48 hours. The health care provider orders isotonic intravenous (IV) therapy. Which IV will the nurse prepare? a. 0.225% sodium chloride (1/4 NS) b. 0.45% sodium chloride (1/2 NS) c. 0.9% sodium chloride (NS) d. 3% sodium chloride (3% NaCl)

ANS: C Patients with prolonged vomiting and diarrhea become hypovolemic. A solution to replace extracellular volume is 0.9% sodium chloride, which is an isotonic solution. 0.225% and 0.45% sodium chloride are hypotonic. 3% sodium chloride is hypertonic

A nurse is auscultating different areas on an adult patient. Which technique should the nurse use during an assessment? a. Uses the bell to listen for lung sounds b. Uses the diaphragm to listen for bruits c. Uses the diaphragm to listen for bowel sounds d. Uses the bell to listen for high-pitched murmurs

ANS: C The bell is best for hearing low-pitched sounds such as vascular (bruits) and certain heart sounds (low-pitched murmurs), and the diaphragm is best for listening to high-pitched sounds such as bowel and lung sounds and high-pitched murmurs.

Which nursing intervention is most effective in preventing hospital-acquired pneumonia in an older-adult patient? a. Discontinue the humidification delivery device to keep excess fluid from lungs. b. Monitor oxygen saturation, and frequently auscultate lung bases. c. Assist the patient to cough, turn, and deep breathe every 2 hours. d. Decrease fluid intake to 300 mL a shift

ANS: C The goal of the nursing action should be the prevention of pneumonia; the action that best addresses this is to cough, turn, and deep breathe to keep secretions from pooling at the base of the lungs. Humidification thins respiratory secretions, making them easier to expel and should be used. Monitoring oxygen status is important but is not a method of prevention. Hydration assists in preventing hospital-acquired pneumonia. The best way to maintain thin secretions is to provide a fluid intake of 1500 to 2500 mL/day unless contraindicated by cardiac or renal status. Restricting fluids is contraindicated in this situation since there is no data indicating cardiac or renal disease.

While the nurse is taking a patient history, the nurse discovers the patient has a type of diabetes that results from a head injury and does not require insulin. Which dietary change should the nurse share with the patient? a. Reduce the quantity of carbohydrates ingested to lower blood sugar. b. Include a serving of dairy in each meal to elevate calcium levels. c. Drink plenty of fluids throughout the day to stay hydrated. d. Avoid food high in acid to avoid metabolic acidosis.

ANS: C The patient has diabetes insipidus, which places the patient at risk for dehydration and hypernatremia. Dehydration should be prevented by drinking plenty of fluids to replace the extra water excreted in the urine. Foods high in acid are not what causes metabolic acidosis. A reduction in carbohydrates to lower blood sugar will not help a patient with diabetes insipidus but it may help a patient with diabetes mellitus. Calcium-rich dairy products would be recommended for a hypocalcemic patient

The nurse is assessing a patient and finds crackles in the lung bases and neck vein distention. Which action will the nurse take first? a. Offer calcium-rich foods. b. Administer diuretic. c. Raise head of bed. d. Increase fluids.

ANS: C The patient is in fluid overload. Raising the head of the bed to ease breathing is the first action. Offering calcium-rich foods is for hypocalcemia, not fluid overload. Administering a diuretic is the second action. Increasing fluids is contraindicated and would make the situation worse.

A nurse is preparing a patient for nasotracheal suctioning. In which order will the nurse perform the steps, beginning with the first step? 1. Insert catheter. 2. Apply suction and remove. 3. Have patient deep breathe. 4. Encourage patient to cough. 5. Attach catheter to suction system. 6. Rinse catheter and connecting tubing. a. 1, 2, 3, 4, 5, 6 b. 4, 5, 1, 2, 3, 6 c. 5, 3, 1, 2, 4, 6 d. 3, 1, 2, 5, 4, 6

ANS: C The steps for nasotracheal suctioning are as follows: Verify that catheter is attached to suction; have patient deep breathe; insert catheter; apply intermittent suction for no more than 10 seconds and remove; encourage patient to cough; and rinse catheter and connecting tubing with normal saline

The supervising nurse is observing several different nurses. Which action will cause the supervising nurse to intervene? a. A nurse administers a vaccine without aspirating. b. A nurse gives an IV medication through a 22-gauge IV needle without blood return. c. A nurse draws up the NPH insulin first when mixing a short-acting and intermediate-acting insulin. d. A nurse calls the health care provider for a patient with nasogastric suction and orders for oral meds

ANS: C The supervising nurse must intervene with the nurse who is drawing up the NPH insulin first; if regular and intermediate-acting (NPH) insulin is ordered, prepare the regular insulin first to prevent the regular insulin from becoming contaminated with the intermediate-acting insulin. All the other actions are appropriate and do not need follow-up. The CDC no longer recommends aspiration when administering immunizations to reduce discomfort. In some cases, especially with a smaller gauge (22) IV needle, blood return is not aspirated, even if the IV is patent. If the IV site shows no signs of infiltration and IV fluid is infusing without difficulty, proceed with IV push slowly. Oral meds are contraindicated in patients with nasogastric suction.

The patient has had a stroke that has affected the ability to speak. The patient becomes extremely frustrated when trying to speak. The patient responds correctly to questions and instructions but cannot form words coherently. Which type of aphasia is the patient experiencing? a. Sensory b. Receptive c. Expressive d. Combination

ANS: C The two types of aphasias are sensory (or receptive) and motor (or expressive). The patient cannot form words coherently, indicating expressive or motor aphasia is present. The patient responds correctly to questions and instructions, indicating receptive or sensory aphasia is not present. Patients sometimes suffer a combination of receptive and expressive aphasia, but this is not the case here

An older-adult patient needs an IM injection of antibiotic. Which site is best for the nurse to use? a. Deltoid b. Dorsal gluteal c. Ventrogluteal d. Vastus lateralis

ANS: C The ventrogluteal site is the preferred and safest site for all adults, children, and infants. While the vastus lateralis is a large muscle that could be used it is not the preferred and safest. The dorsal gluteal site is a location for a subcutaneous injection, and this patient requires an IM injection. The deltoid is easily accessible, but this muscle is not well developed and is not the preferred site.

On admission, a patient weighs 250 pounds. The weight is recorded as 256 pounds on the second inpatient day. Which condition will the nurse assess for in this patient? a. Anorexia b. Weight loss c. Fluid retention d. Increased nutritional intake

ANS: C This patient has gained 6 pounds in a 24-hour period. A weight gain of 5 pounds (2.3 kg) or more in a day indicates fluid retention problems, not nutritional intake. A weight loss is considered significant if the patient has lost more than 5% of body weight in a month or 10% in 6 months. A downward trend may indicate a reduction in nutritional reserves that may be caused by decreased intake such as anorexia

The nurse is assessing skin turgor. Which technique will the nurse use? a. Press lightly on the forearm. b. Press lightly on the fingertips. c. Grasp a fold of skin on the sternal area. d. Grasp a fold of skin on the back of the hand

ANS: C To assess skin turgor, grasp a fold of skin on the back of the forearm or sternal area with the fingertips and release. Since the skin on the back of the hand is normally loose and thin, turgor is not reliably assessed at that site. Pressing lightly on the forearm can be used to assess for pitting edema or pain or sense of touch. Pressing lightly on the fingertips and observing nail color is assessing capillary refill

A nurse is caring for a cancer patient who presents with anorexia, blood pressure 100/60, and elevated white blood cell count. Which primary purpose for starting total parenteral nutrition (TPN) will the nurse add to the care plan? a. Stimulate the patient's appetite to eat. b. Deliver antibiotics to fight off infection. c. Replace fluid, electrolytes, and nutrients. d. Provide medication to raise blood pressure

ANS: C Total parenteral nutrition is an intravenous solution composed of nutrients and electrolytes to replace the ones the patient is not eating or losing. TPN does not stimulate the appetite. TPN does not contain blood pressure medication or antibiotics

A nurse is teaching a health class about the heart. Which information from the class members indicates teaching by the nurse is successful for the flow of blood through the heart, starting in the right atrium? a. Right ventricle, left ventricle, left atrium b. Left atrium, right ventricle, left ventricle c. Right ventricle, left atrium, left ventricle d. Left atrium, left ventricle, right ventricle

ANS: C Unoxygenated blood flows through the venae cavae into the right atrium, where it is pumped down to the right ventricle; the blood is then pumped out the pulmonary artery and is returned oxygenated via the pulmonary vein to the left atrium, where it flows to the left ventricle and is pumped out to the rest of the body via the aorta

The nurse is assessing the tympanic membranes of an infant. Which action by the nurse demonstrates proper technique? a. Pulls the auricle upward and backward. b. Holds handle of the otoscope between the thumb and little finger. c. Uses an inverted otoscope grip while pulling the auricle downward and back. d. Places the handle of the otoscope between the thumb and index finger while pulling the auricle upward.

ANS: C Using the inverted otoscope grip while pulling the auricle downward and back is a common approach with infant/child examinations because it prevents accidental movement of the otoscope deeper into the ear canal, as could occur with an unexpected pediatric reaction to the ear examination. The other techniques could result in injury to the infant's tympanic membrane. Insert the scope while pulling the auricle upward and backward in the adult and older child. Hold the handle of the otoscope in the space between the thumb and index finger, supported on the middle finger.

A nurse is preparing to start a blood transfusion. Which type of tubing will the nurse obtain? a. Two-way valves to allow the patient's blood to mix and warm the blood transfusing b. An injection port to mix additional electrolytes into the blood c. A filter to ensure that clots do not enter the patient d. An air vent to let bubbles into the blood

ANS: C When administering a transfusion you need an appropriate-size IV catheter and blood administration tubing that has a special in-line filter. The patient's blood should not be mixed with the infusion blood. Air bubbles should not be allowed to enter the blood. The only substance compatible with blood is normal saline; no additives should be mixed with the infusing blood.

A nurse is preparing to administer a medication from a vial. In which order will the nurse perform the steps, starting with the first step? 1. Invert the vial. 2. Fill the syringe with medication. 3. Inject air into the airspace of the vial. 4. Clean with alcohol swab and allow to dry. 5. Pull back on the plunger the amount to be drawn up. 6. Tap the side of the syringe barrel to remove air bubbles. a. 4, 1, 5, 3, 6, 2 b. 1, 4, 5, 3, 2, 6 c. 4, 5, 3, 1, 2, 6 d. 1, 4, 5, 3, 6, 2

ANS: C When preparing medication from a vial, the steps are as follows: Firmly and briskly wipe the surface of the rubber seal with an alcohol swab and allow to dry; pull back on the plunger to draw an amount of air into the syringe equal to the volume of medication to be aspirated from the vial; inject air into the airspace of the vial; invert the vial while keeping firm hold on the syringe and plunger; fill the syringe with medication; and tap the side of the syringe barrel carefully to dislodge any air bubbles

A patient's heart rate increased from 94 to 164 beats/min. What will the nurse expect? a. Increase in diastolic filling time b. Decrease in hemoglobin level c. Decrease in cardiac output d. Increase in stroke volume

ANS: C With a sustained heart rate greater than 160 beats/min, diastolic filling time decreases, decreasing stroke volume and cardiac output. The hemoglobin level would not be affected.

The patient presents to the clinic with dysuria and hematuria. How does the nurse proceed to assess for kidney inflammation? a. Uses deep palpation posteriorly. b. Lightly palpates each abdominal quadrant. c. Percusses posteriorly the costovertebral angle at the scapular line. d. Inspects abdomen for abnormal movement or shadows using indirect lighting

ANS: C With the patient sitting or standing erect, use direct or indirect percussion to assess for kidney inflammation. With the ulnar surface of the partially closed fist, percuss posteriorly the costovertebral angle at the scapular line. If the kidneys are inflamed, the patient feels tenderness during percussion. Use a systematic palpation approach for each quadrant of the abdomen to assess for muscular resistance, distention, abdominal tenderness, and superficial organs or masses. Light palpation would not detect kidney tenderness because the kidneys sit deep within the abdominal cavity. Posteriorly, the lower ribs and heavy back muscles protect the kidneys, so they cannot be palpated. Kidney inflammation will not cause abdominal movement. However, to inspect the abdomen for abnormal movement or shadows, the nurse should stand on the patient's right side and inspect from above the abdomen using direct light over the abdomen.

10. During a routine pediatric history and physical, the parents report that their child was a very small, premature infant that had to stay in the neonatal intensive care unit longer than usual. They state that the infant was yellow when born and developed an infection that required "every antibiotic under the sun" to reach a cure. Which exam is a priority for the nurse to conduct on the child? a. Cardiac b. Respiratory c. Ophthalmic d. Hearing acuity

ANS: D Hearing is the priority. Risk factors for hearing problems include low birth weight, nonbacterial intrauterine infection, and excessively high bilirubin levels. Hearing loss due to ototoxicity (injury to auditory nerves) can result from high maintenance doses of antibiotics. Cardiac, respiratory, and eye examinations are important assessments but are not relevant to this child's condition.

13. The nurse is urgently called to the gymnasium regarding an injured student. The student is crying in severe pain with a malformed fractured lower leg. Which proper sequence will the nurse follow to perform the initial assessment? a. Light palpation, deep palpation, and inspection b. Inspection, light palpation, and deep palpation c. Auscultation and light palpation d. Inspection and light palpation

ANS: D Inspection is the use of vision and hearing to distinguish normal from abnormal findings. Light palpation determines areas of tenderness and skin temperature, moisture, and texture. Deep palpation is used to examine the condition of organs, such as those in the abdomen. Caution is the rule with deep palpation. Deep palpation is performed after light palpation; however, deep palpation is not performed on a fractured leg. Auscultation is used to evaluate sound and is not used to assess a fractured leg.

7. A school nurse recognizes a belt buckle-shaped ecchymosis on a 7-year-old student. When privately asked about how the injury occurred, the student described falling on the playground. Which action will the nurse take next? a. Talk to the principal about how to proceed. b. Disregard the finding based upon child's response. c. Interview the patient in the presence of the teacher. d. Contact social services and report suspected abuse.

ANS: D Most states mandate a report to a social service center if nurses suspect abuse or neglect. When abuse is suspected, the nurse interviews the patient in private, not with a teacher. Observe the behavior of the individual for any signs of frustration, explanations that do not fit his or her physical presentation, or signs of injury. The nurse knows how to proceed and does not need to talk to the principal about what to do. Disregarding the finding is not advised because victims often will not complain or report that they are in an abusive situation.

33. A patient in the emergency department is reporting left lower abdominal pain. Which proper order will the nurse follow to perform the comprehensive abdominal examination? a. Percussion, palpation, auscultation b. Percussion, auscultation, palpation c. Inspection, palpation, auscultation d. Inspection, auscultation, palpation

ANS: D The order of an abdominal examination differs slightly from that of other assessments. Begin with inspection and follow with auscultation. By using auscultation before palpation, the chance of altering the frequency and character of bowel sounds is lessened.

5. A head and neck physical examination is completed on a 50-year-old female patient. All physical findings are normal except for fine brittle hair. Which laboratory test will the nurse expect to be ordered, based upon the physical findings? a.Oxygen saturation b. Liver function test c. Carbon monoxide d. Thyroid-stimulating hormone test

ANS: D Thyroid disease can make hair thin and brittle. Liver function testing is indicated for a patient who has jaundice. Oxygen saturation will be used for cyanosis. Cherry-colored lips indicate carbon monoxide poisoning.

20. Upon assessment, the patient is breathing normally and has normal vesicular lung sounds. Which expected inspiratory-to-expiratory breath sounds will the nurse hear? a. The expiration phase is longer than the inspiration phase. b. The inspiratory phase lasts exactly as long as the expiratory phase. c. The expiration phase is 2 times longer than the inspiration phase. d. The inspiratory phase is 3 times longer than the expiratory phase.

ANS: D Vesicular breath sounds are normal breath sounds; the inspiratory phase is 3 times longer than the expiratory phase. Bronchovesicular breath sounds have an inspiratory phase equal to the expiratory phase. Bronchial breath sounds have an expiration phase longer than the inspiration phase at a 3:2 ratio.

A nurse assesses a client who is scheduled for a cardiac catheterization. Which assessment should the nurse complete prior to this procedure? a. Client's level of anxiety b. Ability to turn self in bed c. Cardiac rhythm and heart rate d. Allergies to iodine-based agents

ANS: D Before the procedure, the nurse should ascertain whether the client has an allergy to iodine-containing preparations, such as seafood or local anesthetics. The contrast medium used during the procedure is iodine based. This allergy can cause a life-threatening reaction, so it is a high priority. Second, it is important for the nurse to assess anxiety, mobility, and baseline cardiac status.

A nurse is examining a client reporting right upper quadrant (RUQ) abdominal pain. What technique should the nurse use to assess this client's abdomen? a. Auscultate after palpating. b. Avoid any palpation. c. Palpate the RUQ first. d. Palpate the RUQ last.

ANS: D If pain is present in a certain area of the abdomen, that area should be palpated last to keep the client from tensing up, which could possibly affect the rest of the examination. Auscultation of the abdomen occurs prior to palpation.

A client presents to the family practice clinic reporting a week of watery, somewhat bloody diarrhea. The nurse assists the client to obtain a stool sample. What action by the nurse is most important? a. Ask the client about recent exposure to illness. b. Assess the client's stool for obvious food particles. c. Include the date and time on the specimen container. d. Put on gloves prior to collecting the sample.

ANS: D To avoid possible exposure to infectious agents, the nurse dons gloves prior to handling any bodily secretions. Recent exposure to illness is not related to collecting a stool sample. The nurse can visually inspect the stool for food particles, but it still needs analysis in the laboratory. The container should be dated and timed, but safety for the staff and other clients comes first.

A chemotherapy patient has gained 5 pounds in 2 days. Which assessment question by the nurse is mostappropriate? a. "Are you following any weight loss program?" b. "How many calories a day do you consume?" c. "Do you have dry mouth or feel thirsty?" d. "How many times a day do you urinate?"

ANS: D A rapid gain in weight usually indicates extracellular volume (ECV) excess if the person began with normal ECV. Asking the patient about urination habits will help determine whether the body is trying to excrete the excess fluid or if renal dysfunction is contributing to ECV excess. This is too rapid a weight gain to be dietary; it is fluid retention. Asking about following a weight loss program will not help determine the cause of the problem. Caloric intake does not account for rapid weight changes. Dry mouth and thirst accompany ECV deficit, which would be associated with rapid weight loss.

A nurse is assisting the health care provider in inserting a central line. Which action indicates the nurse is following the recommended bundle protocol to reduce central line-associated bloodstream infections (CLABSI)? a. Preps skin with povidone-iodine solution. b. Suggests the femoral vein for insertion site. c. Applies double gloving without hand hygiene. d. Uses chlorhexidine skin antisepsis prior to insertion.

ANS: D A recommended bundle at insertion of a central line is hand hygiene prior to catheter insertion; use of maximum sterile barrier precautions upon insertion; chlorhexidine skin antisepsis prior to insertion and during dressing changes; avoidance of the femoral vein for central venous access for adults; and daily evaluation of line necessity, with prompt removal of non-essential lines. Povidone-iodine is not recommended

A health care provider prescribes aspirin 650 mg every 4 hours PO when febrile. For which patient will this order be appropriate? a. 7 year old with a bleeding disorder b. 21 year old with a sprained ankle c. 35 year old with a severe headache from hypertension d. 62 year old with a high fever from an infection

ANS: D Aspirin is an analgesic, an antipyretic, and an anti-inflammatory medication. The provider wrote the medication to be given for a fever (febrile). Fevers are common in infections. If a child is bleeding, aspirin would be contraindicated; aspirin increases the likelihood of bleeding. Although it can be used for inflammatory problems (sprained ankle) and pain/analgesia (severe headache), this is not how the order was written

Having misplaced a stethoscope, a nurse borrows a colleague's stethoscope. The nurse next enters the patient's room and identifies self, washes hands with soap, and states the purpose of the visit. The nurse performs proper identification of the patient before auscultating the patient's lungs. Which critical health assessment step should the nurse have performed? a. Running warm water over stethoscope b. Draping stethoscope around the neck c. Rubbing stethoscope with betadine d. Cleaning stethoscope with alcohol

ANS: D Bacteria and viruses can be transferred from patient to patient when a stethoscope that is not clean is used. The stethoscope should be cleaned before use on each patient with isopropyl alcohol. Running water over the stethoscope does not kill bacteria. Betadine is an inappropriate cleaning solution and may damage the equipment. Draping the stethoscope around the neck is not advised.

The nurse is caring for a patient with hyperkalemia. Which body system assessment is the priority? a. Gastrointestinal b. Neurological c. Respiratory d. Cardiac

ANS: D Cardiac is the priority. Hyperkalemia places the patient at risk for potentially serious dysrhythmias and cardiac arrest. Potassium balance is necessary for cardiac function. Respiratory is the priority with hypokalemia. Monitoring of gastrointestinal and neurological systems would be indicated for other electrolyte imbalances.

The nurse is using a closed suction device. Which patient will be most appropriate for this suctioning method? a. A 5-year-old with excessive drooling from epiglottitis b. A 5-year-old with an asthma attack following severe allergies c. A 24-year-old with a right pneumothorax following a motor vehicle accident d. A 24-year-old with acute respiratory distress syndrome requiring mechanical ventilation

ANS: D Closed suctioning is most often used on patients who require invasive mechanical ventilation to support their respiratory efforts because it permits continuous delivery of oxygen while suction is performed and reduces the risk of oxygen desaturation. In this case, the acute respiratory distress syndrome requires mechanical ventilation. In the presence of epiglottitis, croup, laryngospasm, or irritable airway, the entrance of a suction catheter via the nasal route causes intractable coughing, hypoxemia, and severe bronchospasm, necessitating emergency intubation or tracheostomy. The 5year-old child with asthma would benefit from an inhaler. A chest tube is needed for the pneumothorax.

The nurse is caring for a patient with respiratory problems. Which assessment finding indicates a late sign of hypoxia? a. Elevated blood pressure b. Increased pulse rate c. Restlessness d. Cyanosis

ANS: D Cyanosis, blue discoloration of the skin and mucous membranes caused by the presence of desaturated hemoglobin in capillaries, is a late sign of hypoxia. Elevated blood pressure, increased pulse rate, and restlessness are early signs of hypoxia.

The nurse is teaching about the process of exchanging gases through the alveolar capillary membrane. Which term will the nurse use to describe this process? a. Ventilation b. Surfactant c. Perfusion d. Diffusion

ANS: D Diffusion is the process of gases exchanging across the alveoli and capillaries of body tissues. Ventilation is the process of moving gases into and out of the lungs. Surfactant is a chemical produced in the lungs to maintain the surface tension of the alveoli and keep them from collapsing. Perfusion is the ability of the cardiovascular system to carry oxygenated blood to tissues and return deoxygenated blood to the heart

During a routine pediatric history and physical, the parents report that their child was a very small, premature infant that had to stay in the neonatal intensive care unit longer than usual. They state that the infant was yellow when born and developed an infection that required "every antibiotic under the sun" to reach a cure. Which exam is apriority for the nurse to conduct on the child? a. Cardiac b. Respiratory c. Ophthalmic d. Hearing acuity

ANS: D Hearing is the priority. Risk factors for hearing problems include low birth weight, nonbacterial intrauterine infection, and excessively high bilirubin levels. Hearing loss due to ototoxicity (injury to auditory nerves) can result from high maintenance doses of antibiotics. Cardiac, respiratory, and eye examinations are important assessments but are not relevant to this child's condition

The nurse is assessing a patient with emphysema. Which assessment finding requires further follow-up with the health care provider? a. Increased anterior-posterior diameter of the chest b. Accessory muscle used for breathing c. Clubbing of the fingers d. Hemoptysis

ANS: D Hemoptysis is an abnormal occurrence of emphysema, and further diagnostic studies are needed to determine the cause of blood in the sputum. Clubbing of the fingers, barrel chest (increased anteriorposterior chest diameter), and accessory muscle use are all normal findings in a patient with emphysema

The nurse is urgently called to the gymnasium regarding an injured student. The student is crying in severe pain with a malformed fractured lower leg. Which proper sequence will the nurse follow to perform the initial assessment? a. Light palpation, deep palpation, and inspection b. Inspection, light palpation, and deep palpation c. Auscultation and light palpation d. Inspection and light palpation

ANS: D Inspection is the use of vision and hearing to distinguish normal from abnormal findings. Light palpation determines areas of tenderness and skin temperature, moisture, and texture. Deep palpation is used to examine the condition of organs, such as those in the abdomen. Caution is the rule with deep palpation. Deep palpation is performed after light palpation; however, deep palpation is not performed on a fractured leg. Auscultation is used to evaluate sound and is not used to assess a fractured leg

A school nurse recognizes a belt buckle-shaped ecchymosis on a 7-year-old student. When privately asked about how the injury occurred, the student described falling on the playground. Which action will the nurse take next? a. Talk to the principal about how to proceed. b. Disregard the finding based upon child's response. c. Interview the patient in the presence of the teacher. d. Contact social services and report suspected abuse

ANS: D Most states mandate a report to a social service center if nurses suspect abuse or neglect. When abuse is suspected, the nurse interviews the patient in private, not with a teacher. Observe the behavior of the individual for any signs of frustration, explanations that do not fit his or her physical presentation, or signs of injury. The nurse knows how to proceed and does not need to talk to the principal about what to do. Disregarding the finding is not advised because victims often will not complain or report that they are in an abusive situation.

A patient has inadequate stroke volume related to decreased preload. Which treatment does the nurse prepare to administer? a. Diuretics b. Vasodilators c. Chest physiotherapy d. Intravenous (IV) fluids

ANS: D Preload is affected by the circulating volume; if the patient has decreased fluid volume, it will need to be replaced with fluid or blood therapy. Preload is the amount of blood in the left ventricle at the end of diastole, often referred to as end-diastolic volume. Giving diuretics and vasodilators will make the situation worse. Diuretics causes fluid loss; the patient is already low on fluids or the preload would not be decreased. Vasodilators reduced blood return to the heart, making the situation worse; the patient does not have enough blood and fluid to the heart or the preload would not be decreased. Chest physiotherapy is a group of therapies for mobilizing pulmonary secretions. Chest physiotherapy will not help this cardiovascular problem

A nurse is preparing to suction a patient. The pulse is 65 and pulse oximetry is 94%. Which finding will cause the nurse to stop suctioning? a. Pulse 75 b. Pulse 80 c. Oxygen saturation 91% d. Oxygen saturation 88%

ANS: D Stop when oxygen saturation is 88%. Monitor patient's vital signs and oxygen saturation during procedure; note whether there is a change of 20 beats/min (either increase or decrease) or if pulse oximetry falls below 90% or 5% from baseline. If this occurs, stop suctioning. A pulse rate of 75 is only 10 beats different from the start of the procedure. A pulse rate of 80 is 15 beats different from the start of suctioning. Oxygen saturation of 91% is not 5% from baseline or below 90%

Which patient does the nurse most closely monitor for an unintended synergistic effect? a. The 4 year old who has mistakenly taken a half bottle of vitamins b. The 35 year old who has ingested meth mixed with several household chemicals c. The 50 year old who is prescribed a second blood pressure medication d. The 72 year old who is seeing four different specialists

ANS: D The 72 year old seeing four different providers is likely to experience polypharmacy. Polypharmacy places the patient at risk for unintended mixing of medications that potentiate each other. When two medications have a synergistic effect, their combined effect is greater than the effect of the medications when given separately. The child taking too much of a medication by mistake could experience overdose or toxicity. The 50 year old is prescribed two different blood pressure medications for their synergistic effect, but this is a desired, intended event. A patient taking meth and mixing chemicals can be toxic

A nurse is caring for a patient who is receiving peripheral intravenous (IV) therapy. When the nurse is flushing the patient's peripheral IV, the patient reports pain. Upon assessment, the nurse notices a red streak that is warm to the touch. What is the nurse's initial action? a. Record a phlebitis grade of 4. b. Assign an infiltration grade. c. Apply moist compress. d. Discontinue the IV.

ANS: D The IV site has phlebitis. The nurse should discontinue the IV. The phlebitis score is 3. The site has phlebitis, not infiltration. A moist compress may be needed after the IV is discontinued.

The prescriber wrote for a 40-kg child to receive 25 mg of medication 4 times a day. The therapeutic range is 5 to 10 mg/kg/day. What is the nurse's priority? a. Change the dose to one that is within range. b. Administer the medication because it is within the therapeutic range. c. Notify the health care provider that the prescribed dose is in the toxic range. d. Notify the health care provider that the prescribed dose is below the therapeutic range.

ANS: D The dosage range is 200 to 400 mg a day (5 × 40 = 200 and 10 × 40 = 400). The prescribed dose is 100 mg/day (4 ×25 = 100), which is below therapeutic range. The nurse should notify the health care provider first and ask for clarification on the order. The dose is not above the therapeutic range and is not at a toxic level. The nurse should never alter an order without the prescriber's approval and consent.

A patient refuses medication. Which is the nurse's first action? a. Educate the patient about the importance of the medication. b. Discreetly hide the medication in the patient's favorite gelatin. c. Agree with the patient's decision and document it in the chart. d. Explore with the patient reasons for not wanting to take the medication

ANS: D The first response is to explore reasons the patient does not want the medication. After the assessment, the nurse can decide what to do next. Educating is important, but it is not the first action. Ultimately, the patient does have the right to refuse the medication; however, the nurse should first try to find out reasons for the refusal and provide education if needed based upon the assessment findings. Hiding medication and deceiving a patient into taking a medication is unethical and violates right to autonomy.

A patient has an acute intravascular hemolytic reaction to a blood transfusion. After discontinuing the blood transfusion, which is the nurse's next action? a. Discontinue the IV catheter. b. Return the blood to the blood bank. c. Run normal saline through the existing tubing. d. Start normal saline at TKO rate using new tubing.

ANS: D The nurse should first attach new tubing and begin running in normal saline at a rate to keep the vein open, in case any medications need to be delivered through an IV site. The existing tubing should not be used because that would infuse the blood in the tubing into the patient. It is necessary to preserve the IV catheter in place for IV access to treat the patient. After the patient has been assessed and stabilized, the blood can be returned to the blood bank

The supervising nurse is watching nurses prepare medications. Which action by one of the nurses will the supervising nurse stop immediately? a. Rolls insulin vial between hands b. Administers a dose of correction insulin c. Draws up glargine (Lantus) in a syringe by itself d. Prepares NPH insulin to be given intravenously (IV)

ANS: D The only insulin that can be given IV is regular. NPH cannot be given IV and must be stopped. All the rest demonstrate correct practice. Insulin is supposed to be rolled, not shaken. Glargine is supposed to be given by itself; it cannot be mixed with another medication. Correction insulin, also known as sliding-scale insulin, provides a dose of insulin based on the patient's blood glucose level. The term correction insulin is preferred because it indicates that small doses of rapid- or short-acting insulins are needed to correct a patient's elevated blood sugar

A patient in the emergency department is reporting left lower abdominal pain. Which proper order will the nurse follow to perform the comprehensive abdominal examination? a. Percussion, palpation, auscultation b. Percussion, auscultation, palpation c. Inspection, palpation, auscultation d. Inspection, auscultation, palpation

ANS: D The order of an abdominal examination differs slightly from that of other assessments. Begin with inspection and follow with auscultation. By using auscultation before palpation, the chance of altering the frequency and character of bowel sounds is lessened

.A nurse is preparing to start peripheral intravenous (IV) therapy. In which order will the nurse perform the steps starting with the first one? 1. Clean site. 2. Select vein. 3. Apply tourniquet. 4. Release tourniquet. 5. Reapply tourniquet. 6. Advance and secure. 7. Insert vascular access device. a. 1, 3, 2, 7, 5, 4, 6 b. 1, 3, 2, 5, 7, 6, 4 c. 3, 2, 1, 5, 7, 6, 4 d. 3, 2, 4, 1, 5, 7, 6

ANS: D The steps for inserting an intravenous catheter are as follows: Apply tourniquet; select vein; release tourniquet; clean site; reapply tourniquet; insert vascular access device; and advance and secure

Which assessment finding will the nurse expect for a patient with the following laboratory values: sodium 145 mEq/L, potassium 4.5 mEq/L, calcium 4.5 mg/dL? a. Weak quadriceps muscles b. Decreased deep tendon reflexes c. Light-headedness when standing up d. Tingling of extremities with possible tetany

ANS: D This patient has hypocalcemia because the normal calcium range is 8.4 to 10.5 mg/dL. Hypocalcemia causes muscle tetany, positive Chvostek's sign, and tingling of the extremities. Sodium and potassium values are within their normal ranges: sodium 135 to 145 mEq/L; potassium 3.5 to 5.0 mEq/L. Light-headedness when standing up is a manifestation of ECV deficit or sometimes hypokalemia. Weak quadriceps muscles are associated with potassium imbalances. Decreased deep tendon reflexes are related to hypercalcemia or hypermagnesemia.

A head and neck physical examination is completed on a 50-year-old female patient. All physical findings are normal except for fine brittle hair. Which laboratory test will the nurse expect to be ordered, based upon the physical findings? a. Oxygen saturation b. Liver function test c. Carbon monoxide d. Thyroid-stimulating hormone test

ANS: D Thyroid disease can make hair thin and brittle. Liver function testing is indicated for a patient who has jaundice. Oxygen saturation will be used for cyanosis. Cherry-colored lips indicate carbon monoxide poisoning.

A registered nurse interprets that a scribbled medication order reads 25 mg. The nurse administers 25 mg of the medication to a patient and then discovers that the dose was incorrectly interpreted and should have been 15 mg. Who is ultimately responsible for the error? a. Health care provider b. Pharmacist c. Hospital d. Nurse

ANS: D Ultimately, the person administering the medication is responsible for ensuring that it is correct. The nurse administered the medication, so in this case it is the nurse. Accept full accountability and responsibility for all actions surrounding the administration of medications. Do not assume that a medication that is ordered for a patient is the correct medication or the correct dose. This is the importance of verifying the six rights of medication administration. The ultimate responsibility and accountability are with the nurse, not the health care provider, pharmacist, or hospital

Upon assessment, the patient is breathing normally and has normal vesicular lung sounds. Which expected inspiratory-to-expiratory breath sounds will the nurse hear? a. The expiration phase is longer than the inspiration phase. b. The inspiratory phase lasts exactly as long as the expiratory phase. c. The expiration phase is 2 times longer than the inspiration phase. d. The inspiratory phase is 3 times longer than the expiratory phase.

ANS: D Vesicular breath sounds are normal breath sounds; the inspiratory phase is 3 times longer than the expiratory phase. Bronchovesicular breath sounds have an inspiratory phase equal to the expiratory phase. Bronchial breath sounds have an expiration phase longer than the inspiration phase at a 3:2 ratio

A nurse is preparing an intravenous IV piggyback infusion. In which order will the nurse perform the steps, starting with the first one? 1. Compare the label of the medication with the medication administration record at the patient's bedside. 2. Connect the tubing of the piggyback infusion to the appropriate connector on the upper Y-port. 3. Hang the piggyback medication bag above the level of the primary fluid bag. 4. Clean the main IV line port with an antiseptic swab. 5. Connect the infusion tubing to the medication bag. 6. Regulate flow. a. 5, 2, 1, 4, 3, 6 b. 5, 2, 1, 3, 4, 6 c. 1, 5, 4, 3, 2, 6 d. 1, 5, 3, 4, 2, 6

ANS: D When preparing and administering IV piggybacks, use the following steps: Compare the label of medication with the medication administration record at the patient's bedside; connect the infusion tubing to the medication bag; hang the piggyback medication bag above the level of the primary fluid bag; clean the main IV line port with an antiseptic swab; connect the tubing of the piggyback infusion to the appropriate connector on the upper Y-port; and regulate flow

Which of the following should a nurse assess before administering medications through a nasogastric tube?

Amount of residual volume left in stomach -checking residual volume prevents putting medications into an already full stomach

An 80-year-old woman with a history of diabetes and arthritis has made an appointment with her health care provider for complaints of urinary incontinence (UI). The patient states that she has recently become incontinent of urine and thinks it is because of her age. What is the best response from the nurse?

Are you having issues walking to your bathroom?

A 36-year-old patient diagnosed with meningitis has a fever of 102.3° F. A family member verbalizes a concern that the patient is "breathing fast." Upon assessment, the nurse notes a respiratory rate of 20 breaths/min, pulse oximetry is 92% on 2L/O2, and lungs clear to auscultation. What is the best explanation for the rapid respiratory rate that the nurse can give the family member? a. "He is most likely anxious because he is in the hospital." b. "His fever has increased his metabolic rate and is causing him to breathe faster." c. "He is hyperventilating because he needs more oxygen." d. "He has an acid-base imbalance, which is causing him to hyperventilate."

B

A college student who smokes asks a healthcare professional if there really is a connection between smoking and lung cancer. What is the healthcare worker's best response? a. The risk for lung cancer for smokers than for nonsmokers is 5 times greater. b. The risk for lung cancer for smokers than for nonsmokers is 10 times greater. c. The risk for lung cancer for smokers than for nonsmokers is 50 times greater. d. Lung cancer affects smokers and non-smokers equally due to occupational hazards.

B

A healthy 33-year-old mother of three children reports "having no energy." She asks the nurse, how she could increase her energy level. What is the best response for the nurse to give? a. "You should decrease iron intake." b. "Daily exercise has been shown to increase a person's energy level." c. "Energy caffeinated drinks are a good substitute for exercise." d. "Try to exercise 90 minutes every other day."

B

A nurse is being oriented to work for an intensive care unit. The hemodynamic data indicate that the patient has a decreased preload. Which information indicates the nurse has a correct understanding of the concept of preload? a. It is the amount of blood ejected from the left ventricle each minute. b. It is the amount of blood in the heart at the end of ventricular diastole. c. It is the resistance to the ejection of blood from the left ventricle. d. It is the rhythmic relaxation and contraction of the heart chambers.

B

A patient presents with an acute myocardial infarction that resulted in right ventricular damage. The nurse needs to assess the patient for right-sided heart failure, which includes which of the following? a. Crackles on auscultation b. Jugular neck vein distention c. Increased myocardial perfusion d. Orthopnea

B

A student nurse caring for a patient with a chest tube has been asked what equipment should be at the bedside to assess for an air leak. Which information indicates the student nurse has a correct understanding of the equipment needed? a. Suction equipment wrapped in plastic to keep instrument clean b. Hemostat covered with gauze to prevent penetration of the chest tube c. Cup of water to place the end of the chest tube d. Petroleum gauze to use as a dressing

B

The nurse is evaluating a patient who has a chest tube. To properly maintain chest tube function, what is the nurse's best action? a. Strip the tube every hour to maintain drainage. b. Place the device below the patient's chest. c. Double clamp the tubes except during assessments. d. Remove the tubing from the drainage device to check for proper suctioning.

B

You are caring for a patient who has diabetes complicated by kidney disease. You need to make a detailed assessment when administering medications because this patient may experience problems with: A. Absorption. B. Biotransformation. C. Distribution. D. Excretion.

D. Excretion

A patient is admitted to the emergency department with suspected carbon monoxide poisoning. Even though her color is ruddy not cyanotic, the nurse understands the patient is at a risk for decreased oxygen-carrying capacity of blood because carbon Monoxide does which of the following: A) Stimulates hyperventilation causing respiratory alkalosis B) Forms a strong bond with hemoglobin thus preventing oxygen binding in the lungs C) Stimulates hypoventilation causing respiratory acidosis D) Causes alveoli to overinflate leading to atelectasis

B (Carbon monoxide strongly binds to hemoglobin making it unavailable for oxygen binding and transport.)

The nurse is caring for a patient who has decreased mobility. Which intervention is a simple and cost-effective method for reducing the risks of pulmonary complication? Antibiotics Frequent change of position Oxygen humidification Chest physiotherapy

B (Movement not only mobilizes secretions but helps strengthen respiratory muscles by impacting the effectiveness of gas exchange processes.)

Which of the following skills can be delegated to nursing assistive personnel (NAP)? A) Nasotracheal suctioning B) Oropharyngeal suctioning of a stable patient C)Suctioning a new artificial airway D)Permanent tracheostomy tube suctioning E)Care of an endotracheal tube (ETT)

B D (Oropharyngeal suctioning of a stable patient and permanent tracheostomy tube suctioning may be safely delegated to a NAP. The other skills require nursing assessment and clinical decision making as the skill progresses.)

A 59-year-old man has presented to the emergency department with chest pain. Which of the following components of his subsequent blood work is most clearly indicative of a myocardial infarction (MI)? A) CK-MB B) Troponin C) Myoglobin D) C-reactive protein

B) Troponin Troponin is the biomarker of choice in the diagnosis of MI, with sensitivity and specificity that exceed those of CK-MB and myoglobin. CRP levels are not used to diagnose acute MI.

Auscultation of a patient's heart reveals the presence of a murmur. This assessment finding is a result of A) Increased viscosity of the patient's blood. B) Turbulent blood flow across a heart valve. C) Friction between the heart and the myocardium. D) A deficit in heart conductivity that impairs normal contractility.

B) Turbulent blood flow across a heart valve. Turbulent blood flow across the affected valve results in a murmur. A murmur is not a direct result of variances in blood viscosity, conductivity, or friction between the heart and myocardium.

Nurses are legally required to document medications that are administered to patients. The nurse is mandated to document which of the following? A. Medication before administering it B. Medication after administering it C. Rationale for administering it D. Prescriber rationale for prescribing it

B. Medication after administering it

After seeing a patient, the physician gives a nursing student a verbal order for a new medication. The nursing student first needs to: A. Follow ISMP guidelines for safe medication abbreviations. B. Explain to the physician that the order needs to be given to a registered nurse. C. Write down the order on the patient's order sheet and read it back to the physician. D. Ensure that the six rights of medication administration are followed when giving the medication.

B. Rationale: Nursing students cannot take orders.

A nursing student takes a patient's antibiotic to his room. The patient asks the nursing student what it is and why he should take it. Which information does the nursing student include when replying to the patient? A. Only the patient's physician can give this information. B. The student provides the name of the medication and a description of its desired effect. C. Information about medications is confidential and cannot be shared. D. He has to speak with his assigned nurse about this.

B. Rationale: Patients need to know information about their medications so they can take them correctly and safely.

If a patient who is receiving intravenous (IV) fluids develops tenderness, warmth, erythema, and pain at the site, the nurse suspects: A. Sepsis. B. Phlebitis. C. Infiltration. D. Fluid overload.

B. Rationale: Redness, warmth, and tenderness at the IV site are signs of phlebitis.

The nurse receives an order to start giving a loop diuretic to a patient to help lower his or her blood pressure. The nurse determines the appropriate route for administering the diuretic according to: A. Hospital policy. B. The prescriber's orders. C. The type of medication ordered. D. The patient's size and muscle mass.

B. Rationale:The order from the prescriber needs to indicate the route of administration.

While performing a head to toe assessment, you perform the Romberg test. You do this to test the patients

Balance The most common test of balance is the Romberg test. Ask the patient to stand about 2 feet in front of you, with her feet together, toes pointed forward, and her hands at her sides. While you extend you hands to that one is on either side of the patient, ask her to close her eyes. Watch to see how well she can maintain balance in that position. A minimum of swaying is normal, but if the patient sways more than a couple of inches, stop the test and document that the patient demonstrated difficulty maintaining balance on Romberg testing.

A client is admitted to the emergency department with a suspected cervical spine fracture at the C3 level. The nurse is most concerned about the client's ability to:

Breathe

A 3-month-old infant is being seen for a well-child check at the pediatric clinic. The nurse is assessing the guardian's knowledge level about the infant's growth and development. One of the topics that the nurse has chosen to address in this session is the risk for airway obstruction. What is the main reason that the nurse has chosen this topic? a. Infants can have severe allergic reactions to food based on exposure to secondhand smoke. b. Infants are prone to lower airway infections that can become obstructive. c. Infants have a tendency to place foreign objects in their mouths. d. Infants can have airway obstruction from excessive drooling associated with teething.

C

A 45-year-old patient was diagnosed with an anterior myocardial infarction. The patient asks the nurse why his chest hurt when he had his heart attack. What is the best response from the nurse? a. "One of your heart valves wasn't working properly and caused an obstructed blood flow." b. "One of your coronary arteries had a spasm, and your heart muscle wasn't able to get enough blood." c. "Your heart muscle was deprived of oxygen, which caused chest pain." d. "The heart muscle is sensitive to changes in electrical conduction."

C

A 58-year-old Caucasian woman is at the clinic for her annual check-up. She tells the nurse that she noticed her blood pressure is higher than it was when she was younger. She asks the nurse why this would happen. What is the nurse's best response? a. "Your race and gender are predisposing factors for heart disease." b. "Well, if you stop smoking your blood pressure would go down." c. "As we age, our blood vessels become less elastic, which causes higher blood pressure." d. "I don't think it's anything to worry about."

C

A health care worker received an annual tuberculosis test administered 56 hours ago. The injection site is very red and flat. The certified nurse who is reading the test should take which action? a. Advise the health care worker another test must be done because the test was not read within the proper time. b. Tell the health care worker the results are positive and cannot return to work. c. Document the results as a negative reaction. d. Measure the area in millimeters.

C

A patient on the surgical unit is 1 day postoperative for surgery to remove stomach cancer. In addition to the physiological stress this patient has undergone, the nurse recognizes that this patient will have to deal with the psychological stress of finding out that the cancer has metastasized to the liver. Which physiological change would the nurse expect to see as a response to stress? a. Decreased heart rate b. Increased hemoptysis output c. Increased respiratory rate d. Decreased cardiac output

C

A patient who is 7 months pregnant with her first child is visiting the health care provider for her scheduled prenatal checkup. She tells the nurse that she is short of breath and fatigued. What is the best response from the nurse? a. "You should have let us know immediately instead of waiting until your appointment." b. "I'll make a note of it on your chart." c. "That is normal; your uterus is causing pressure on your diaphragm, making it more difficult to breathe." d. "Oxygen is needed for you and the baby. I will give you some oxygen to help you."

C

A patient with pulmonary congestion needs to cough to clear secretions. The nurse instructs the patient to inhale and perform a series of coughs during exhalation. What type of cough did the nurse teach the patient? a. Quad b. Huff c. Cascade d. Splinting

C

The RN and nursing assistive personnel (NAP) are caring for six patients on the pulmonary unit. Which of the following tasks would be most appropriate for the nurse to delegate to the NAP? a. Taking vital signs on a 56-year-old man with severe dyspnea b. Suctioning a patient with hemoptysis c. Encouraging a postoperative patient to use the incentive spirometer

C

The family member of a 73-year-old patient with chronic obstructive pulmonary disease (COPD) is concerned about the patient's recent weight loss. When questioned by the nurse, the patient denies any change in diet or activity, but admits to losing 10 lb in the past 6 weeks. What is the nurse's best response to the family member's concern? a. "Maybe the patient has a higher metabolic rate than you." b. "It doesn't seem fair that some people can lose weight so easily." c. "This disease affects the respiratory system and causes the body to burn more calories to supply the energy to breathe." d. "You need to discuss this with the health care provider so testing can be ordered for tuberculosis."

C

The nursing student is formulating a nursing care plan for a patient with pneumonia. The care plan is directed toward meeting the potential oxygenation needs of the patient. Which of the following examples would be the best way for the nursing student to write an expected outcome for the care plan? a. "The patient will have less pain." b. "The patient will be able to breathe better." c. "The patient's pulse oximetry reading will remain greater than 92%." d. "The patient's interactions will be normal."

C

While assessing the cardiovascular status of a patient, the nurse performs auscultation. Which of the following practices should the nurse implement into the assessment during auscultation? A) Position the patient supine. B) Ask the patient to hold his or her breath. C) Palpate the radial pulse while auscultating the apical pulse. D) Use the bell of the stethoscope when auscultating S1 and S2.

C) Palpate the radial pulse while auscultating the apical pulse. In order to detect a pulse deficit, simultaneously palpate the radial pulse when auscultating the apical area. The diaphragm is more appropriate than the bell when auscultating S1 and S2. A sitting or side-lying position is most appropriate for cardiac auscultation. It is not necessary to ask the patient to hold his or her breath during cardiac auscultation.

A patient is transitioning from the hospital to the home environment. A home care referral is obtained. What is a priority in relation to safe medication administration for the discharge nurse? A. Set up the follow-up appointments with the physician for the patient. B. Ensure that someone will provide housekeeping for the patient at home. C. Ensure that the home care agency is aware of medication and health teaching needs. D. Make sure that the patient's family knows how to safely bathe him or her and provide mouth care.

C. Rationale: A nursing responsibility is to collaborate with community resources when patients have home care needs or difficulty understanding their medications.

A nurse is administering medications to a 4-year-old patient. After he or she explains which medications are being given, the mother states, "I don't remember my child having that medication before." What is the nurse's next action? A. Give the medications B. Identify the patient using two patient identifiers C. Withhold the medications and verify the medication orders D. Provide medication education to the mother to help her better understand her child's medications

C. Rationale: Do not ignore patient or caregiver concerns; always verify orders whenever a medication is questioned before administering it.

A patient is receiving an intravenous (IV) push medication. If the drug infiltrates into the outer tissues, the nurse: A. Continues to let the IV run. B. Applies a warm compress to the infiltrated site. C. Stops the administration of the medication and follows agency policy. D. Should not worry about this because vesicant filtration is not a problem.

C. Rationale: When an IV medication infiltrates, stop giving the medication and follow agency policy.

The nurse is caring for a client who has undergone cardiac catheterization. The client says to the nurse, "The doctor said my cardiac output was 5.5 L/min. What is normal cardiac output?" Which of the following is the nurse's best response?

CORRECT ANSWER IS C A) "It is best to ask your doctor." B) "Did the test make you feel upset?" C) "The normal cardiac output for an adult is 4 to 6 L/min." D) "Are you able to explain why are you asking this question?" The client asked a direct question that the nurse should be able to answer. Normal cardiac output for an adult is 4 to 6 L/min. Questions regarding diagnosis and prognosis may be referred to physicians. There is no harm in answering this question. When using therapeutic communication, the nurse should never ask a client to justify his or her feelings by inquiring why a question was asked. There is no evidence that this client is upset

Symptoms associated with anemia include which of the following? (Select all that apply.)

CORRECT ANSWERS ARE A & D A) Increased breathlessness B) Decreased breathlessness C) Increased activity tolerance D) Decreased activity tolerance Clients with anemia have fatigue, decreased activity tolerance, and increased breathlessness, as well as pallor (especially seen in the conjunctiva of the eye) and an increased heart rate.

Which of the following assessment data indicate that the client's airway needs suctioning? (Select all that apply.)

CORRECT ANSWERS ARE A, C & E Suctioning is necessary when the client is unable to clear respiratory secretions from the airways. Signs that a client's airway needs suctioning include a change in respiratory rate or adventitious sounds, nasal secretions, gurgling, drooling, restlessness, gastric secretions or vomitus in the mouth, and coughing without clearance of secretions from the airway.

A client with chronic obstructive pulmonary disease (COPD) is experiencing dyspnea and anxiety. The nurse helps the client to breathe better by doing which of the following? (Select all that apply.)

CORRECT ANSWERS ARE B & C A) Implementing guided imagery B) Instructing the client to perform pursed-lip breathing C) Elevating the head of the bed to semi-Fowler's or Fowler's position Elevating the head of the bed to Fowler's position (45-degree angle) or semi-Fowler's position (30- to 45-degree angle) causes the diaphragm to lower from gravity and thus increases the space for lung expansion. Pursed-lip breathing prolongs exhalation and maintains the alveoli open longer, thus extending the period of oxygen and carbon dioxide exchange. Too high an elevation of the head of the bed could force the diaphragm into the thorax and reduce lung expansion. Fluids could help liquify the pulmonary secretions in the future, but right now the client needs more acute care. Guided imagery may help in the future, but now is not the time to implement this intervention.

The nurse suspects left-sided heart failure in a newly admitted client when the nurse notes which of the following symptoms? (Select all that apply.)

CORRECT ANSWERS ARE B & D A) Distended neck veins B) Bilateral crackles in the lungs C) Weight gain of 2 lb in past 2 days D) Shortness of breath, especially at night Left-sided heart failure results in ineffective ejection of blood from the left ventricle. This causes a backup of blood into the lungs. Thus, symptoms of left-sided heart failure are usually related to the lungs.

The nurse on a rehabilitation unit is caring for a 77-year-old patient who had undergone total knee replacement surgery. Since surgery, the patient has had several instances of urinary incontinence. The health care provider is contemplating the order of a Foley catheter. What is the nurses best response to this suggestion?

Can we try the toilet schedule before?

Fever increases the tissues' need for oxygen, and as a result:

Carbon dioxide production increases Fever increases the tissues' need for oxygen, and as a result, carbon dioxide production increases. When fever persists, the metabolic rate remains high and the body begins to break down protein stores, which results in muscle wasting and decreased muscle mass.

A client asks why smoking is a major risk factor for heart disease. In formulating a response, the nurse incorporates the understanding that nicotine

Causes vasoconstriction

The patient has an order for 2 tablespoons of Milk of Magnesia. How much medication does the nurse give him or her? A. 2 mL B. 5 mL C. 16 mL D. 30 mL

D. Rationale: 1 tablespoon = 15 mL; 2 tablespoons = 30 mL.

A male patient has been admitted with a fever and malaise. The health care provider has ordered a clean catch midstream specimen for urinalysis on this patient. To collect the urine specimen, the nurse should instruct the patient to do which of the following?

Cleanse his penis, begin his stream, and then void into a sterile cup.

The nurse is caring for a patient from a long-term care facility who has a Foley catheter. The urine in the bag is dark yellow and has a cloudy appearance and a strong odor. According to the transfer sheet, the Foley was placed 5 weeks before the hospital admission. What should the nurse do?

Contact the health care provider for an order to change the catheter and provide an update on the appearance of the urine.

Which nursing diagnosis is most appropriate for an infant with acute bronchiolitis due to respiratory syncytial virus (RSV)? 1. Activity Intolerance 2. Decreased Cardiac Output 3. Pain, Acute 4. Tissue Perfusion, Ineffective (peripheral)

Correct Answer: 1 Rationale 1: Activity intolerance is a problem because of the imbalance between oxygen supply and demand. Cardiac output is not compromised during an acute phase of bronchiolitis. Pain is not usually associated with acute bronchiolitis. Tissue perfusion (peripheral) is not affected by this respiratory-disease process.

The nurse admits a child with a ventricular septal defect (VSD) to the unit. Which nursing diagnosis for this child is the most appropriate? 1. Impaired Gas Exchange Related to Pulmonary Congestion Secondary to the Increased Pulmonary Blood Flow 2. Deficient Fluid Volume Related to Hyperthermia Secondary to the Congenital Heart Defect 3. Acute Pain Related to the Effects of a Congenital Heart Defect 4. Hypothermia Related to Decreased Metabolic State

Correct Answer: 1 Rationale 1: Because of the increased pulmonary congestion, Impaired Gas Exchange would be an appropriate nursing diagnosis. Ventricular septal defects do not cause pain, fever, or deficient fluid volume.

Following parental teaching, the nurse is evaluating the parents' understanding of environmental control for their child's asthma management. Which statement by the parents' indicates appropriate understanding of the teaching? 1. "We will replace the carpet in our child's bedroom with tile." 2. "We're glad the dog can continue to sleep in our child's room." 3. "We'll be sure to use the fireplace often to keep the house warm in the winter." 4. "We'll keep the plants in our child's room dusted."

Correct Answer: 1 Rationale 1: Control of dust in the child's bedroom is an important aspect of environmental control for asthma management. When possible, pets and plants should not be kept in the home. Smoke from fireplaces should be eliminated.

The nurse is performing the initial assessment of a child newly diagnosed Kawasaki disease. Which symptoms would the nurse expect to assess with this child? 1. Dry, swollen, fissured lips 2. Non-palpable lymph nodes 3. Conjunctivitis with exudates 4. Cyanosis of the hands and feet

Correct Answer: 1 Rationale 1: Dry, swollen, fissured lips are symptoms of Kawasaki disease. Lymph nodes can be palpable, conjunctivitis is present but without exudates, and hands and feet are typically erythematous.

A child is admitted with infective endocarditis. Which nursing intervention is most appropriate for this child? 1. Start an intravenous line. 2. Place the child in contact isolation. 3. Place the child on seizure precautions. 4. Assist with a lumbar puncture.

Correct Answer: 1 Rationale 1: Infective endocarditis is treated with intravenous antibiotics for two to eight weeks. It is not contagious, so the child is not placed in contact isolation. Seizures are not a risk of infective endocarditis. A lumbar puncture is not a diagnostic test done for infective endocarditis.

The mother of a child with a heart defect is questioning the nurse about the child's diuretic. When teaching the mother about the medication, what should the emphasis from the nurse? 1. Close monitoring of output 2. The digitalization process 3. The possibility that pulses in the child might be weak 4. The child's increased appetite

Correct Answer: 1 Rationale 1: It is important to monitor the output of the child on a diuretic to determine effectiveness of the drug. Digitalization pulses are not associated with diuretics. The child will usually have a decreased appetite.

A child is admitted to the hospital with the diagnosis of laryngotracheobronchitis (LTB). Which nursing intervention is the priority for this child? 1. Administer nebulized epinephrine and oral or IM dexamethasone. 2. Administer antibiotics and assist with possible intubation. 3. Swab the throat for a throat culture. 4. Obtain a sputum specimen.

Correct Answer: 1 Rationale 1: Nebulized epinephrine and dexamethasone are given for LTB. Antibiotic administration and possible intubation are associated with epiglottitis. Throat cultures are not obtained for LTB because it is viral and swabbing the throat could cause complete obstruction to occur. Sputum specimens will not assist in the diagnosis of LTB.

The nurse is admitting an infant diagnosed with supraventricular tachycardia. Which intervention is the priority for this infant? 1. Apply ice to the face. 2. Perform Valsalva's maneuver. 3. Administer a beta blocker. 4. Prepare for cardioversion.

Correct Answer: 1 Rationale 1: Supraventricular tachycardia episodes are initially treated with vagal maneuvers to slow the heart rate when the infant is stable. In stable infants, the application of ice or iced saline solution to the face can reduce the heart rate. The infant is not capable of performing Valsalva's maneuver. Calcium channel blockers, not beta blockers, are the drugs of choice. Cardioversion is used in an urgent situation, but is not typically the initial treatment.

The nurse has admitted a child with tricuspid atresia. The nurse would expect which initial lab result? 1. A high hemoglobin 2. A low hematocrit 3. A high white blood cell count 4. A low platelet count

Correct Answer: 1 Rationale 1: The child's bone marrow responds to chronic hypoxemia by producing more red blood cells to increase the amount of hemoglobin available to carry oxygen to the tissues. This occurs in cases of cyanotic heart defects such as tricuspid atresia. Therefore, the hematocrit would not be low, the white blood cell count would not be high (unless an infection were present), and the platelets would be normal.

A child is showing signs of acute respiratory distress. Which position will the nurse place this child? 1. Upright 2. Side-lying 3. Flat 4. In semi-Fowler's

Correct Answer: 1 Rationale 1: Upright is correct because it allows for optimal chest expansion. Side-lying, flat, and semi-Fowler's (head up slightly) do not allow for as optimal chest expansion as the upright position

The nurse educator is teaching a group of nursing students how to perform a respiratory assessment for a newborn in the newborn intensive care unit (NICU) diagnosed with respiratory distress syndrome (RDS). Which normal characteristics of the newborn's respiratory system increase the risk for obstruction? Standard Text: Select all that apply. 1. Shorter and narrower airway 2. Higher trachea 3. Bronchial branching at different angles 4. Inadequate smooth muscle bundles 5. Diaphragmatic breather

Correct Answer: 1,2,3 Rationale 1: Normal characteristics of the pediatric respiratory system that increase the risk for obstruction include a shorter and narrower airway, a higher trachea, and a different angle for bronchial branching. Inadequate smooth muscle bundles and being diaphragmatic breathers are characteristics that do not increase the risk of obstruction.

The nurse is providing care to an infant who is diagnosed with bronchiolitis. Which breath sounds indicate the infant is experiencing respiratory distress? Standard Text: Select all that apply. 1. Tachypnea 2. Wheezing 3. Grunting 4. Retractions 5. Eupnea

Correct Answer: 1,2,3 Rationale 1: Wheezing and grunting are adventitious respiratory sounds that indicates respiratory distress in the neonate. Tachypnea is the term used to indicate a respiratory rate of greater than 60 breaths per minute in an infant. While this does indicate respiratory distress, tachynpea is not a type of breath sound. Retractions, or the use of accessory muscles, are indicative of respiratory distress in the neonate, but this is not a type of breath sound. Eupnea is the medical term for "normal breathing."

The nurse is assessing a school-age client who experienced blunt force trauma to the chest when an airbag deployed following a motor vehicle crash. Which areas of assessment are essential for this client? Standard Text: Select all that apply. 1. Monitor responsiveness and behavior. 2. Monitor SpO2. 3. Auscultate the lungs for crackles, wheezes, decreased breath sounds. 4. Document input and output. 5. Note changes in voice quality or coughing.

Correct Answer: 1,2,3,5 Rationale 1: The areas of assessment that are essential for this client include: monitoring for responsive and behavior in order to detect hypoxia and the potential for airway obstruction; monitoring SpO2 frequently to identify changes indicating deterioration in condition; auscultating the lungs for crackles, wheezes, decreased breath sound; and noting changes in voice quality or coughing. Documenting input and output is not a priority for this client.

An infant with tetralogy of Fallot is having a hypercyanotic episode ("tet" spell). Which nursing interventions are appropriate for the nurse to implement for this infant? Standard Text: Select all that apply. 1. Place the child in knee-chest position. 2. Draw blood for a serum hemoglobin. 3. Administer oxygen. 4. Administer morphine and propranolol intravenously as ordered. 5. Administer Benadryl as ordered.

Correct Answer: 1,3,4 Rationale 1: When an infant with tetralogy of Fallot has a hypercyanotic episode, interventions should be geared toward decreasing the pulmonary vascular resistance. Therefore, the nurse would place the infant in knee-chest position (to decrease venous blood return from the lower extremities), and administer oxygen, morphine, and propranolol (to decrease the pulmonary vascular resistance). The nurse would not draw blood until the episode had subsided, because unpleasant procedures are postponed. Benadryl is not appropriate for this child.

A patient has been newly diagnosed with chronic lung disease. In discussing his condition with the nurse, which of his statements would indicate a need for further education? A) "I'll make sure that I rest between activities so I don't get so short of breath." B) "I'll practice the pursed-lip breathing technique to improve my exercise tolerance." C) "If I have trouble breathing at night, I'll use two to three pillows to prop up." D) "If I get short of breath, I'll turn up my oxygen level to 6 L/min."

D

A school nurse is planning care for a school-age child recently diagnosed with asthma. Which items will the school nurse include in the plan of care at the school? Standard Text: Select all that apply. 1. Maintain a log of quick-relief medication administration. 2. Call the parents if quick-relief medications work appropriately. 3. Assess for symptoms of exercise-induced bronchospasm. 4. Coordinate education of the child's teachers. 5. Conduct a support group for all children with asthma.

Correct Answer: 1,3,4,5 Rationale 1: Appropriate interventions for the school nurse to include in the plan of care include: keeping a log of the quick-relief medications administered; assessing the child for exercise-induced bronchospasms and reporting, if needed; coordinating education of the child's teachers; and conducting a support group for all children in the school with asthma. The nurse would only call the parents if the quick-relief mediation was not effective in treating the child's symptoms.

The nurse is providing care to a school-age client admitted to the emergency department following a motor vehicle crash. The client is exhibiting symptoms of hypovolemic shock. Which nursing interventions are appropriate for this client? Standard Text: Select all that apply. 1. Monitor hemoglobin and hematocrit. 2. Monitor liver enzymes. 3. Administer oxygen, as needed. 4. Administer a dextrose solution. 5. Monitor blood glucose.

Correct Answer: 1,3,5 Rationale 1: Nursing care for a client experiencing hypovolemic shock is aimed at monitoring the child's condition and response to clinical therapy. It is appropriate for the nurse to monitor hemoglobin, hematocrit, and blood glucose. The nurse will also administer oxygen. The nurse will administer large volumes of crystalloid fluids (normal saline or lactated Ringer's), not dextrose. It is not necessary to monitor liver enzymes for this client.

The nurse is providing care to an infant in the emergency department. Upon assessment, the infant is noted to have to be experiencing tachypnea, wheezing, retractions, and nasal flaring. The infant is irritability and the parents state the infant has had poor fluid intake for two days. Pulse ox reading is currently at 85% on room air. The infant's blood gas is pending. Which diagnosis does the nurse anticipate for this infant? 1. Bronchitis 2. Bronchiolitis 3. Pneumonia 4. Active pulmonary tuberculosis

Correct Answer: 2 Rationale 1: The nurse anticipates the infant will be diagnosed with bronchiolitis. Symptoms of bronchiolitis include mild respiratory symptoms that progress to tachypnea, wheezing, retractions, nasal flaring, irritability, poor fluid intake, hypoxia, cyanosis, and decreased mental status. Symptoms of bronchitis include a dry hacking cough, increases in severity at night, painful chest and ribs. Symptoms of pneumonia include initial rhinitis and cough, followed by fever, crackles, wheezes, dyspnea, tachypnea, restlessness, diminished breath sounds. Symptoms of active pulmonary tuberculosis include persistent cough, decreased appetite, weight loss or failure to gain weight, low-grade fever, night sweats, chills, enlarged lymph nodes.

A child recently had a heart transplant and the nurse teaches the parents the importance of administering cyclosporine A. Which statement by the parents indicates an appropriate understanding of the teaching session? 1. "Cyclosporin A reduces serum-cholesterol level." 2. "Cyclosporin A prevents rejection." 3. "Cyclosporin A treats hypertension." 4. "Cyclosporin A treats infections."

Correct Answer: 2 Rationale 1: Cyclosporin A is given to prevent rejection. Lovastatin is given to reduce serum-cholesterol level, calcium channel blockers may be used to treat hypertension, and an antibiotic may be given to treat an infection.

The nurse is teaching the parents of a newly diagnosed cystic fibrosis patient how to administer the pancreatic enzymes. How often will the nurse teach the parents to administer the enzymes? 1. Two times per day 2. With meals and snacks 3. Every 6 hours around the clock 4. Four times per day

Correct Answer: 2 Rationale 1: Pancreatic enzymes are administered with meals and large snacks. A scheduled time would not be appropriate because the enzymes are used to assist in digestion of nutrients.

The nurse is teaching the parents of a group of cardiac patients. Which teaching guideline will the nurse include for any child who has undergone cardiac surgery? 1. The child should be restricted from most play activities. 2. The child should be evaluated to determine if prophylactic antibiotics for dental, oral, or upper-respiratory-tract procedures are necessary. 3. The child should not receive routine immunizations. 4. The child can be expected to have a fever for several weeks following the surgery.

Correct Answer: 2 Rationale 1: Parents should be taught that the child may need prophylactic antibiotics for some dental procedures, according to the American Heart Association, to prevent endocarditis. The child should live a normal and active life following repair of a cardiac defect. Immunizations should be provided according to the schedule, and any unexplained fever should be reported.

A child is admitted to the hospital with pneumonia. The child's oximetry reading is 88 percent upon admission to the pediatric floor. Which is the priority nursing intervention for this child? 1. Obtain a blood sample to send to the lab for electrolyte analysis. 2. Begin oxygen per nasal cannula. 3. Medicate for pain. 4. Begin administration of intravenous fluids.

Correct Answer: 2 Rationale 1: Pulse oximetry reading should be 92 or greater. Oxygen by nasal cannula should be started initially. Medicating for pain, administering IV fluids, and sending lab specimens can be done once the child's oxygenation status has been addressed

The nurse is teaching a group of mothers of infants about the benefits of immunization. Which immunization will the nurse teach to the mothers that can assist in preventing the life-threatening disease epiglottitis? 1. Measles, mumps, and rubella (MMR) 2. Haemophilus influenzae type B (HIB) 3. Hepatitis B 4. Polio

Correct Answer: 2 Rationale 1: The Haemophilus influenzae type B (HIB) immunization can assist in prevention of epiglottitis. Hepatitis B, measles, mumps, rubella, and the polio virus are not causative agents for epiglottitis

The nurse is providing care to an adolescent child who is at risk for developing adult-onset cardiovascular disease. Which teaching points will decrease the adolescent's risk? Standard Text: Select all that apply. 1. Encourage a decrease in smoking. 2. Limit fat intake to 20% to 35% of intake. 3. Encourage participation in vigorous exercise for at least 30 minutes. 4. Maintain a normal weight. 5. Include high-fat dairy products in the daily diet.

Correct Answer: 2,3,4 Rationale 1: Teaching points that will decrease the adolescent's risk of developing adult-onset cardiovascular disease include: limiting fat intake to 20% to 35% of total daily intake; encouraging the participation in vigorous exercise at least 30 minutes each day; and maintaining a normal weight. The adolescent and family members should be encouraged to stop smoking, not just to decrease smoking. The family should be educated to include low-fat dairy products in the daily diet.

The nurse is preparing to discharge an infant with a congenital heart defect. The infant will be cared for at home by the parents until surgery. Which items will the nurse include in the discharge teaching for this infant and family? Standard Text: Select all that apply. 1. Allow the infant to feed for 60 minutes. 2. Hold the infant at a 45 degree angle. 3. Encourage frequent hand hygiene. 4. Notify the health care provider for fever. 5. Pump the breasts and feed with a bottle if weight gain is an issue.

Correct Answer: 2,3,4,5 Rationale 1: Children are often managed at home until surgery. The parents should hold the infant at a 45 degree angle to decrease tachypnea. The parents should also encourage frequent hand hygiene to decrease the risk of infection. It is important to notify the health care provider for a fever, as the infant will be at risk for dehydration and digoxin toxicity. If the mother is breastfeeding and the infant is losing weight, the mother should be encouraged to pump the milk and feed the infant from a bottle, but each feeding should be limited to 30 minutes. Tube feedings may be needed for this infant to conserve calories expenditure.

A child is prescribed rifampicin for treatment of tuberculosis. For which length of time will the nurse tell the parents that this child must remain on the medication? 1. 2 months 2. 4 months 3. 6 months 4. 8 months

Correct Answer: 3 Rationale 1: Active and latent TB are treated with isoniazid, rifampicin, pyrazinamide, and ethambutol. Therapy for active TB usually involves a 6-month regimen consisting of isoniazid, rifampicin, pyrazinamide, and ethambutol for the first 2 months and isoniazid and rifampicin for the remaining 4 months. Therefore, the child will remain on rifampicin for a total of 6 months.

Which athletic activity can the nurse recommend for a school-age client with pulmonary-artery hypertension? 1. Cross-country running 2. Soccer 3. Golf 4. Basketball

Correct Answer: 3 Rationale 1: A child with pulmonary-artery hypertension should have exercise tailored to avoid dyspnea. Golf would require less exertion than soccer, basketball, or cross-country running.

Parents of a child admitted with respiratory distress are concerned because the child won't lie down and wants to sit in a chair leaning forward. Which response by the nurse is the most appropriate? 1. "This helps the child feel in control of his situation." 2. "The child needs to be encouraged to lie flat in bed." 3. "This position helps keep the airway open." 4. "This confirms the child has asthma."

Correct Answer: 3 Rationale 1: Leaning forward helps keep the airway open. The child is not in control just because he is leaning forward. Lying flat in bed will increase the respiratory distress. This position does not confirm asthma.

A newborn is suspected of having cystic fibrosis. As the child is being prepared for transfer to a pediatric hospital, the mother asks the nurse which symptoms made the practitioner suspect cystic fibrosis. Which response by the nurse is the most appropriate? 1. Steatorrheic stools 2. Constipation 3. Meconium ileus 4. Rectal prolapse

Correct Answer: 3 Rationale 1: Newborns with cystic fibrosis may present in the first 48 hours with meconium ileus. Steatorrhea, constipation, and rectal prolapse may be signs of cystic fibrosis seen in an older infant or child.

A child has been admitted to the hospital unit in congestive heart failure (CHF). Which symptom would the nurse anticipate upon assessment of the child? 1. Weight loss 2. Bradycardia 3. Tachycardia 4. Increased blood pressure

Correct Answer: 3 Rationale 1: Tachycardia is a sign of congestive heart failure because the heart attempts to improve cardiac output by beating faster. Bradycardia is a serious sign and can indicate impending cardiac arrest. Blood pressure does not increase in CHF, and the weight, instead of decreasing, increases because of retention of fluids.

A toddler is started on digoxin (Lanoxin) for cardiac failure. Which is the initial symptom the nurse would assess if the child develops digoxin (Lanoxin) toxicity? 1. Lowered blood pressure 2. Tinnitus 3. Ataxia 4. A change in heart rhythm

Correct Answer: 4 Rationale 1: An early sign of digoxin (Lanoxin) toxicity is a change in heart rhythm. Digoxin (Lanoxin) toxicity does not cause lowered blood pressure, tinnitus (ringing in the ears), or ataxia (unsteady gait).

The nurse finds that an infant has stronger pulses in the upper extremities than in the lower extremities, and higher blood pressure readings in the arms than in the legs. Which assessment will the nurse perform next on this infant? 1. Pedal pulses 2. Pulse oximetry level 3. Hemoglobin and hematocrit values 4. Blood pressure of the four extremities

Correct Answer: 4 Rationale 1: Coarctation of the aorta can present with stronger pulses in the upper extremities than in the lower extremities and higher blood pressure readings in the arms than in the legs because of obstruction of circulation to the lower extremities. Blood pressure values of the four limbs should be the next assessment data collected. Pedal pulses, pulse oximetry, and labs themselves will not provide the data needed.

A nurse is assessing a neonate. Which assessment finding indicates that the neonate's respiratory status is worsening? 1. Acrocyanosis 2. Arterial CO2 of 40 3. Periorbital edema 4. Grunting respirations with nasal flaring

Correct Answer: 4 Rationale 1: Grunting respirations with nasal flaring indicates respiratory status is becoming worse. Acrocyanosis (cyanosis of the extremities) is a normal finding in a neonate. CO2 of 40 is within a normal range. Periorbital edema does not necessarily mean deterioration in respiratory status.

The practitioner changes the medications for the child with asthma to salmeterol (Serevent). The mother asks the nurse what this drug will do. The nurse explains that salmeterol (Serevent) is used to treat asthma because the drug produces which characteristic? 1. Decreases inflammation 2. Decreases mucous production 3. Controls allergic rhinitis 4. Dilates the bronchioles

Correct Answer: 4 Rationale 1: Salmeterol (Serevent) is a long-acting beta2-agonist that acts by bronchodilating. Steroids are anti-inflammatory, anticholinergics decrease mucous production, and antihistamines control allergic rhinitis.

The nurse is admitting a patient with chronic obstructive pulmonary disease (COPD). During the initial head-to-toe assessment the patient's pulse oximetry reading is 89% on room air. What is the nurse's first priority? a. Administer oxygen immediately @ 4L/NC. b. Call the primary health care provider for an order for oxygen. c. Assist the patient into a recumbent position. d. Determine the patient's normal pulse oximetry reading.

D

A nurse delegates the task of neonatal vital-sign assessment to a nurse technician. Which instruction will the nurse give to the technician prior to assign care? 1. Report any neonate using abdominal muscles to breathe. 2. Report any neonate with apnea for 10 seconds. 3. Count respirations for 15 seconds and multiply by 4 to get the rate for 1 minute. 4. Report any neonate with a breathing pause that lasts 20 seconds or longer.

Correct Answer: 4 Rationale 1: The abnormal assessment finding for vital signs that the nurse should instruct a nurse technician to report is any breathing pause by a neonate lasting longer than 20 seconds. This can indicate apnea and could lead to an apparent life-threatening event (ALTE). A breathing pause of 10 seconds or less is called periodic breathing and is a normal pattern for a neonate. Respirations should be counted for 1 minute, not 15 seconds. It is normal for neonates to use abdominal muscles for breathing.

The nurse is checking peripheral perfusion to a child's extremity following a cardiac catheterization. Which assessment finding indicates adequate peripheral circulation to the affected extremity? 1. A capillary refill of greater than three seconds 2. A palpable dorsalis pedis pulse but a weak posterior tibial pulse 3. A decrease in sensation with a weakened dorsalis pedis pulse 4. A capillary refill of less than three seconds with palpable warmth

Correct Answer: 4 Rationale 1: The nurse checks the extremity to determine adequacy of circulation following a cardiac catheterization. An extremity that is warm with capillary refill of less than three seconds has adequate circulation. Other indicators of adequate circulation include palpable pedal (dorsalis and posterior tibial) pulses, adequate sensation, and pinkness of skin color. If the capillary refill is over three seconds; if any of the pedal pulses are absent and/or weakened; or if the extremity is cool, cyanotic, or lacking sensation, circulation may not be adequate.

A child with asthma will be receiving an oral dose of prednisone. The order reads prednisone 2 mg/kg per day. The child weighs 50 lbs. The child will receive ____ milligrams daily. (Round the answer.)

Correct Answer: 45.5 = 46 Rationale: 22.7 × 2 = 45.5 (46)

When performing a respiratory assessment, you auscultate wet, popping sounds at the inspiratory phase of each respiration as

Crackles Crackles, which are sometimes called rales, are wet, popping sounds created by air moving through liquid or by collapsed alveoli snapping open on inspiration. They are most common at the end of inspiration.

A 47-year-old woman with a history of diabetes and hypertension calls the clinic complaining of epigastric pain and shortness of breath with activity. She has taken antacids with no relief. What type of pain is the patient experiencing? a. Pericardial pain b. Pleuritic chest pain c. Musculoskeletal pain d. Cardiac pain

D

A patient has been admitted to the pulmonary unit of the hospital with right lower lobe pneumonia and history of chronic obstructive pulmonary disease (COPD). During the initial assessment the nurse notes that the patient has a respiration rate of 18 with retractions, tachycardia, and complains of dyspnea and dizziness. The nurse identifies that these are clinical signs of which condition? a. Paroxysmal nocturnal dyspnea b. Orthopnea c. Hemoptysis d. Hypoxia

D

What is the best indicator the nurse can use to determine the adequacy of a patient's cardiac output? a. Stroke volume b. Myocardial contractility c. Afterload d. Cardiac index

D

A patient is admitted with the diagnosis of severe left-sided heart failure. What adventitious lung sounds are expected on auscultation? Sonorous wheezes in the left lower lung Rhonchi mid sternum Crackles only in apex of lungs Inspiratory crackles in lung bases

D (Decreased effective contraction of left side of heart leads to back up of fluid in the lungs increasing hydrostatic pressure and causing pulmonary edema.)

Which of the following diagnosis is a patient who started smoking in adolescence and continues to smoke for 40 years at this risk for? Alcoholism and hypertension Obesity and diabetes Stress-related illnesses Cardiopulmonary disease and lung cancer

D (Effects of nicotine on blood vessels and lung tissue have increasing pathological effects on the cardiovascular and pulmonary systems.)

The nurse is reviewing the results of the patient's diagnostic testing. Of the following results, the finding that falls within expected or normal limits is: A.) Palpable, elevated hardened area around a tuberculosis skin testing site. B.) Sputum for culture and sensitivity identifies mycobacterium tuberculosis C.) Presence of acid fast bacilli in sputum D.) Arterial oxygen tension (PaO2) of 95 mmHg

D (Arterial oxygen tension (PaO2) of 95 mmHg (A palpable, elevated, hardened area surrounding a tuberculosis skin testing site is indicative of an antigen-antibody reaction and is considered a positive skin test. Sputum for culture and sensitivity noted the presence of an organism and acid fast bacilli. Normal arterial oxygen tension (PaO2) ranges between 95-100 mmHg.)

Which of the following instructions given to a patient who is about to undergo Holter monitoring is most appropriate? A) "You may remove the monitor only to shower or bathe." B) "You should connect the monitor whenever you feel symptoms." C) "You should refrain from exercising while wearing this monitor." D) "You will need to keep a diary of all your activities and symptoms."

D) "You will need to keep a diary of all your activities and symptoms." A Holter monitor is worn for at least 24 hours while a patient continues with usual activity and keeps a diary of activities and symptoms. The patient should not take a bath or shower while wearing this monitor.

The blood pressure of a 71-year-old patient admitted with pneumonia is 160/70 mm Hg. Which of the following is an age-related change that contributes to this finding? A) Stenosis of the heart valves B) Decreased adrenergic sensitivity C) Increased parasympathetic activity D) Loss of elasticity in arterial vessels

D) Loss of elasticity in arterial vessels An age-related change that increases the risk of systolic hypertension is a loss of elasticity in the arterial walls. Because of the increasing resistance to flow, pressure is increased within the blood vessel and hypertension results.

A nurse is administering eardrops to an 8-year-old patient with an ear infection. How does the nurse pull the patient's ear when administering the medication? A. Outward B. Back C. Upward and back D. Upward and outward

D. Rationale: Eardrops are administered with the ear positioned upward and outward for patients greater than 3 years of age.

A nurse accidently gives a patient a medication at the wrong time. The nurse's first priority is to: A. Complete an occurrence report. B. Notify the health care provider. C. Inform the charge nurse of the error. D. Assess the patient for adverse effects.

D. Rationale: Patient safety and assessing the patient are priorities when a medication error occurs.

A patient is to receive cephalexin (Kefl ex) 500 mg PO. The pharmacy has sent 250-mg tablets. How many tablets does the nurse administer? A. ½ tablet B. 1 tablet C. 1 ½ tablets D. 2 tablets

D. Rationale: Using dimensional analysis: Tablets = 1 tablet/250 mg x 500 mg = 500/250 = 2 tablets.

A patient has been diagnosed with severe iron deficiency anemia. During physical assessment, which of the following symptoms are associated with decreased oxygenation? Increased breathlessness but increased activity tolerance Decreased breathlessness and decreased activity tolerance Increased activity tolerance and decreased breathlessness Decreased activity tolerance and increased breathlessness

Decreased activity tolerance and increased breathlessness (Hypoxia occurs due to decreased circulating blood volume which leads to decreased oxygen to muscles causing fatigue and decreased activity tolerance as well as a feeling of shortness of breath.)

Left-sided heart failure is characterized by:

Decreased functioning of the left ventricle Left-sided heart failure is an abnormal condition characterized by decreased functioning of the left ventricle. If left ventricular failure is significant, the amount of blood ejected from the left ventricle drops greatly, which results in decreased cardiac output

The nurse is caring for a client with a chest tube in the right thorax. On first assessment the nurse notes that there is bubbling in the water-seal chamber. This client is scheduled to undergo a chest x-ray examination, and the transporters have arrived to take him by wheelchair to the radiology department. The nurse considers whether the chest tube should be clamped or not during the trip to the radiology department. The nurse makes the which correct decision?

Do not clamp the chest tube and disconnect it from the wall suction. A bubbling chest tube (in the water-seal portion) should never be clamped because it provides the only exit for air accumulating in the pleural space. If the tube is clamped, tension pneumothorax could occur, which could be fatal. There is no advantage to clamping the chest tube but venting the system. Clamping of the chest tube prevents communication of the chest tube with the venting system or with the wall suction. There is no such thing as "temporary suction" for a chest tube system.

Carbon monoxide (CO) is a toxic inhalant that decreases the oxygen-carrying capacity of blood by:

Forming a strong bond with hemoglobin CO is the most common toxic inhalant and decreases the oxygen-carrying capacity of blood. In CO toxicity, hemoglobin strongly binds with carbon monoxide, creating a functional anemia. Because of the strength of the bond, carbon monoxide does not easily dissociate from hemoglobin, which makes hemoglobin unavailable for oxygen transport.

When assessing peripheral vascular status of the lower extremities, you place your fingertips on your patients great toe and those of the tow next to it. Which pulse are you palpating?

Dorsal Pedis In the lower extremities, the most common pulse tested is the doralis pedis pulse, found on the dorm of the foot between the extender tendons to the great toe and the toe next to it.

For which of the following inhalation medication delivery methods is it important for the nurse to assess the patient's ability to inhale deeply before administering the medication?

Dry poweder inhaler (DPI) -this method has no propellant and requires a deep inhalation to trigger the release of medication

Most common causative pathogen in UTI

Escheria coli

A simple and cost-effective method for reducing the risks of stasis of pulmonary secretions and decreased chest wall expansion is:

Frequent change of position Changing the client's position frequently is a simple and cost-effective method for reducing the risk of pneumonia associated with stasis of pulmonary secretions and decreased chest wall expansion. Oxygen humidification, chest physiotherapy, and use of antiinfectives are all helpful, but are not cost effective.

As part of your general survey, you find that your patient has a body mass index (BMI) of 23. From the finding you determine that

Has a body mass index within normal ,limits. BMI is a measurement of an adults body fat based on height and weight. Generally,a BMI between 18.5 and 24.9 reflects a normal amount of body fat. A patient with a BMI below 18.5 is considered underweight; a patient with a BMI of 25 or above is considered overweight; and one with a BMI of 30 or above is considered obese.

A person who starts smoking in adolescence and continues to smoke into middle age:

Has an increased risk for cardiopulmonary disease and lung cancer The risk of lung cancer is 10 times greater for a person who smokes than for a nonsmoker. Cigarette smoking worsens peripheral vascular and coronary artery disease. Inhaled nicotine causes vasoconstriction of peripheral and coronary blood vessels, increasing blood pressure and decreasing blood flow to peripheral vessels.

A nurse is teaching the daughter of an older adult patient how to instill eye drops in the patient's right eye. Which of the following statements indicates that the daughter has understood the directions?

I will pull down her lower eyelid and drop the medication inside. -This method will allow the medication to be distributed evenly across the eye with less discomfort.

A nurse is administering a subcutaneous injection to a patient. What data should the nurse recognize as the highest priority to prevent potential complications?

Identify if the Pt has allergies to the medication

When using and maintaining your stethoscope, it is important to

Inset earpieces at an angle toward your nose. Angling the earpiece toward your nose helps ensure that sounds are effectively transmitted to your eardrums.

When performing a complete, head to toe assessment, which physical-assessment technique should you perform

Inspection Inspection is the process of observation. You will first inspect the body systematically, observing for normal as well as abnormal physical signs. When assessing, and auscultation. Abdominal assessments is an exception, since any manipulation of or pressure on the abdomen may stimulate peristalsis, the waves of contraction that propel contents through the gastrointestinal tract, and thus alter the patients bowel sound. So, when assessing the abdomen, inspection is still first, but auscultation comes before percussion and palpation.

You are performing a physical examination of the spine for an adult. Which of the following findings is common in an older adult?

Kyphosis kyphosis, is pronounces "hunchback" curvature of the spine, is an abnormal angulation of the posterior curve of the thoracic spine, usually a result of osteoporosis. It is most common in older adults and tends to increase with aging. This pronounces convexity of the thoracic spines also common in older patients who have had vertebral fractures.

Mrs. Nelson is talking with the nurse about the dietary needs of her 23-month old daughter, Laura. Which of the following responses by the nurse would be appropriate?

Laura needs fewer calories in relation to her body weight now than she did as an infant

A patient who has been hospitalized after experiencing a heart attack will most likely receive a diet consisting of:

Low fat, low sodium, and high carbohydrates. The recommended diet from the AHA to reduce risk factors for the development of hypertension and coronary heart disease

You are instructing the patient about self-administration of insulin. What site would you suggest?

Lower abdomen (at least 2 in from umbilicus, should be recommended for self-administration of insulin)

Pt asks why insulin is injected subcutaneously. Why?

Medication absorption is slower b/c it has a relatively poor vascular system, an important factor in the effectiveness of insulin therapy. (It is the abundance of the vascular supply that is the primary factor in the speed with which meds enter into the bloodstream

A nurse is caring for a patient who has been prescribed a fluticasone propinate (Flovent HFA) inhaler with a spacer. The patient asks the nurse why a spacer is needed with the inhaler. Which of the following responses by the nurse is correct?

More medication is delivered to the lungs when you use a spacer. -A spacer slows down and breaks up the medication, allowing the patient to better control the flow of medication. This, in turn, decreases the amount of medication deposited in the oropharynx

What is a term that indications a medication given by injection?

Parenteral

All of the following patients are at risk for alteration in nutrition except:

Patient L, whose weight is 10% above his ideal body weight

The nurse is assessing a 76-year-old man in a nursing home with a diagnosis of UTI. The nurse notes that the patient is complaining of right flank pain. To assess for tenderness, the nurse should gently do which of the following?

Percuss the costoverbial angle

A nurse is preparing to instill antibiotic ear drops into a toddler's ear. Which of the following techniques should the nurse use when adminsitering ear drops to this patient?

Pull the patient's auricle down and back to open the canal when administering ear drops -the auricle should be pulled down and back for young children, and up and out for adults.

While performing an abdominal assessment, you place your fingertips over the patients painful area and gradually increase the pressure. Patient reports increased pain on release of pressure, you document that your patient has positive

Rebound tenderness This procedure elicits rebound tenderness- an increase in pain when deep palpation over a tender are is released. Rebound tenderness in the right lower quadrant at McBurney's point (one third the distance from the anterior iliac crest to the umbilicus) is one sign of acute appendicitis.

The nurse is concerned when a client's heart rate, which is normally 95 beats per minute, rises to 220 beats per minute, because a rate this high will:

Reduce coronary artery perfusion Coronary arteries fill and perfuse the myocardium (heart muscle) during diastole. When the heart rate is elevated, more time is spent in systole and less in diastole; hence, the myocardium may not be perfused adequately. The client may be exhausted, but the primary concern is myocardial perfusion. Major organs will adjust to increased blood flow. This is usually not a problem. With a heart rate this high, metabolic rate will be increased, not decreased.

A nurse is preparing to administer an insulin injection to a patient. What is appropriate?

Rotate injection sites to avoid tissue injury

A nurse suspects that a patient may be experiencing urinary retention. What should the nurse expect to find on assessment of this patient?

Small amounts of urine voided 2 to 3 times per hour

Which of the following is the most accurate method of bedside confirmation of placement of a small- bore nasogastric tube?

Test the pH of withdrawn gastric contents. The measurement of pH of secretions withdrawn from the feeding tubes helps to differentiate the location of the tube.

A client is receiving oxygen via a nonrebreathing mask. A crucial nursing assessment the nurse performs is to be sure that:

The bag attached to the mask is inflated at all times If the bag attached to a nonrebreathing mask is deflated, the client is at risk for breathing in large amounts of exhaled carbon dioxide. The bag should be maximally inflated at all times

Most common health care-acquire infections

UTI

A nurse is adminsitering aspirin 82 mg PO daily as prescribed. The medication is scheduled for 0800 hours. Which of the following demonstrates proper use of one of the six rights of medication administration?

The nurse documents that the apirin was given at 0825 -all routinely ordered medications should be given within 60 min of the time ordered (30 min before or after the prescribed time)

Phenytoin (Dilantin), 100 mg PO, is ordered to be given through a nasogastric tube. Phenytoin is available as 30 mg/5 mL. How much would the nurse administer?

X= 16.66 OR 17 mL

Captopril (Capoten), 12.5 mg PO, is ordered. Captopril is available as 25-mg tablets. How many tablets would the nurse administer?

X=1/2 OR 0.5 TABLET

Metoprolol (Lopressor), 25 mg PO, is ordered. Metoprolol is available as 50-mg tablets. How many tablets would the nurse administer?

X=1/2 OR 0.5 TABLET

An adult client visits the clinic and tells the nurse that she has had headaches recently that are intense and stabbing and often occur in the late evening. The nurse should suspect the presence of a. cluster headaches b. migraine headaches c. tensio headaches d. tumor-related headaches

a. cluster headaches

A patient with a fecal impaction has an order to remove stool digitally. In which order will the nurse perform the steps, starting with the first one? 1. Obtain baseline vital signs. 2. Apply clean gloves and lubricate. 3. Insert index finger into the rectum. 4. Identify patient using two identifiers. 5. Place patient on left side in Sims' position. 6. Massage around the feces and work down to remove. a.4, 1, 5, 2, 3, 6 b.1, 4, 2, 5, 3, 6 c.4, 1, 2, 5, 3, 6 d.1, 4, 5, 2, 3, 6

a. 4 ,1,5,2,3,6

You are to administer a medication to Mr. Brown. In addition to checking his identification bracelet, you can correctly verify his identity by doing which of the following? A. Asking the patient his name B. Reading the patient's name on the sign over the bed C. Asking the patient's roommate to verify his name D. Asking, "Are you Mr. Brown?"

a. A sign over the patient's bed may not always be current. The roommate is an unsafe source of information. The patient may not hear his name but may reply in the affirmative anyway (e.g., a person with a hearing deficit).

2. The process by which a drug is transferred from its site of entry into the body to the bloodstream is known as which of the following? A. Absorption B. Distribution C. Metabolism D. Excretion

a. Distribution, metabolism, and excretion occur after the drug has been absorbed.

A patient with colon cancer has recently undergone surgery to remove a portion of the colon. The patient asks how often the colostomy pouching system should be changed. What is the best response by the nurse? a. Every 3 to 7 days b. Every 10 to 14 days c. When the pouch is one third to one half full of stool d. Not until the system starts to leak or smell bad

a. Every 3-7 days c. when the pouch is 1/3 to 1/2 full of stool

A nurse has delegated the administration of a tap water enema to a nursing assistive personnel (NAP). The assistive personnel demonstrates understanding of the procedure when she states which of the following? (Select all that apply.) a. "I will lower the enema when the patient complains of cramping." b. "I will speed up the enema administration when the patient complains of cramping." c. "I will withdraw the tube when the patient complains of cramping." d. "I will clamp the tubing when the patient complains of cramping." e. "I will fill the bag with hot water because it will cool while I am administering the enema." f. "I will have the patient sit on the toilet while I am administering the enema."

a. I will lower the enema when patient complains of cramping d. I will clamp the tubing when the patient complains of cramping

Which of the following conditions could affect the function of the digestive process? (Select all that apply.) a. Increase in mobility b. Diagnostic testing c. Increase in nutrition d. Medications e. Increase in fluid intake f. Surgery

a. Increase in mobility b. Diagnostic testing d. medications f. surgery

The nurse receives a patient from the emergency department with the diagnosis of ileus. The nurse expects the health care provider to order NPO for dietary status, and insert a nasogastric tube. The nurse knows that the purpose of the nasogastric tube is to do which of the following? a. Decompress the stomach until peristalsis returns. b. Provide tube feedings until peristalsis resumes. c. Allow for the release of flatulence. d. To keep the stomach expanded until peristalsis resumes.

a. decompress the stomach until peristalsis returns

A nurse is teaching a patient about the large intestine in elimination. In which order will the nurse list the structures, starting with the first portion? a.Cecum, ascending, transverse, descending, sigmoid, and rectum b.Ascending, transverse, descending, sigmoid, rectum, and cecum c.Cecum, sigmoid, ascending, transverse, descending, and rectum d.Ascending, transverse, descending, rectum, sigmoid, and cecum

a. cecum, ascending, transverse, descending, sigmoid, and rectum

A patient is using laxatives three times daily to lose weight. After stopping laxative use, the patient has difficulty with constipation and wonders if laxatives should be taken again. Which information will the nurse share with the patient? a.Long-term laxative use causes the bowel to become less responsive to stimuli, and constipation may occur. b.Laxatives can cause trauma to the intestinal lining and scarring may result, leading to decreased peristalsis. c.Long-term use of emollient laxatives is effective for treatment of chronic constipation and may be useful in certain situations. d.Laxatives cause the body to become malnourished, so when the patient begins eating again, the body absorbs all of the food, and no waste products are produced.

a. long-term laxative use causes the bowel to become less responsive to stimuli, and constipation may occur

A female client visits the clinic and tells the nurse that she frequently experiences severe recurring headaches that sometimes last for several days and are accompanied by nausea and vomiting. The nurse determines that the type of headache the client is describing is a a. migraine headache b. cluster headache c. tension headache d. tumor-related headache

a. migraine headache

The nurse assesses an adult client's head and neck. While examining the carotid arteries, the nurse assesses each artery individually to prevent a a. reduction of the blood supply to the brain b. rapid rise in the client's pulse rate c. premature ventricular heart sound d. decreased pulse pressure

a. reduction of the blood supply to the brain

The home health nurse is visiting a 67-year-old widow who lives at home by herself. The patient voices a concern about constipation. What is the best way for the nurse to approach the patient's concern? a. "Tell me why you think you are constipated." b. "Have you noticed that your stools are hard?" c. "How frequently are you having a bowel movement?" d. "What color is your stool?"

a. tell me why you think you are constipated

(**)The nurse is devising a plan of care for a patient with the nursing diagnosis of Constipation related to opioid use. Which outcome will the nurse evaluate as successful for the patient to establish normal defecation? a.The patient reports eliminating a soft, formed stool. b.The patient has quit taking opioid pain medication. c.The patient's lower left quadrant is tender to the touch. d.The nurse hears bowel sounds in all four quadrants.

a. the patient reports eliminating a soft, formed stool

Nurse will be administering several meds to pt. who has enteral feedings via bore NG tube. Nurse administers meds correctly by...

administering meds via large bulb syringe (WRONG) **Infusing each medication by gravity and flushing with water before and after instillation.

The health care provider ordered a 24-hour urine specimen to test the renal function of a patient admitted with acute renal failure. The nurse has prepared all the necessary equipment and has asked the patient if he or she needs to void. The nurse knows that the 24-hour collection period will begin

after the first voided specimen is discarded.

Which should nurse assess before administering meds through NG tube?

amount of residual volume left in stomach

.A patient is scheduled for an intravenous pyelogram (IVP). What should the nurse do for this diagnostic examination?

asses pt for iodine allergy

reflex urinary incontince is at risk for developing

autonomic dysreflexia (severe elevation of BP, pulse rate, and diaphoresis)

The nurse caring for several patients on the surgical unit of the hospital. The nurse knows that constipation can be a significant health hazard and encourages the postoperative patients to drink fluids. Which one of the following patients is most at risk from complications related to constipation? a. A 35-year-old man with back surgery b. A 47-year-old woman with an abdominal hysterectomy c. A 29-year-old women with carpal tunnel surgery d. A 77-year-old man with hip surgery

b. 47-year-old with abdominal hysterectomy

The nurse is caring for a patient on the GI floor who has anemia. When reviewing the patient's recent lab work, which lab test would the nurse expect to be decreased? a. Total bilirubin b. Hemoglobin and hematocrit c. Serum amylase d. Ova and parasites

b. Hemoglobin and hematocrit

A 45-year-old Catholic Hispanic-American patient has been admitted to the hospital with pneumonia. On admission, the patient did not identify any food preferences or food allergies. The nurse notes that the patient has requested that the family provide all meals during the hospital stay. This is most likely related to which of the following? a. Food preferences b. Hispanic cultural traditions c. Religious preferences d. Food sensitivities

b. Hispanic cultural traditions

To maintain normal elimination patterns in a hospitalized patient, why should the nurse encourage the patient to take time to defecate 1 hour after meals? a. The presence of food stimulates peristalsis. b. Mass colonic peristalsis occurs at this time. c. Irregularity helps to develop a habitual pattern. d. Neglecting the urge to defecate can cause diarrhea.

b. Mass colonic peristalsis occurs at this time

A patient requires 40 units of NPH insulin and 10 units of regular insulin daily subcutaneously. Which of the following gives the correct sequence when mixing insulins? A. Inject air into the regular insulin vial and withdraw 10 units; then, using the same syringe, inject air into the NPH vial and withdraw 40 units of NPH insulin. B. Inject air into the NPH insulin vial, being careful not to allow the solution to touch the needle; next, inject air into the regular insulin vial and withdraw 10 units; then, withdraw 40 units of NPH insulin. C. Inject air into the regular insulin vial, being careful not to allow the solution to touch the needle; next, inject air into the NPH insulin vial and withdraw 40 units; then, withdraw 10 units of regular insulin. D. Inject air into the NPH insulin vial and withdraw 40 units; then, using the same syringe, inject air into the regular insulin vial and withdraw 10 units of regular insulin.

b. Regular or short-acting insulin should never be contaminated with NPH or any insulin modified with added protein. Placing air in the NPH vial first without allowing the needle to contact the solution ensures that the regular insulin will not be contaminated.

Ms. Hall has an order for hydromorphone, 2 mg, intravenously, q 4 hours p.r.n. pain. The nurse notes that according to Ms. Hall's chart, she is allergic to Dilaudid. The order for medication was signed by Dr. Long. Which of the following would be the correct procedure in this situation? A. Administer the medication; the doctor knows best. B. Call Dr. Long and ask that she change the medication. C. Ask the supervisor to administer the medication. D. Ask the pharmacist to provide a medication to take the place of Dilaudid.

b. The nurse is responsible for any medications he or she gives and must contact the doctor to inform her of the patient's allergy to the drug. The nurse should not give the medication and might speak with the supervisor only if he or she is uncomfortable with the physician's answer once she is notified. The nurse is legally unable to order a replacement medication, as is the pharmacist.

Which patient is most at risk for increased peristalsis? a.A 5-year-old child who ignores the urge to defecate owing to embarrassment b.A 21-year-old female with three final examinations on the same day c.A 40-year-old female with major depressive disorder d.An 80-year-old male in an assisted-living environment

b. a 21-year old female with three final examinations on the same day

THe nurse is preparing to perform a head and neck assessment of an adult client who has immigrated to the US for Cambodia. The nurse should first a. explain to the client why the assessment is necessary b. ask the client if touching the head is permissible c. determine whether the client desires a family member present d. examine the lymph nodes of the neck before examining the head

b. ask the client if touching the head is permissible

A student nurse is assisting with colon cancer screening at the local health care clinic. The student is completing fecal occult blood testing on the stool specimens. This test is also referred to as a(n) _____ test. a. melena b. guaiac c. amylase d. alkaline phosphatase

b. guiaic

A client visits the outpatient center with a complaint of sudden head and neck pain and stiffness. The client's oral temperature is 37.7C (100F). The nurse suspects the client is experiencing symptoms of a. migraine headache b. meingeal irritation c. trigeminal neuralgia d. otitis media

b. meingeal irritation

The nurse has attempted to administer a tap water enema for a patient with fecal impaction with no success. The fecal mass is too large for the patient to pass voluntarily. Which is the next priority nursing action? a.Preparing the patient for a second tap water enema b.Obtaining an order for digital removal of stool c.Positioning the patient on the left side d.Inserting a rectal tube

b. obtaining an order for digital removal of stool

(**)The nurse is emptying an ileostomy pouch for a patient. Which assessment finding will the nurse report immediately? a.Liquid consistency of stool b.Presence of blood in the stool c.Malodorous stool d.Continuous output from the stoma

b. presence of blood in the stool

A patient was involved in a motor vehicle accident and underwent a loop colostomy. The patient questions the nurse about what is draining out of each side of the colostomy. What is the nurse's best response? a. "There is stool draining out of both sides." b. "Stool is draining out the stomach side and mucus is draining from the rectum side." c. "There is mucus and stool draining from both sides." d. "There is stool draining out of the stomach side and nothing draining out of the rectum side."

b. stool is draining from the stomach side and mucus is draining from the rectum side

The nurse is preparing to assess the neck of an adult client. To inspect movement of the client's thyroid gland, the nurse should ask the client to a. inhale deeply b. swallow a small sip of water c. cough deeply d. flex the neck to each side

b. swallow a small sip of water

Which nursing intervention is most effective in promoting normal defecation for a patient who has muscle weakness in the legs? a.Administer a soapsuds enema every 2 hours. b.Use a mobility device to place the patient on a bedside commode. c.Give the patient a pillow to brace against the abdomen while bearing down. d.Elevate the head of the bed 20 degrees 60 minutes after breakfast while on bedpan.

b. use a mobility device to place the patient on a bedside commode

Hematuria

blood in urine

A patient has an abnormal, unexpected response to a drug. This is defined as which of the following? A. Drug tolerance B. A cumulative effect C. An idiosyncratic effect D. An anaphylactic reaction

c. Drug tolerance results when the body becomes accustomed to the drug over time. A cumulative effect occurs when the body cannot metabolize one dose of a drug before another one is given. An anaphylactic reaction is a life-threatening, immediate response to a drug.

The nurse manager on your unit prepared medications for Mr. Giles. She is called to the phone and asks you to give the patient his medications. Which is the best response to this request? A. Give Mr. Giles the medication and record it in his chart. B. Tell the nurse manager that you do not have time and ask her to get someone else. C. Tell the nurse manager that because you did not pour the medication, you cannot administer it. D. Give the medication to Mr. Giles but have the nurse manager chart it.

c. Nurses should never give medications prepared by someone else because they are responsible for what they administer.

The nurse takes an 8 a.m. medication to the patient and properly identifies her. The patient asks the nurse to leave the medication on the bedside table and states that she will take it with breakfast when it comes. What is the best response to this request? A. Leave the medication and return later to make sure that it was taken. B. Tell her that it is against the rules, and take the medication with you. C. Tell her that you cannot leave the medication but will return with it when breakfast arrives. D. Take the drug from the room and record it as refused.

c. Safe nursing practice requires that a medication never be left at the patient's bedside. It is not correct to say that the patient has refused medication in this situation.

A medication order reads: "K-Dur, 20 mEq PO b.i.d." When does the nurse correctly give this drug? A. Daily before bedtime B. By mouth every other day C. Twice a day by the oral route D. Once a week after recording an apical rate

c. The abbreviation "b.i.d." refers to twice-a-day administration.

A nurse discovers that she has made a medication error. Which of the following should be her first response? A. Record the error on the medication sheet. B. Notify the physician regarding course of action. C. Check the patient's condition to note any possible effect of the error. D. Complete an incident report, explaining how the mistake was made.

c. The nurse's first responsibility is the patient, and careful observation is necessary to assess for any effect of the medication error. The other nursing actions are pertinent, but only after checking the patient's welfare.

You are to administer a medication using a nasogastric tube. Before giving the medication, what should you do? A. Crush the enteric-coated pill for mixing in a liquid B. Flush open the tube with 60 mL of very warm water C. Check for proper placement of the nasogastric tube D. Take the patient's vital signs

c. Tube placement should always be checked before administering any medication to prevent the possibility of aspiration if the tube is not in the stomach. Enteric-coated pills should never be crushed, and very warm water may injure stomach mucosa. Taking vital signs is not necessary unless a particular medication requires it before administration.

Mr. King is receiving heparin subcutaneously. Which of the following demonstrates correct technique for this procedure? A. Aspirate before giving and gently massage after the injection. B. Do not aspirate; massage the site for 1 minute. C. Do not aspirate before or massage after the injection. D. Massage the site of the injection; aspiration is not necessary but will do no harm.

c. When giving heparin subcutaneously, do not aspirate or massage, so as not to cause trauma or bleeding in the tissues.

A nurse is checking orders. Which order should the nurse question? a.A normal saline enema to be repeated every 4 hours until stool is produced b.A hypertonic solution enema for a patient with fluid volume excess c.A Kayexalate enema for a patient with severe hypokalemia d.An oil retention enema for a patient with constipation

c. a kayexalate enema for a patient with severe hypokalemia

Which patient will the nurse assess most closely for an ileus? a.A patient with a fecal impaction b.A patient with chronic cathartic abuse c.A patient with surgery for bowel disease and anesthesia d.A patient with suppression of hydrochloric acid from medication

c. a patient with surgery for bowel disease and anesthesia

The nurse will anticipate which diagnostic examination for a patient with black tarry stools? a.Ultrasound b.Barium enema c.Endoscopy d.Anorectal manometry

c. endoscopy

The nurse observes a continual oozing of stool from the rectum of a patient who has been immobilized following surgery. The nurse recognizes that this condition most likely a result of which of the following? a. Diarrhea b. Flatulence c. Fecal impaction - double glove d. The Valsalva maneuver

c. fecal impaction (double glove!)

A nurse is assisting a patient in making dietary choices that promote healthy bowel elimination. Which menu option should the nurse recommend? a.Broccoli and cheese soup with potato bread b.Turkey and mashed potatoes with brown gravy c.Grape and walnut chicken salad sandwich on whole wheat bread d.Dinner salad topped with hard-boiled eggs, cheese, and fat-free dressing

c. grape and walnut chicken salad sandwich on whole wheat bread

A female client visits the clinic and tells the nurse that she wants to "stay healthy". The nurse observes that the client has diffuse neck enlargement, is perpiring, and is quite fidgety. The client tells the nurse that she is "hungry all the time, but I have lost weight." A priority nursing dagnosis for the client is a. imbalanced nutrition: less than body requirements related to excessive energy level and nervousness b. ineffective health maintenance related to increased metabolism, diaphoresis, and hunger c. health-seeking behaviors related to verbalization of wanting to stay healthy and concers over weight loss d. thyroid dysfunction related to neck swelling, perpiration, and fidgeting and concern over weight loss

c. health-seeking behaviors related to verbalization of wanting to stay healthy and concers over weight loss

While assessing an older adult client's neck, the nurse observes that the client's trachea is pulled to the left side. The nurse should a. ask the client to flex his neck to the left side b. observe whether the client has difficulty swallowing water c. refer the client to a physician for further evaluation d. palpate the cricoid cartilage for smoothness

c. refer the client to a physician for further evaluation

(**)The nurse is performing a fecal occult blood test. Which action should the nurse take? a.Test the quality control section before testing the stool specimens. b.Apply liberal amounts of stool to the guaiac paper. c.Report a positive finding to the provider. d.Don sterile disposable gloves.

c. report a positive finding to the provider

which nutrient is the body's most preferred energy source?

carbohydrate

vertebra prominens

cervical vertebra (C7) that can be easily palpated when the neck is flexed

A patient expresses concerns over having black stool. The fecal occult test is negative. Which response by the nurse is most appropriate? a."This is probably a false negative; we should rerun the test." b."You should schedule a colonoscopy as soon as possible." c."Are you under a lot of stress?" d."Do you take iron supplements?"

d. "Do you take iron supplements?"

The nurse is caring for a patient with abdominal pain. While obtaining a stool specimen for occult blood, the nurse notices that the specimen is black. The nurse recognizes that the color change may be the result of which of the following? a. Absence of bile b. Malabsorption of fat c. Diarrhea d. Iron supplements or GI bleeding

d. Iron supplements or GI bleeding

The medication order reads: "Hydromorphone, 2 mg IV every 3 to 4 hours p.r.n. pain." The prefilled cartridge is available with a label reading "Hydromorphone 2 mg/1 mL." The cartridge contains 1.2 mL of hydromorphone. Which of the following actions is correct? A. Give all the medication in the cartridge because it expanded when it was mixed B. Call the pharmacy and request the proper dose C. Refuse to give the medication D. Dispose of 0.2 mL correctly before administering the drug

d. Many cartridges are overfilled, and some of the medication needs to be discarded. Always check the volume needed to provide the correct dose with the volume in the syringe. Giving the excess medication in the cartridge may result in adverse effects for the patient. For this dose, it is not necessary to call the pharmacy or refuse to give the medication, provided the order is written correctly.

The health care provider orders a patient to have a fecal occult blood test. To obtain an accurate result, the nurse instructs the patient to do which of the following? a. Submit one sample for analysis. b. Take extra amounts of vitamin C supplements. c. Stop taking aspirin 14 days prior to the beginning of the test. d. Refrain from ingesting red meats for 3 days before testing.

d. Refrain from ingesting red meats for 3 days before testing

Which drug name is the name selected by the pharmaceutical company selling the drug and protected by trademark? A. Chemical name B. Generic name C.Official name D.Trade name

d. The chemical name identifies the drug's chemical composition and molecular structure. The generic name is assigned by the manufacturer who develops the drug. The official name is the name that identifies the drug in the official publication.

Why is the intravenous method of medication administration called the "most dangerous route of administration"? A. The vein can take only a small amount of fluid at a time. B. The vein may harden and become nonfunctional. C. Blood clots may become a serious problem. D. The drug is placed directly into the bloodstream, and its action is immediate.

d. The intravenous route is a direct access to the bloodstream, and medications act quickly when given intravenously. The condition of the veins is not as important as the rapid effect of the medication administered intravenously.

While assessing the head and neck of an adult client, the client tells the nurse that she has been experiencing sharp shooting facial pains that last from 10 to 20 seconds but are occuring more frequently. The nurse should refer the client for possible a. cancerous lesions b. arterial occlusion c. inner ear disease d. trigeminal neuralgia

d. trigeminal neuralgia

A patient refuses to take her noon medication, saying that she does not need it. Which of the following would be the best response? A. Tell her that she must take the medication because the doctor ordered it. B. Tell her that you went through a lot of preparation to get her medications ready, and it's the least she can do. C. Tell her that you don't care whether she takes the medications or not. D. Tell her that you will return the medications to the cart but would like to discuss her reasons for refusing to take the medications.

d. The patient has the right to refuse medications, but the nurse should assess the patient's reasons for refusal, document them, and report them to the physician.

(**)The nurse is managing bowel training for a patient. To which patient is the nurse most likely providing care? a.A 25-year-old patient with diarrhea b.A 30-year-old patient with Clostridium difficile c.A 40-year-old patient with an ileostomy d.A 70-year-old patient with stool incontinence

d. a 70-year old patient with stool incontinence

A patient will be undergoing abdominal surgeries, which will most likely result in an ostomy. The patient asks the nurse, "What will the stool from my ostomy look like?" What is the best answer? a. "Your stools won't change from what they currently are." b. "The consistency of your stools will be very soft." c. "The consistency of your stools will be liquid." d. "The consistency of your stools will depend on the location of stoma (ostomy)."

d. consistency will depend on the location of the stoma (ostomy)

The nurse is teaching a health class about the gastrointestinal tract. The nurse will explain that which portion of the digestive tract absorbs most of the nutrients? a.Ileum b.Cecum c.Stomach d.Duodenum

d. duodenum

Elevating the head of the bed to the maximum allowed amount of 30 degrees for a patient in balanced suspension traction helps to promote normal elimination by which of the following? a. Decreasing peristaltic movement b. Promoting contraction of the thigh muscles c. Strengthening the resistance of the internal and external sphincters d. Exerting increased pressure on the rectum

d. exerting increased pressure on the rectum

A patient is concerned about intermittent constipation and is confused about all the laxatives that are available. One of the laxatives that the patient has used in the past was mineral oil. The nurse explains that this type of laxative is an example of a(n) _____ laxative. a. stimulant b. osmotic agent c. emollient d. lubricant

d. lubricant

While assessing an adult client's head and neck, the nurse observes asymmetry in fromt of the client's ears lobes. The nurse refers the client to the physician because the nurse suspects the clent is most likely experiencing a/an a. enlarged thyroid b. lymph node abscess c. neurologic disorder d. parotid gland enlargement

d. parotid gland enlargement

(**) A patient with a hip fracture is having difficulty defecating into a bedpan while lying in bed. Which action by the nurse will assist the patient in having a successful bowel movement? a.Preparing to administer a barium enema b.Withholding narcotic pain medication c.Administering laxatives to the patient d.Raising the head of the bed

d. raising the head of the bed

(**)A patient has a fecal impaction. Which portion of the colon will the nurse assess? a.Descending b.Transverse c.Ascending d.Rectum

d. rectum

The nurse is preparing to assess the lymph nodes of an adult client. The nurse should instrut the client to a. lie in a supine position b. lie in a side-lying position c. stand upright in front of the nurse d. sit in an upright position

d. sit in an upright position

A client visits the clinic and tells the nurse that he is depressed because of a recent job loss. He complains of dull, aching, tight, and diffuse headaches that have lasted for several days, The nurse should recognize that these are symptoms of a. cluster headaches b. tumor-related headaches c. migraine headaches d. tension headaches

d. tension headaches

An older client visits the clinic accompanied by his daughter. The daughter tells the nurse that her father has been experiencing sever headaches that usually beginin the morning and become worse when he coughs. The client tells the nurse that he feels dizzy when he has the headaches. The nurse refers the client for futher evalus=ation because these symptoms are characteristic of a a. migraine headache b. cluster headache c. tension headache d. tumor-related headache

d. tumor-related headache

A patient calls the office to tell you that his or her urine has changed to an orange color. The best response of the nurse is which of the following?

did you recently start a new prescription for Pyridium?

Continent reservoir

distal end of ileum proximal of colon

A patient is to receive his daily isoniazid (INH) dosage for tuberculosis. He states he is feeling nauseated with this medication and refuses to take it. The nurse knows that the correct way to indicate this refusal is to:

document the reason for refusal along with the date and time in the patient's medical record. -the patient has the right to refuse medication. Refusals must be documented in the patient's record with the date, time, and reason for refusal if known

Pt. is to receive daily isoniazid (INH) dosage for TB. Pt. states nausea w/med and refuses to take it. Nurse correctly documents refusal by...

documenting reason w/ date and time in medical record

A 20-year-old female college student who suffers from frequent UTIs visits the student health clinic. The student asks how to decrease the frequency of UTIs. What is the nurses most appropriate response?

drink at least 6-8 glasses of water daily

The nurse is caring for a 45-year-old patient with a suspicious tumor in the bladder. The health care provider has ordered a procedure to identify the tumor tissue. Which test is done to collect tissue specimens?

endoscopy

UI from outside UT

functional incontinence

.A patient has just been diagnosed with diabetes mellitus. The patient voices concerns about possible kidney disease in the future. The patient asks, In which part of the kidney is urine produced? The nurses response is that urine is formed in the:

glomerulus

While examining your patients head and face, you determine that cranial nerve 1 is intact when the patient follows your...

identified a minty scent. Cranial nerve I, the olfactory nerve, controls the sense of smell. to test this nerves function, ask the patient to identify a nonirritating aroma, such as mint or coffee.

Common urinary elim. prob. is:

inability to store, or fully empty urine from bladder

80% of UTI result from the use of

indwelling catheters

A pt comes into the hospital with foul-smelling, cloudy urine and dysuria. What could possible be wrong

lower UTI

For which of the following inhalation med. delivery methods is it important for nurse to assess pt's ability to inhale deeply before administering meds?

nasal spray (WRONG) **dry powder inhaler

Pt. has been prescribed Flovent HFA inhaler w/ spacer. Pt. questions need for spacer. Correct response?

no spills (WRONG) **suspends the med better and distributes through lung better

A patient with a Foley catheter needs a urine sample for culture and sensitivity. What is the most appropriate action for the nurse to take

nsert a sterile blunt cannula in the catheter port to withdraw urine.

Incontinent reservoir

oreterostomy

involuntary loss of urine UI

overflow urinary incontince


Conjuntos de estudio relacionados

Unit 5 Chapter 9: Development Questions

View Set

Chapter 11 SB Question Humanities

View Set

Chapter 4 (Managing Marketing Information to Gain Customer Insights)

View Set

corporate communications - chapter 1,2,3,4,5,6

View Set

Business Law & Ethics Exam 1, Business Law Exam 1, blw 411 1-6, law exam 3, BLW 411 Ch 22 Final Exam, Bus Law 391, BLW Chapter 3 Practice, BLW 411 Ch 16 Final Exam, LEGAL AND SOCIAL ENVIRONMENT OF BUSINESS FINAL

View Set